*HURST REVIEW Qbank/Customize Quiz - Basic Care and Comfort, *HURST REVIEW Qbank/Customize Quiz - Adult Health, *HURST REVIEW Qbank/Customize Quiz - Management of care, *HURST REVIEW Qbank/Customize Quiz - Leadership, *HURST REVIEW Qbank/Customize Qu...

Lakukan tugas rumah & ujian kamu dengan baik sekarang menggunakan Quizwiz!

A client has a prescription for nitroglycerin gr 1/400 SL prn for angina pain. How many tablets should the nurse give the client? Use numbers and decimals only. Exhibit Medication Cart Drug Availability: Nitroglycerin Tablets USP 0.3 mg (1/200 gr)

Answer: 0.5 Rationale: Prescribed: nitroglycerin gr 1/400 Step 1 is to convert grains to mg (gr 1 = 60 mg) 1/400 = 1/400 x 60/1 = 60/400 = 3/20 = 0.15 mg Step 2: Think - 0.15 is ½ of 0.30. You want to give the equivalent of ½ tablet as needed. Step 3: D/H x Q = X 0.15/0.3 x 1 tablet = 0.5 tablet nitroglycerin SL

Which arterial blood gas value would the nurse expect to see when monitoring a client in a hyperosmolar hyperglycemic state (HHS). 1. pH 7.32 2. PaCO2 47 3. HCO3 22 4. PaO2 78

3. Incorrect: Normal HCO3 is 22-26. You would expect to see a normal bicarb level in this client. For a client in DKA, the expected HCO3 would be less than 22. 1. Incorrect: In HHS, the client will not be in acidosis. That is the primary difference between HHS and DKA. Normal pH is 7.35-7.45. A pH of 7.32 indicates acidosis and will be expected for a client in DKA. 2. Incorrect: Normal PaCO2 is 35-45. This is a high CO2 level that would be seen with DKA, not HHS. 4. Incorrect: Normal PaO2 is 80-100. An expected finding in HHS will be normal or increased PaO2, not decreased.

What action by the nurse is most helpful when responding to a bomb threat phone call? 1. Ask where and when the bomb is going to explode. 2. Quickly terminate the conversation and call in the bomb threat. 3. Document on the hospital Bomb Threat Checklist. 4. Immediately seek cover and warn others.

1. CORRECT. The nurse should keep the caller on the phone for as long as possible and try to obtain information, while being alert for voice characteristics and background noises. While keeping the caller on the line, the nurse should motion to another employee to call in the bomb threat. 2. INCORRECT. You should keep the caller on the line and signal someone to call in the threat. Keeping the caller on the line keeps them busy and may allow enough time for law enforcement to trace the call. 3. INCORRECT. It is important to document on the hospital Bomb Threat Checklist, but the most immediate action is to keep the caller on the line. 4. INCORRECT. While your initial response may be to run for cover, you should remain calm and not panic, continuing to keep the caller on the line.

While making rounds, the nurse discovers a small fire in a client's room. What should the nurse do first? 1. Remove the client from the room immediately. 2. Leave the client's room to obtain a fire extinguisher. 3. Instruct the unlicensed assistive personnel (UAP) to pull the fire alarm. 4. Evacuate all clients from the unit.

1. Correct: Rescue/Remove the client; first step in Rescue, Alarm, Contain, Extinguish (RACE). 2. Incorrect: Never leave the client in an unsafe environment. Remove the client from the area. 3. Incorrect: Not first action in RACE. Get the client out of the area first. The UAP may need to help you with this. Don't send the UAP away. 4. Incorrect: Not first action in RACE. Remove the client in immediate danger first. All clients may not have to be evacuated if the fire is contained and extinguished.

The nurse is reviewing the medication prescriptions with a client for which English is a second language (ESL). Which nursing intervention most likely will prevent a medication error with this client? 1. Use the teach-back method so that client is repeating the instructions back to the nurse. 2. Give printed information to the client. 3. Ask the client if they have questions before the client leaves the healthcare setting. 4. Refer medication questions to the pharmacist.

1. Correct: The teach-back method of asking the client to repeat the teaching instructions to the nurse will most likely reveal any misunderstanding. This allows the nurse to reinforce any areas where clarification is needed. 2. Incorrect: Printed information may or may not be helpful, depending on the client's level of understanding. 3. Incorrect: The client may not know which questions to ask regarding the medication, particularly if there is a language barrier. 4. Incorrect: The client may not ask another person for help. There has been no relationship established with the pharmacist since the nurse has been providing the teaching. The nurse should not put this responsibility on someone else in the interdisciplinary team.

The parents of a 4 year old child have recently had a new baby and the parents report that the 4 year old had been dry all night for 8 months and is now wetting the bed again. What should the nurse assess first? 1. Urinalysis 2. Normal urination habits. 3. Adjustment to the new baby. 4. Fluid intake after 6 pm.

3. Correct: Regression is the cause 4 is young to see urinary problems. 2. Incorrect: Assessing the normal urination habits is not first. Assessing the urinalysis is priority. 3. Incorrect: Regression is the likely cause. 4. Incorrect: The child's fluid intake may be too high after 6 pm, but ruling out a urinary tract infection is the first assessment and requires immediate treatment if there is an infection.

What medications should the nurse anticipate the primary healthcare provider prescribing for the client with portal hypertension and bleeding esophageal varices associated with advanced cirrhosis? Select all that apply 1. Oxygen 2. Clopidogrel 3. Propranolol 4. Vitamin K 5. Lactulose

1., 3., 4., & 5. Correct: We know that they need oxygen because they may have been bleeding. Propranolol acts to reduce portal venous pressure and reduce esophageal varices bleeding. Vitamin K is a clotting factor and helps to correct clotting abnormalities because of the damaged liver. Lactulose decreases what? Ammonia, which is elevated with cirrhosis. 2. Incorrect: You don't want to give them a platelet aggregation inhibitor. They are already bleeding.

Which discussion points should a nurse plan to include when teaching a group of college students on prevention of sexually transmitted infections (STI)? Select all that apply 1. Safe sex practices 2. Routine human immunodeficiency virus (HIV) testing 3. Proper use of birth control pills 4. Sexual abstinence 5. Vaccinations for STIs

1., 4. & 5. Correct: All of these topics should be included when discussing prevention of STIs. Safe sex practices include proper use of condoms. Abstinence is the best way to prevent STIs. Vaccines are available for some STIs such as human papillomavirus vaccine (HPV). 2. Incorrect: Routine HIV testing is not a way to prevent HIV or other STIs. It will provide early diagnosis. The best course of action is to prevent occurrence. 3. Incorrect: Birth control pills help prevent unplanned pregnancy. STIs can still be contracted if proper safe sex techniques are not implemented.

The nurse is caring for a client on the surgical unit. Which prescriptions could the nurse safely administer to the client? Select all that apply 1. Chlordiazepoxide 10 mg p.o. q 4h p.r.n. for agitation 2. Regular insulin 10 U stat 3. MS 2 mg IVP every 2 hours as needed for pain 4. Cefepime 1 gram IVPB every 8 hours 5. Diphenhydramine 25 mg p.o. hour of sleep for three nights

1., 4. & 5. Correct: These medication prescriptions are correctly written following approved Joint Commission abbreviations. 2. Incorrect: The "U" can be mistaken for "0" (zero), the number "4" (four) or "cc." Units should be written out completely. 3. Incorrect: MS can mean morphine sulfate or magnesium sulfate. Write "morphine sulfate." Write "magnesium sulfate."

The primary healthcare provider's prescription for a client instructs the nurse to give digoxin 0.125 mg intravenously as a one-time dose. The available medication is in a concentration of 0.5 mg/2 mL. How many milliliters should the nurse give? Round answer using one decimal point.

____ mL= 2 mL x 0.125 mg / 0.5 mg = 0.5 mL

The nurse is monitoring the IV medications that a client is receiving by an IV infusion pump. How many milligrams per hour of epinephrine should the nurse determine that the client is receiving? Round to the second decimal place. Use numbers only to answer. Exhibit IV Infusion Pump: Infusing 15.0 mL/h (1.0 mcg/min) Epinephrine

Answer: 0.06 Rationale: Step 1: Determine how many micrograms there are in 1 hour. 1.0 mcg/ min x 60 minutes = 60 mcg/hour. Step 2: Convert micrograms to milligrams. There are 1000 mcg in 1 mg. Step 3: Calculate micrograms There are 1000 mcg in 1 mg. How many mg are in 60 mcg? 1000 mcg/1 mg = 60 mcg/x mg 1000x = 60 1000x/1000 = 60/1000 x = 0.06 mg/hour

Which food items, if chosen by a new unlicensed assistive personnel (UAP), would indicate to the nurse that the UAP understands a clear liquid diet? Select all that apply 1. White grape juice 2. Gelatin 3. Vanilla pudding 4. Lemon Popsicle 5. Fat free Broth 6. Tea with honey

1., 2., 4., 5., & 6. Correct: A clear liquid diet is made up of only clear fluids and foods that are clear fluids when they are at room temperature. These choices are considered to be clear liquids. 3. Incorrect. This is considered appropriate for a full liquid diet.

A client who needs to have a stool specimen for an occult blood test is instructed by the nurse to avoid which substances two hours prior to testing? Select all that apply 1. Liver 2. Tomato 3. Ibuprofen 4. Sardines 5. Ascorbic acid

1., 3., 4., 5. Correct: The following foods can cause a false-positive reading: red meats, liver, turnips, broccoli, cauliflower, melons, salmon, sardines, and horseradish. Medications altering the test include aspirin, ibuprofen, ascorbic acid, indomethacin, colchicines, corticosteroids, cancer chemotherapeutic agents, and anticoagulants. Ingestion of vitamin rich foods can cause a false negative result. 2. Incorrect: A tomato is not on the food list for false-positive reading and do not have to be avoided.

Which food item would the nurse include when planning diet instructions to promote bone growth for a client with a broken tibia? 1. Lettuce 2. Apples 3. Yogurt 4. Green beans

3. Correct: The serving size of 150 g of yogurt has a calcium content of 240 mg. 1. Incorrect: The serving size 50 g of lettuce has a calcium content of 19 mg. 2. Incorrect: The serving size of 182 g of apple has a calcium content of 11 mg. 4 Incorrect: The serving size of 90 g cooked green beans has a calcium content 50 mg.

A 3 year old child is being treated for asthma. The child weighs 31.5 lb (14.3 kg). The primary healthcare provider has prescribed Albuterol syrup 5 mg PO every 8 hours. What action should the nurse take? Exhibit: Albuterol Classification: Beta 2 Agonist Dosing: 2-6 years: 0.1 mg (0.25 mL)/kg PO q8hr initially, not to exceed 2 mg (5 mL) q8hr; if necessary, may be increased to 0.2 mg/kg PO q8hr, not to exceed 4 mg (10 mL) q8hr 1. Administer the dose immediately to relieve respiratory efforts. 2. Split the dose in two equal parts and administer every 4 hours. 3. Notify the charge nurse that the child needs a different type medication. 4. Notify the primary healthcare provider.

4. Correct: The nurse is responsible for assuring that medications are administered safely. One aspect of safe medication administration is to assure that the prescribed dose is safe for administration. For this age child, the maximum dose that should be administered is 0.2 mg/kg PO every 8 hours. So, we need to determine how much albuterol this child, weighing 14.3 kg can safely have. To calculate this, we would first use the child's weight in this recommended dose formula: The maximum amount is 0.2 mg X 14.3 kg = 2.86 mg every 8 hours. The exhibit also says not to exceed 4 mg every 8 hours, which the prescribed amount exceeds. Remember, the maximum dose based on the individual child's weight is the safest guideline to use. The prescription for this child is to receive 5 mg PO every 8 hours. Is this a safe dose? No! Therefore, the nurse should not administer the medication and should promptly notify the primary healthcare provider. Administering the prescribed dosage could be dangerous to this child! 1. Incorrect: Although this option for administering the dose immediately to relieve respiratory efforts sounds good, could we cause more harm to the client by administering this dose of albuterol? Yes! Relieving respiratory efforts is a goal of therapy, but should be accomplished using safe dosages of the medication. This dose of medication should not be administered. 2. Incorrect: You may have realized that the single dose as prescribed was too much. But would splitting the dose in two equal parts and administering it every 4 hours be safe? No! First of all, the maximum safe dose would still be exceeded. And, as a nurse, it would be out of your scope of practice to alter an existing prescription. 3. Incorrect: What came to your mind when you looked at this? Did you consider if the medication is appropriate for use in a child with asthma? Albuterol acts as a bronchodilator, causing relaxation of the bronchial smooth muscles in the airways and is used to manage asthma and acute bronchospasm. The exhibit provides safe dosing information for a 3 year old child, so it is an appropriate agent for use in this child with asthma. Notifying the charge nurse that the child needs a different type medication would not be appropriate.

What would be the best way for the nurse to evaluate the effectiveness of fluid resuscitation during the emergent phase of burn management? 1. Weight increases by 2 pounds in 24 hours 2. Urinary output is greater than fluid intake 3. Blood pressure is 90/60 mmHg 4. Urine output greater than 35 mL/hour

4. Correct: Urine output of 30 to 50 mL/hour indicates adequate fluid replacement. 1. Incorrect: May indicate fluid retention. 2. Incorrect: Does not indicate fluid balance. 3. Incorrect: Blood pressure alone does not indicate adequate fluid balance.

The nurse is preparing to administer 1000 mL D5W with 40 mEq KCL IV over 12 hours. How many gtts/min will the nurse need to set the IV rate at? Exhibit IV Tubing: 10 Drops/mL

Answer: 14 Rationale: Prescription: 1000 mL D5W with 40 mEq KCL IV over 12 hours. Step 1: Determine what the drop factor is on the IV tubing. The one shown in the exhibit has a drop factor of 10 drops/mL. Step 2: Remember the formula for gtts/min = mL/hr x drop factor/time in minutes = gtts/min Step 3: 1000 ml/12 hours x 10 gtts/1 mL x 1 hour/60 min = 1000/12 x 10/1 x 1/60 = 10000/720 = 13.8 or 14 gtts/min

Which is the correct method for removing personal protective equipment (PPE)? 1. Contaminated gloves should be removed in the client's room. 2. The glove that is removed first should be placed in the wastebasket before the other glove is removed. 3. Remove face shield or goggles first. 4. Shoe covers should be removed last.

1. Correct: Avoid contaminating self, others, or environment when removing equipment. 2. Incorrect: The first glove is held in the still gloved hand and the second glove is slid over the first removed glove. 3. Incorrect: Hand hygiene is performed before removing face shield or goggles. 4. Incorrect: Shoe covers are removed with gloved hands.

The nurse is monitoring the infection risk in a client that is to begin chemotherapy. Which activity should alert the nurse that the client is at a higher risk for infection? Select all that apply 1. Enjoys getting manicures and pedicures every two weeks. 2. Loves to go with the children to the local water park. 3. Relaxes in hot tubs when traveling. 4. Selects steamed vegetables as part of routine dietary intake. 5. Prefers to go barefooted when at home. 6. Keeps cats in the home and cleans the litter boxes once a week.

1., 2., 3., 5., & 6. Correct: Infection is one of the most common life-threatening complications associated with cancer and chemotherapy. You know that both the cancer and chemotherapy weakens the immune system. Therefore, clients on chemotherapy should be familiar with activities that should be avoided due to the risk of infection with the immunosuppressed state. There are several things that are known to increase the risk of infection in these clients. Did you pick up on these? Well, let's look at a few of these. Clients on chemo should not get manicures or pedicures at salons or spas and should avoid having false nails or nail tips applied. There is too great a risk of contamination at the public salons, so clients are encourage to use their own personal and well-cleaned tools for nail care at home. Another source of bacterial contamination is public water parks. Although these parks take measures to reduce the risk of infection to the general public, the risk is too great for a client on chemo. Swimming can result in accidental ingestion of water which increases the risk of cryptosporidium or other waterborne pathogens. Same thing applies to hot tubs. ​So why is going barefoot at home such a big deal? Well, this increases the risk of cuts, scrapes, or other injury that would increase the portal for infectious agents to enter. In addition, the exposure to potential infectious agents is greater. The oncologist may direct the client in the best way to deal with this client having cats in the home and cleaning the litter box due to the risk of exposure to bacteria and parasites. If allowed to clean the litter box, latex or rubber gloves, along with a mask over the nose and mouth is generally recommended to reduce the risk of infection. In addition, the client should be instructed to thoroughly wash the hands with soap and water after cleaning the litter box or after touching the cats. 4. Incorrect: Although the intake of fresh fruits and vegetables has been controversial, most agree that if washed properly, even fresh fruits and vegetables can be consumed. However, the oncologist should be the one to approve the dietary intake of these. Here, we have vegetables that have not only been washed, but steamed as well. These should be safe for consumption for clients on chemo.

What intervention should the nurse take when providing oral care for the unconscious client? 1. Brush teeth with a stiff toothbrush. 2. Use thumb and index finger to hold the client's mouth open while brushing teeth. 3. Position the client on their side. 4. Rinse by injecting water into the center of client's mouth.

3. Correct: Placing client on side helps fluid run out of the mouth. 1. Incorrect: A soft bristled brush should be used. 2. Incorrect: Fingers should not be placed in client's mouth. 4. Incorrect: Should be injected into the sides of the client's mouth.

The nurse is teaching a group of clients who have osteoarthritis how to protect joints. What should the nurse include? Select all that apply 1. Use small joints and muscles. 2. Turn door knobs clockwise. 3. Sit in a chair that has a low, straight back. 4. Push off with the palms of hands when getting out of bed. 5. Use hairbrush with extended handle.

4. & 5. Correct: Pushing off with the palms of the hands is using a larger joint and muscles. Using the fingers will cause more joint injury. Use long handled devices such as a hairbrush with an extended handle to decrease stress on joints (in this case the wrist). 1. Incorrect: Larger joints and muscles can take more stress and weight than smaller ones. Using small joints again and again puts more stress on them and may lead to deformity. Try to spread the strain and weight over several joints. This helps you use each part of your body to its best advantage. 2. Incorrect: Do not turn a door knob clockwise. Turn it counterclockwise to avoid twisting the arm and promoting ulnar deviation. 3. Incorrect: Sit in a chair that has a high, straight back. This will provide more support for the back.

The nurse is caring for a client who has a history of sleep apnea. The client is scheduled for a colon resection the following morning and asks if the sleep apnea machine should be brought to the hospital. What is the nurse's best response? 1. Yes, bring the sleep apnea machine. 2. No, do not bring the sleep apnea machine. 3. It is your choice. 4. Call your primary healthcare provider.

1. Correct: A client with sleep apnea is at risk for cardiac and respiratory complications post op due to decreasing oxygenation. So yes, the client needs to use the CPAP machine. Remember this client will also be receiving narcotics for pain and have a decreased activity level as well. All of these things can decrease oxygenation. 2. Incorrect: The client will need to have the machine after surgery. 3. Incorrect: Best response is for nurse to recommend that the client bring machine. 4. Incorrect: The nurse can answer this question.

The nurse has just received a client from the special procedures lab for a liver biopsy. What is the position of choice for this client post procedure? 1. Fowler's 2. Right side 3. Left side 4. Prone

2. Correct: How do you stop bleeding from a puncture site? With pressure, right? Yes. So where is the liver? In the right upper abdomen under the rib cage. So position the client on the right side so that pressure is applied to the liver's puncture site. Then apply pressure with a sand bag or rolled up towel. This will help to stop bleeding. 1. Incorrect: This will not help control the bleeding. Pressure needs to be applied to the liver, so we want the liver coming forward toward the abdominal wall and pressure to be applied with a sand bag or rolled up towel. 3. Incorrect: The liver is on the right, not the left. Without the liver next to the abdominal wall, pressure cannot be exerted on the liver's puncture site. 4. Incorrect: We don't turn clients onto abdomen. You will not be able to assess for bleeding with the client in this position.

A nurse manager has recognized that nurses on one shift do not seem to be working well together and, on occasions, refuse to help each other when needed. What strategy could the nurse manager use that would help with team building? Select all that apply 1. Avoid discussing conflicts to build a positive work environment. 2. Model behaviors that create a caring environment and promote trust. 3. Create a shared vision of the unit and agency mission and purpose. 4. Recognize nurses who demonstrate commitment to team efforts. 5. Make nurses aware of the messages that their behaviors send to the team. 6. Have nurses agree upon roles, responsibilities, and proper lines of communication.

2., 3., 4., 5., and 6. Correct: The nurse manager needs to incorporate strategies that are effective in team building. One important thing that a nurse manager can do when trying to get nurses to work as a team is to actually model behaviors that promote trust and create a caring environment for not only the clients, but also the nurses and other staff as well. Trust is a cornerstone when trying to build team relationships. In order for nurses to recognize a need for teamwork and reduce conflict, they should have a clear understanding of the unit and agency mission and purpose. The unit manager should assure that this is clearly documented and articulated to the nurses and staff on the unit. The nurse manager should help each nurse and staff member understand how they fit into the overall purpose and goals of the unit and agency. We all know that recognition tends to foster positive behaviors. The nurse manager should recognize nurses who demonstrate commitment to team efforts. This can be done with tangible or nontangible rewards. So, why should nurses be made aware of the messages being sent to the other team members by their behaviors? These nurses may not realize how their unwillingness to work as a team negatively impacts the healthcare team as a whole. They may think that as long as they take care of their clients the way that they want to, everything should be fine. Nurse managers can help nurses to see how their behaviors affect client care and team relations. Once the nurses have agreed upon the roles and responsibilities as part of the healthcare team and understand the lines of communication, they are more likely to follow through with these. Communication by the nurse manager will be crucial in carrying out this team building strategy where all team members agree upon what needs to be accomplished and who to communicate with along the way. 1. Incorrect: It is the nurse manager's responsibility to address the conflict and issues that arise. Failure of the nurse manager to address conflicts within the workplace often fuels more conflict. In addition, the team members often lose respect for the nurse manager who does not discuss and help to resolve the issues. Conflict avoidance can have long term effects on the nursing unit and the agency and can stifle productivity and success of the unit.

A factory employee is brought to the emergency room on first shift with a severe hand laceration occurring at work. The employee is quite upset, indicating previous competency on the machine. When reviewing medications, the nurse notes the client has recently started alprazolam at bedtime. What vital information about this medication should the nurse provide to the client? 1. Consider getting new glasses. 2. Stand up slowly when sitting. 3. Do not operate dangerous machines. 4. Instructions for taking medication appropriately.

3. CORRECT: The vital information provided when a client starts any benzodiazepine includes no driving and no operating heavy machinery. The major side effects of this category of drugs include trouble concentrating, impaired coordination, drowsiness and fatigue, all of which may have contributed to this client's accident. The fact the client uses this drug for sleep and then goes to work indicates a lack of comprehension about side effects. 1. INCORRECT: There is no indication the client even wears glasses; however, glasses would not address the problem of slowed reflexes or poor coordination when working around even familiar machinery. 2. INCORRECT: Although standing up slowly is an important safety issue of which the client should be aware, this is not the most vital teaching the nurse should present. 4. INCORRECT: The question does not indicate the client's present dose of alprazolam nor the frequency. It is impossible to determine whether the client is taking the medication correctly.

The nurse is preparing to administer nadolol to a hospitalized client. Which client data would indicate to the nurse that the medication should be held and the primary healthcare provider notified? 1. Blood pressure 102/68 2. Glucose 118 mg/dL 3. UOP 440 mL over previous 8 hour shift. 4. Heart rate 56/min

4. Correct: This is a beta blocker. If a client's heart rate is less than 60 beats per minute, notify the primary healthcare provider and ask if the client should receive this medication. You can identify that nadolol is a beta blocker because it ends in "lol". 1. Incorrect: Beta blocker are prescribed to lower BP. When the baseline BP is not known, worry about a BP of 90/60 or below. If the client's BP drops below 90/60, this beta blocker should be held and the primary healthcare provider notified. 2. Incorrect: This is a normal glucose level. If the client is a diabetic, beta blockers can mask the signs of hypoglycemia. 3. Incorrect: Urinary output is adequate. Beta blockers do not alter renal function.

A client receiving treatment for hypertension is scheduled to receive hydrochlorothiazide 25 mg orally. Based on the label on the bottle, how many tablets should the nurse administer? Exhibit Hydrochlorothiazide Tablets, USP 50 mg Each tablet contains: Hydrochlorothiazide USP 50 mg

The prescription is for 25 mg. Strength available is 50 mg/tab. 50 mg /1 tab = 25 mg/x tab 50 mg x = 25 mg tab 50 mg x/ 50 mg = 25 mg tab/50 mg X = 0.5 tablets.

The nurse is caring for a burn client in the emergent phase. The client becomes extremely restless while on a ventilator. What is the priority nursing assessment? 1. Patency of endotracheal tube. 2. Adventitious breath sounds. 3. Fluid in the ventilator tubing. 4. Ventilator settings.

1. Correct: With restlessness, think hypoxia so the nurse should start assessment with airway first. Check for patency of the ET tube. If this is patent, then the other options would be next. 2. Incorrect: This is the next best answer, but hypoxia and airway comes first. 3. Incorrect: This is the third step. Rule out the other two before checking tubing for kinks or obstructions. 4. Incorrect: Start with the client first. Then move toward the ventilator. Always assess the client first.

The charge nurse delegates a licensed practical nurse (LPN) to perform an intervention that is not within the scope of practice for the LPN. Which response by the LPN is appropriate in response to the inappropriate delegation? 1. Notify the primary healthcare provider. 2. Refuse the delegated intervention. 3. Discuss the assignment with another LPN. 4. Ask the charge nurse to evaluate the intervention.

2. Correct: The LPN has the right to refuse a delegated intervention that is not within the scope of practice for the LPN. The charge nurse should be informed that the delegated intervention is not appropriate according to the state's Nurse Practice Act. 1. Incorrect: There are situations in which the LPN must notify the primary healthcare provider. This is not a situation that requires the LPN to notify the primary healthcare provider. 3. Incorrect: Discussing the assignment with another LPN is delaying the client's needed intervention. The LPN should refuse the intervention. 4. Incorrect: The charge nurse cannot change the scope of practice for the LPN by evaluating the intervention. Only the state Board of Nursing can legally determine the LPN's scope of practice.

A nurse is performing eye care for an unconscious client. Which interventions should the nurse include? Select all that apply 1. Administer moist compresses to cover eyes every 2 hours. 2. Clean eyes with saline and cotton balls, wiping from outer to inner canthus. 3. Use a new cotton ball for each cleansing wipe. 4. Instill artificial tears into the lower eyelids as prescribed. 5. Protect the eyes with a protective shield. 6. Monitor eyes for redness and exudate.

1., 3., 4., 5., & 6. Correct: All of these interventions are appropriate for eye care of the comatose client. These actions prevent infection, keep eyes moist, and protect the eye from injury. 2. Incorrect: Clean the eyes with saline solution and cotton balls. Wipe from the inner to outer canthus. This prevents debris from being washed into the nasolacrimal duct.

A nurse is triaging a 2 year old child in the pediatric emergency department. The nurse notes that the child will not lie down and is consistently drooling. A croaking sound is heard on inspiration. What is the priority nursing intervention? 1. Examine the oral pharynx using a tongue depressor. 2. Administer a sedative so the child can be examined. 3. Have a second nurse hold the child down for the assessment. 4. Notify the primary healthcare provider immediately.

4. Correct: This is the safest answer. The child could suddenly obstruct the airway upon examination of throat. 1. Incorrect: If it looks like epiglottitis, do not examine as this could cause sudden airway obstruction which could be fatal. 2. Incorrect: The client is having trouble breathing, so do not sedate the client. Sedatives would depress the respirations more and potentially cause the client to go into respiratory arrest. Remember, the NCLEX® lady does not want you to be a killer nurse. 3. Incorrect: This will cause more respiratory and emotional distress to the child. This is an unsafe answer.

A licensed practical nurse (LPN) on the Labor and Delivery unit is assisting the nurse with multiple admissions. What tasks could the LPN complete until the nurse is available? Select all that apply 1. Take initial vital signs. 2. Measure cervical dilation. 3. Check fundal height and fetal heart rate (FHR). 4. Obtain urine for protein and glucose. 5. Collect vaginal swab to test for chlamydia.

1, 4, & 5. Correct: The LPN scope of practice varies from state to state, although basic tasks are consistent. Taking vital signs, even initially, is among the tasks that can be delegated to the LPN. Other appropriate duties include collecting urine for ordered tests and even obtaining a vaginal swab. These can definitely be delegated to a licensed practical nurse. 2. Incorrect: Measuring cervical dilation is an invasive assessment not within the LPN scope of practice. An experienced registered nurse or primary healthcare provider must be specifically trained to perform this procedure. 3. Incorrect: Fundal height is a determination of uterine size to assess fetal growth and development which cannot be delegated to an LPN. Additionally, determining fetal heart rate involves assessment of fetal well being and not within the LPN scope of practice.

A primipara at 36 weeks gestation is seen in the OB/GYN clinic. Which sign/symptom should the nurse immediately report to the primary healthcare provider? 1. Puffy hands and face 2. Reports indigestion 3. Pedal edema 4. Trace proteinurea

1. Correct: Facial and upper extremity edema can be a sign of preeclampsia, which can endanger both the mother and fetus. Preeclampsia is a pregnancy complication characterized by high blood pressure and signs of damage to another organ system, often the kidneys. Preeclampsia usually begins after 20 weeks of pregnancy in a woman whose blood pressure had been normal. Even a slight rise in blood pressure may be a sign of preeclampsia. Left untreated, preeclampsia can lead to serious, even fatal, complications. Signs and symptoms of preeclampsia include hypertension and may include: Proteinuria; Severe headaches; Changes in vision; Upper abdominal pain; Nausea or vomiting; Decreased urine output; Thrombocytopenia; Impaired liver function; Shortness of breath; Sudden weight gain, and edema, particularly in face and hands. 2. Incorrect: Indigestion should be assessed for severity, but it is a common symptom in 3rd trimester of pregnancy. 3. Incorrect: Pedal edema should be assessed but is common in 3rd trimester of pregnancy. 4. Incorrect: Trace proteinuria is a benign sign in 3rd trimester of pregnancy.

The nurse is transferring the client from the bed to the wheelchair. Which nursing intervention would the nurse implement after assisting the client to a sitting position on the side of the bed. 1. Assess the client for lightheadedness. 2. Move the wheelchair closer to the bed. 3. Lower the bed to the lowest position. 4. Position the foot of the stronger leg closer to the bed.

1. Correct: Prior to moving the client from the side of the bed to the wheelchair, assess the client for orthostatic hypotension or postural hypotension. The client may experience a sudden decrease in blood pressure after changing the position from lying down to sitting up. 2. Incorrect: The wheelchair should be positioned in the correct position prior to positioning the client on the side of the bed. Client safety has priority. The nurse should not leave the client to move the wheelchair to the bedside. 3. Incorrect: The bed should have been lowered to the position prior to moving the client to the side of the bed. The client's feet should rest on the floor. This will assist the client in supporting themselves. 4. Incorrect: Positioning the foot of the stronger leg closer to the bed is a transfer step after assessing the client for orthostatic hypotension. Whether the stronger or weaker leg is positioned closer to the bed will not affect the client's blood pressure status.

The nurse is caring for a client following a transurethral resection of the prostate (TURP). The client has a 3 way irrigation catheter in place. Which finding would indicate the need to increase the irrigation rate? 1. Clots in urine 2. Unable to palpate bladder 3. Slightly pink tinged urine 4. No report of bladder spasms

1. Correct: The irrigation should be increased if you see clots in order to keep the catheter patent. 2. Incorrect: If you are unable to palpate the bladder, then it is not distended. That is good. Bladder pressure would mean that the indwelling urinary catheter is obstructed. Then you would either increase flow or manually irrigate catheter to ensure patency and no retention of fluid in the bladder. 3. Incorrect: The irrigation is regulated so that the urine is free of clots and slightly pink tinged. This is an indication that the irrigation rate is appropriate. 4. Incorrect: Absence of bladder spasms means there are no clots occluding the catheter.

A newly appointed nurse manager on the unit has a stable staff who have worked together for 5 or more years. The unlicensed assistive personnel (UAPs) are accustomed to informally arranging their lunch time; however, the nurse manager has implemented a plan to assign breaks and lunch. The UAPs are angry and refuse to change to the new system. What should be the nurse manager's first action in this situation? 1. Plan a unit staff meeting to discuss the problem and receive input for resolution. 2. Inform the staff that the plan will be implemented and those not following the plan will be disciplined. 3. Ask the charge nurse to address the problem daily as it occurs. 4. Plan a meeting with all UAPs to discuss the problem and reason for the new assignments.

1. Correct: The key word in the stem is first. So yes, get everyone together and discuss the problem and find areas of compromise where possible. 2. Incorrect: Too authoritative. This is good staff that has worked together on the unit for a long time. We want them to be happy and get the work done. Again, the key word in the stem is first. 3. Incorrect: No, this is a manager's issue resulting from a new system. This may need to be done but is not the first action. 4. Incorrect: Explaining the rationale to one group does not promote teamwork. It is better to plan a unit staff meeting and not a meeting for only the UAPs.

The nurse tries to notify the primary healthcare provider (PHP) that the dosage of newly prescribed medication is higher than recommended. The PHP cannot be located and the medication is scheduled to be administered in 30 minutes. Which intervention should the nurse implement next? 1. Inform the charge nurse. 2. Administer the medication as prescribed. 3. Document the prescribed medication dosage in the nursing notes. 4. Administer the recommended dosage until the PHP is contacted.

1. Correct: The medication that was prescribed is higher than the recommended dosage. Since the PHP cannot be notified, the charge nurse should be notified to clarify what is the next action for the nurse to initiate. 2. Incorrect: It is beyond the scope of practice for a nurse to independently decide to administer a medication dosage that is higher than recommended. The nurse should not administer the medication. 3. Incorrect: The prescribed medication that is higher than recommended should not be documented in the nursing notes. The medication has not been administered, and the nurse is following the appropriate steps to clarify the dosage of the medication. 4. Incorrect: Client safety is always a priority. It is beyond the scope of practice for a nurse to independently decide to administer a different medication dosage without a prescription from the PHP. The nurse should not administer the medication.

What should the nurse include in the teaching plan for a client receiving external beam radiation? Select all that apply 1. Small marks will be placed on the skin to mark the treatment area. 2. Lotion may be used around the treatment area to decrease dryness. 3. The radiation therapist can see, hear, and talk with you at all times during treatment. 4. Stay away from babies for 24 hours. 5. You will have to hold your breath during radiation treatment.

1., & 3. Correct: Small ink marks or small tattoos will be placed on the skin to mark the treatment area. Do not remove the marks. The radiation therapist can see, hear, and talk to the client at all times during treatment. Relieve anxiety by letting client know he/she is not alone. 2. Incorrect: The client should avoid the use of potential irritants (perfume, powders or cosmetics) on the skin in the treatment field. 4. Incorrect: The client receiving external beam radiation is not radioactive and will not radiate others. The client can safely be around other people, babies, and children. Clients who are undergoing brachytherapy or receiving radio pharmaceuticals require that you be aware that the client is emitting radioactivity. 5. Incorrect: The client will need to stay very still so radiation goes to the exact same place each time but can breathe as always. The client does not have to hold breath.

The nurse is teaching a community education course regarding complementary and/or alternative therapies. Which therapies would the nurse include in the course as complementary and/or alternative therapies? Select all that apply 1. Acupuncture 2. Yoga 3. Tai chi 4. Reiki 5. Zumba

1., 2, 3, & 4. Correct: All are considered complementary and/or alternative therapies. Acupuncture involves stimulating specific points on the body. This is most often done by inserting thin needles through the skin, to cause a change in the physical functions of the body. Research has shown that acupuncture reduces nausea and vomiting after surgery and chemotherapy. It can also relieve pain. The practice of yoga makes the body strong and flexible, and improves the functioning of the respiratory, circulatory, digestive, and hormonal systems. Yoga brings about emotional stability and clarity of mind. Tai chi is an ancient Chinese discipline involving a continuous series of controlled usually slow movements designed to improve physical and mental well-being. Reiki is a healing technique based on the principle that the therapist can channel energy into the client by means of touch, to activate the natural healing processes of the body and restore physical and emotional well-being. 5. Incorrect: Zumba is a type of dance exercise and is not considered a form of alternative therapy.

An unconscious client is admitted to the ICU with a closed head injury suffered in a fall. Despite aggressive efforts, the client expired within 24 hours. The nurse must complete postmortem care while awaiting the coroner. The nurse knows what action is not appropriate in this situation? Select all that apply 1. Remove indwelling catheter 2. Disconnect the ET tube from ventilator 3. Remove hospital ID band 4. Cap all intravenous lines 5. Wash body head to toe

1., 3., & 5.CORRECT. The client expired of injuries within 24 hours of being admitted to the hospital, which requires investigation by a coroner. It must be determined if death resulted from fall injuries, or whether any action, or lack thereof, by medical personnel contributed to the client's demise. When completing postmortem care on a "coroner's case," the nurse must leave all invasive lines and tubes in place for investigative purposes. Therefore, it would not be appropriate for the nurse to remove the foley catheter, although the urine can be emptied from the bag. It is also incorrect to remove any hospital identification bands. Washing the body should never be done since evidence could be disturbed or even removed. 2. INCORRECT. This action is acceptable since the client would not be transported while still attached to a ventilator. However, the endotracheal tube itself must remain taped in place when the client is transported to the coroner's facility. 4. INCORRECT. It is not necessary to leave the IV bags attached to intravenous catheters. The tubing and bags may be removed as long as the intravenous catheter itself remains intact on the client. Any variation of this standard procedure would be determined by the facility or coroner in advance.

An elderly client has been admitted to the hospital with a diagnosis of cerebral vascular accident (CVA) with right-sided paralysis. When the nurse instructs staff to reposition client every two hours, the family asks about the purpose of this action. What is the best explanation by the nurse? 1. Improves circulation to the affected side of the body. 2. Decreases potential skin breakdown from immobility. 3. Prevents blood stasis in the client's lower extremities. 4. Alleviates sensory deprivation by varying environment.

2. CORRECT. An immobile client is subjected to shearing forces and tissue breakdown because of prolonged contact between the skin and linens. Pressure sores can develop quickly when a client remains in one position over long periods of time, particularly on protruding areas of the body such as hips, elbows, sacrum or heels. Repositioning the client every two hours decreases the potential for skin breakdown and allows for inspection of all vulnerable body areas. 1. INCORRECT. While moving a paralyzed client might stimulate the overall circulation, and even allow for passive range of motion, repositioning a client does not specifically increase blood flow to one side of the body. 3. INCORRECT. Though moving a client can stimulate the circulation, repositioning every two hours is not sufficient to prevent blood stasis in lower extremities, particularly when this client cannot move the right side independently. 4. INCORRECT. Sensory deprivation is not a major concern for the client initially and repositioning is not meant to address sensory needs. The purpose of repositioning is prevention of skin breakdown.

The nurse delegated feeding of a client to the unlicensed assistive personnel (UAP). Two hours after other trays were picked up from the rooms, the nurse notes that the client's untouched tray is still at the bedside. What should the nurse do first? 1. Feed the client after warming the food. 2. Speak to the UAP to determine what happened with the feeding. 3. Pick up the tray and tell the UAP that they didn't do a good job. 4. Provide a between meal supplement to the client.

2. Correct: Communication is important in delegation, as is follow-up. There may be a good reason that the tray was not served. The key word in the stem is first. The other options may be correct but are not the best first action. 1. Incorrect: The client does need to have food; however, there is another action that should be performed first. The reason for the UAP not feeding the client needs to be determined. 3. Incorrect: The nurse retains the responsibility for the delegated task. The nurse should not assume that the UAP just did not do their job, but needs to ascertain the reason for not feeding the client. 4. Incorrect: The concern here is the client being fed their meal. Speak to the UAP first and then decide if a between meal supplement is needed.

Which discharge referral would be a priority for the nurse to make in order to promote continuity of care for a client following a colectomy and colostomy formation due to colon cancer? 1. Home health 2. Meals on Wheels 3. Hospice care 4. Registered dietitian

1. Correct: Clients often go home quickly and do not completely understand discharge instructions. The first priority would be for colostomy care, which can be provided by home health. 2. Incorrect: Meals on Wheels will be important later during rehabilitation but is not the priority. 3. Incorrect: Hospice care is premature. The question does not reveal if surgery was successful or not to remove the colon cancer. 4. Incorrect: A dietary consult may be necessary later, but is not the priority at present.

Which information should the nurse plan to teach family members of a client diagnosed with hepatitis B? 1. Do not share personal items with the client, such as razors or toothbrushes. 2. Wash dishes separately from the rest of the family's. 3. Wear a surgical mask when in close proximity to the client. 4. Use a separate bathroom from the client.

1. Correct: Hepatitis B is a bloodborne pathogen that can spread via sharing personal items, such as razors or toothbrushes where infected blood can get into a person's cut, mucous membranes, etc. 2. Incorrect: Unlike some forms of hepatitis, Hepatitis B is not spread through sharing eating utensils, contaminated food or water. Hepatitis B is spread by infected blood or body fluids. 3. Incorrect: Hepatitis B is not airborne, therefore, there is no need to wear a mask. 4. Incorrect: Hepatitis B is not spread by sharing a bathroom. It is blood borne, not spread by the fecal route.

The charge nurse is planning the staff assignments for the clients on a neurological unit. Which client should be assigned to a nurse who was pulled from a medical unit to the neurological unit? 1. Client admitted 24 hours ago with a diagnosis of a stroke, who is now reporting a headache that intensifies when moving in the bed. 2. Client admitted 48 hours ago with an ischemic stroke and a history of seizures, who has been prescribed clonazepam. 3. Client with an oral temperature of 103.2 F (39.5 C) 36 hours post intracranial surgery. 4. Client diagnosed with a hemorrhagic stroke 1 week ago, who currently has a blood pressure of 170/96.

2. Correct: The medical nurse can be assigned to this client. Clients are frequently admitted to a medical unit with a diagnosis of seizures and prescribed an anti seizure medication. The stem does not indicate any loss of neurological function resulting from the seizure activity. 1. Incorrect: This client is exhibiting early signs of increased intracranial pressure. An experienced neurological nurse should be assigned to this client to assess and manage for signs and symptoms of increasing intracranial pressure. 3. Incorrect: An experienced neurological nurse should be assigned to this client due to the possibility that damage to the hypothalamus which controls body temperature has occurred. An increased temperature will have a direct effect on the brain's metabolism and function. There is a possibility that a hypothermia blanket may be prescribed. 4. Incorrect: The treatment of hypertension is critical in the management of a post hemorrhagic stroke. An experienced nurse would be assigned to this unstable client due to the possibility of a reoccurring hemorrhagic stroke resulting from the client's hypertension.

After receiving report from the previous shift nurse, Which client should the nurse assess first? 1. Client diagnosed with an ischemic stroke who is exhibiting increased restlessness. 2. Client diagnosed with dementia who needs assistance with ambulating. 3. Client with a halo device requesting to be transferred to the bedside chair. 4. Client diagnosed with a traumatic brain injury who cannot recall portions of the accident.

1. Correct: The client diagnosed with an ischemic stroke needs to be assessed first to be evaluated for signs of increased intracranial pressure (ICP). A neurological assessment should be initiated. Increased restlessness is an early sign of increased ICP. 2. Incorrect: Client safety should be evaluated. The client does require assistance with ambulating. But the client with potential increased ICP requires an immediate neurological assessment. 3. Incorrect: A client with a halo traction may require assistance to transfer to the chair. The nurse identifies that a neurological assessment on another client has priority. 4. Incorrect: The client with the traumatic head injury cannot recall portions of the accident, but is not presenting with any life-threatening symptoms.

A client on a surgical unit frequently quarrels with the staff. Which nursing intervention should the charge nurse implement? 1. Involve the client in their plan of care. 2. Delegate 2 nurses to work with the client. 3. Accept the client's behavior as confrontational. 4. Encourage the client to be more cooperative.

1. Correct: The client has the right to be involved in the decision making of their care. The healthcare team should recognize the client as the center of the team. This action will promote the client's self-esteem, and may reduce the quarrelsome behavior. 2. Incorrect: Delegating 2 nurses to work with the client does not address the client's behavior. This action is a defensive intervention, and does not address the quarrelsome behavior. 3. Incorrect: The nurse is assuming that the client's quarrelsome behavior is normal for this client. This assumption is not appropriate, and the feelings and concerns of the client should be addressed. 4. Incorrect: By encouraging the client to be more cooperative, the nurse is denying the client's feelings and concerns.

After applying oxygen using bi-nasal prongs to a client who is having chest pain, the nurse should implement which intervention? 1. Have the client take slow deep breaths in through the mouth and out through the nose. 2. Post signs on the client's door and in the client's room indicating that oxygen is in use . 3. Apply Vaseline petroleum to both nares and 2 x 2 gauze around the oxygen tubing at the client's ears. 4. Encourage the client to hyperextend the neck, take a few deep breaths and cough.

2. Correct: This is an oxygen therapy safety precaution that the nurse should implement after applying oxygen. It is also the only correct and safe option in the question. 1. Incorrect: The bi-nasal prongs would mean that the oxygen is going in through the nose. Breathing deeply through the mouth and out through the nose would not increase oxygenation for a client having chest pain and would disrupt the flow of oxygen through the nose. 3. Incorrect: The nurse should avoid using petroleum products where oxygen is in use because they are flammable. 4. Incorrect: These client actions have nothing to do with oxygen administration and would cause more distress to the client with chest pain.

Which task would be appropriate for the nurse to assign to an LPN/LVN? 1. Changing a colostomy bag. 2. Hanging a new bag of total parenteral nutrition (TPN). 3. Teaching insulin self administration to a diabetic client. 4. Administering IV pain medication to a two day post op client.

1. Correct: The only procedure listed that is within the LPN/LVN's practice range is changing the colostomy bag. This is a task that can be delegated to the LPN/LVN. 2. Incorrect: Hanging a new bag of TPN is parenteral therapy requiring a central line. This is outside the scope of practice for the LPN/LVN. Therefore, the RN must perform this task and cannot delegate this to the LPN/VN. 3. Incorrect: Teaching is outside the scope of practice for the LPN/LVN. Teaching can be reinforced by the LPN/LVN, but they cannot perform the initial teaching. Teaching insulin self administration cannot be delegated to the LPN. 4. Incorrect: The administration of parenteral pain medications is not in the scope of practice for the LPN/LVN. This should not be delegated to the LPN/LVN.

A client is admitted to the unit from the ED department. What acid base imbalance do the lab values indicate to the nurse? Exhibit ABGs: pH 7.48 PaCO2 38 HCO3 30 1. Metabolic alkalosis 2. Compensated metabolic alkalosis 3. Respiratory alkalosis 4. Compensated respiratory alkalosis

1. Correct: The pH is 7.48 which is alkalosis. PaCO2 is 38, which is normal.​HCO3 is 30, which is high (alkaline). The problem is a metabolic problem, making this Metabolic Alkalosis. 2. Incorrect: The pH is 7.48 which is alkalosis. PaCO2 is 38, which is normal. HCO3 is 30, which is high (alkaline). The problem is a metabolic problem, making this Metabolic Alkalosis. 3. Incorrect: The pH is 7.48 which is alkalosis. PaCO2 is 38, which is normal. HCO3 is 30, which is high (alkaline). The problem is a metabolic problem, making this Metabolic Alkalosis. 4. Incorrect: The pH is 7.48 which is alkalosis. PaCO2 is 38, which is normal. HCO3 is 30, which is high (alkaline). The problem is a metabolic problem, making this Metabolic Alkalosis.

The nurse is preparing to administer cefazolin 0.5 grams in 100 mL D5W IVPB over 30 minutes. How many mL/hour will the nurse need to set the IV infusion pump at? Round to the nearest whole number.

Answer: 200 Rationale: Prescription: Cefazolin 0.5 grams in 100 mL D5W IVPB over 30 minutes. Step 1: Determine what the drop factor is on the IV tubing. IV infusion pumps have a drop factor of 60 gtts/min. Step 2: Remember the formula to regulate an IV by infusion pump- Total mL prescribed/Total hr prescribed Step 3: 100 ml/30 x 60 gtts/1 mL = 100/30 x 60/1 = 200 mL/hr

Twelve hours post coronary artery bypass surgery (CABG), the nurse notes the client's level of consciousness has decreased from alert to somnolent. BP 88/50, HR 130 and thready, resp 32, urinary output (UOP) has dropped from 100 mL one hour earlier to 20 mL this hour. What would be the nurse's first action? 1. Administer 100% oxygen per mask. 2. Lower the head of the bed. 3. Give furosemide STAT. 4. Re-check the BP in the other arm.

1. Correct: This client has developed signs of cardiogenic shock, one of the complications post CABG. Cardiac output is decreased, so the client needs more oxygen for the circulating blood volume. 2. Incorrect: Lowering the HOB will not help in cardiogenic shock but will actually make it harder for the heart to pump. 3. Incorrect: Poor kidney perfusion is the reason for the decreased UOP. The kidneys are trying to conserve what little volume the body has to maintain vital organ perfusion as long as possible. 4. Incorrect: Rechecking the BP will not help the problem. With the other symptoms, this BP is most likely accurate. This would only delay treatment and would not fix the problem.

A client delivered a term infant four hours ago. The infant was stillborn. Which room would be most appropriate for the nurse to assign to this client? 1. A private room on the gynecological unit. 2. A private room on the postpartum unit. 3. Discharge her home as soon as her condition is stable. 4. Room her with another client with a pregnancy loss.

1. Correct: This client needs a private room so she can feel free to grieve and have family members stay with her for support. She should be transferred to a gynecological unit so the sights and sounds of the maternity unit do not contribute to her pain. 2. Incorrect: Difficult for mother with stillborn to be on postpartum unit with mothers and their babies. The mother should not be surrounded by these reminders. 3. Incorrect: She does not need to be rushed out of the hospital. She needs to have time with her stillborn and also still needs to be assessed for postpartum complications. Remember that she is going through all of these postpartum stages of normal delivery and requires observations. 4. Incorrect: I know we say like illnesses go together but not here. This client needs privacy and time with her family.

The women's health charge nurse is making assignments for the next shift. The unit is short one staff member and will receive a nurse from the medical surgical unit. Which group of clients should she assign to the medical surgical nurse? 1. Total abdominal hysterectomy, bladder suspension with A&P repair, client with breast reduction. 2. C-section planning discharge, postpartum infection, mastectomy. 3. Vaginal delivery of fetal demise, C-section with pneumonia, 32 week gestation with lymphoma. 4. 28 week gestation of bed rest, postpartum with HELLP syndrome, breast reconstruction.

1. Correct: This group of clients is primarily med surgical. 2. Incorrect: This group of clients needs specific teaching. 3. Incorrect: This group of clients needs specialized care. 4. Incorrect: No, the monitoring is too specific for the med-surg nurse.

A client with asthma uses a corticoid inhaler. What teaching should the nurse provide to decrease the risk of an oral fungal infection? 1. Lessen the exposure of the oral mucosa to the ICS by exhaling rapidly. 2. Rinse the mouth completely and brush teeth following the use of the ICS. 3. Use alcohol based mouth rinses with ICS. 4. Drink water prior to using the ICS.

2. Correct: Thrush, is an oral fungal infection, which is one of the most common side effects of ICS. Up to 1/3 of all clients on ICS develop this infection. Rinsing and brushing helps to remove the medication residual from the oral mucosa and upper pharyngeal area. 1. Incorrect: This is not appropriate because exhaling rapidly would result in a loss of the medication and reduce the effectiveness. 3. Incorrect: This is not accurate because alcohol based mouth rinses have not been shown to reduce the risk of thrush. Alcohol based mouthwash can be drying to the oral mucosa. 4. Incorrect: Drinking water, prior to using the ICS is not an effective means of preventing thrush.

The nurse is caring for a burn victim with a skin graft to the hand. The area is pale and mottled but has good capillary refill. What is the nurse's best action at this time? 1. Warm the room. 2. Submerge the hand in warm water. 3. Order a K pad and apply to hand. 4. Have the client exercise the fingers to increase blood flow.

1. Correct: When caring for clients with skin grafts, we want good circulation, so warm that room up. 2. Incorrect: This will not improve circulation and can lead to infection. 3. Incorrect: This will not improve circulation. Someone who has a skin graft doesn't have good sensation so there is risk of another burn to the graft with this. 4. Incorrect: Working those stiff, cold fingers will further imbalance the oxygen supply. This will not help, particularly if the environment remains cool.

A male client diagnosed with primary hyperaldosteronism is receiving spironolactone. Which potential side effect should the nurse educate the client regarding? Select all that apply 1. Erectile dysfunction 2. Gastrointestinal upset 3. Gynecomastia 4. Hypernatremia 5. Hypokalemia

1., 2., & 3. Correct: Spironolactone blocks androgen and progesterone receptors and may inhibit the action of these hormones. Side effects can include gynecomastia, decreased sexual desire, impotence, menstrual irregularities, and gastrointestinal distress. 4. Incorrect: Hyponatremia, rather than hypernatremia, may be seen. 5. Incorrect: Hyperkalemia, rather than hypokalemia, may be seen.

The home health nurse is assessing the home environment for threats to the safety of the toddler who lives in the home. Which observations should be included in this assessment? Select all that apply 1. Do stairs have guard gates? 2. Are safety covers on electrical outlet plugs? 3. Is the swimming pool inaccessible to the toddler? 4. Are cleaning supplies located out of the toddler's reach? 5. Are stairs brightly lit?

1., 2., 3. & 4. Correct: Toddlers may fall if left unsupervised around stairs. Make sure that gates are in place and that they are used. Toddlers are at risk for exploring the outlets by putting metal objects into the outlets or putting their fingers in them. They should be covered unless in use. Toddlers can drown in small amounts of water and they may try to explore swimming pools if they are accessible. Pools should have fences or locking stairs and the child should never be left unsupervised around the pool. Toddlers are curious and may get into cabinets containing harmful substances. 5. Incorrect: This assessment would be important for the visually impaired or elderly, but not specifically for toddlers. The guard gates should be in place so that the toddler does not have access to the stairs.

A nurse working in a clinic is planning to assess a client for any sensory deficits. What assessments should the nurse include? Select all that apply 1. Ask the client about any recent changes in vision. 2. Observe the client's conversation with others. 3. Assess two-point discrimination. 4. Perform the Rinne test. 5. Test near vision with the Snellen chart.

1., 2., 3., & 4. Correct: Physical assessment determines whether the senses are impaired. During the physical examination, the nurse assesses vision and hearing and the olfactory, gustatory, tactile, and kinesthetic senses. The exam should reveal the client's specific visual and hearing abilities, perception of heat, cold, light touch, pain in the limbs, and awareness of the position of body parts. Start with a history and ask about recent changes in vision. Observing client conversation with others can indicate hearing or communication problems. Two-point discrimination will assess tactile sense. The Rinne test assesses hearing. 5. Incorrect. Near vision is assessed by using the Rosenbaum eye chart. It consists of paragraphs of text or characters in different sizes on a 3.5 x 6.5 inch card. Be sure the client has a literacy appropriate for the text used.

What discharge education should a nurse provide to a client post hip replacement with a metal joint? Select all that apply 1. Weight bearing limits. 2. Use of a high seated chair. 3. Sexual intercourse in dependent position for up to six months. 4. Avoid taking showers. 5. Use of long handled tongs to assist with dressing.

1., 2., 3., & 5. Correct: Weight bearing limits on the involved extremity varies according to the healthcare providers preference but are commonly prescribed. The client needs to avoid flexion. This includes sitting in low chairs and getting into a bathtub; elevated toilet seats and raised seats are necessary. Sexual intercourse should be carried out with the client in a dependent position (flat on the back) for 3-6 months to avoid excessive adduction and flexion of the new hip. To avoid flexion when dressing, adaptive devices and utensils may be used to help with bathing, dressing and personal hygiene. 4. Incorrect: Showers are preferable as getting into a tub would cause flexion of the new hip. This could cause the hip to dislocate.

A nurse is teaching a client the advantages of having a PICC line inserted rather than a peripheral IV. What information should the nurse include? Select all that apply 1. TPN may be infused using a PICC line. 2. Use of a PICC can allow for early client discharge. 3. PICC lines do not have to be replaced as often as a peripheral IV line. 4. PICC lines have the same risk of infection as a peripheral IV line. 5. PICC lines do not need to be flushed as frequently. 6. PICC placement decreases the need for skin puncture when blood sampling is needed.

1., 2., 3., & 6. Correct: Peripheral IV lines must be changed every 72-96 hours. PICC lines may remain in place for extended periods of time. A PICC can be cared for at home by home care nurses, family members, or in outpatient clinics. TPN cannot be administered via a peripheral line since it is hypertonic. PICC lines offer a lower chance for infection than a peripheral line. As long as the PICC is functioning and there is no evidence of infection, the PICC line can remain in place until it is no longer needed. 4. Incorrect: PICC lines are long lasting, so the risk of infection from changing sites is eliminated. Additionally, sterile technique is used for insertion, with sterile dressing changes. Precautions should still be taken to prevent complications. 5. Incorrect: Both peripheral and central lines need to be flushed to maintain patency.

A client diagnosed with new onset atrial fibrillation has been prescribed dabigatran. What should the nurse teach this client? Select all that apply 1. Place medication in a weekly pill organizer so that medication is not forgotten. 2. Do not take with clopidogrel. 3. Dabigatran decreases the risk of stroke associated with atrial fibrillation. 4. Take this medication with food. 5. aPTT and INR levels will be drawn monthly.

2., 3., & 4. Correct: Do not take dabigatran with any other anticoagulants, including clopidogrel due to increased bleeding risk. Dabigatran decreases the risk of stroke and systemic embolism in clients with atrial fibrillation that is not associated with a cardiac valve problem. Take this medication with food to decrease gastric side effects such as dyspepsia and gastritis. Proton pump inhibitors and histamine 2 receptor blockers may also decrease gastric side effects. 1. Incorrect: After container is opened, medication should be used within 30 days. It is sensitive to moisture and should not be stored in weekly pill organizers. To maintain efficacy, keep medication in manufacturer- supplied bottle. 5. Incorrect: This medication does not require monitoring of INR levels. However, the client should be informed about the risk of bleeding and to monitor for signs of bleeding.

The homecare nurse is providing family teaching on safety issues for a client diagnosed with Parkinson's disease. What adaptations should the nurse instruct the family to initiate? Select all that apply 1. Install grab bars on tub walls. 2. Place nightlights in hallways. 3. Add bran and fiber to daily diet. 4. Remove scatter rugs or loose cords. 5. Keep bedroom dark, cool and quiet. 6. Put tennis balls on legs of walker.

1., 2., 3., 4., & 5. Correct: Parkinson's disease causes deterioration of the basal ganglia, ultimately impacting motor control and function. As muscles become stiff and rigid, mobility slows, resulting in poor coordination and loss of balance. Safety is a chief concern in all ADLs, requiring modifications in activity, nutrition, and the client's environment. Because Parkinson's disease affects mobility, modification such as grab bars and night lights are essential. Clients develop constipation because of decreased peristalsis, so adding bran and fiber can address impending bowel issues. Scatter, or throw, rugs along with loose extension cords on the floor create a fall risk because the client is unable to pick up feet during ambulation. The shuffling gait that develops increases the risk for falls. These clients also have problems with insomnia along with poor REM sleep, leading to daytime drowsiness. Making the bedroom conducive to sleep may help alleviate symptoms for a period of time. A dark, cool room with no distractions is the most appropriate sleep environment. 6. Incorrect: The proper method of utilizing a walker is to step into the walker, pause and then move it forward before stepping again. Even though clients with Parkinson's disease have a shuffling gait and stooped posture, sliding a walker with tennis balls on the walker legs presents a serious safety issue. The client would not have the ability to control the speed or hold the walker steady while stepping into it.

A school nurse is planning to teach kindergarten students about oral health. Which points should the nurse include? Select all that apply 1. Do not drink soft drinks between meals. 2. Eat raw vegetables to help keep teeth clean. 3. Brush teeth twice a day with toothpaste that does not have fluoride. 4. Use a pea size amount of toothpaste. 5. Floss teeth daily.

1., 2., 4., & 5. Correct: Avoid sweet foods and drinks between meals. Take them in moderation at meals. Eat coarse, fibrous foods, cleansing foods, such as fresh fruits and raw vegetables. If unable to brush after a meal, vigorously rinse mouth with water. Teeth should be flossed daily. 3. Incorrect: Brush the teeth thoroughly with toothpaste that has fluoride.

What signs and symptoms will the nurse look for when caring for an infant with severe dehydration? Select all that apply 1. Dark, yellow urine 2. Lethargic 3. Bulging fontanels 4. Tachypnea 5. Decreased urine output

1., 2., 4., & 5. Correct: These would be signs and symptoms of dehydration in an infant. Amber or dark urine is an indication of dehydration. Urine should be a clear, pale yellow. Fussiness and irritability are seen in infants when they do not feel well. As dehydration worsens, lethargy and unresponsiveness can develop. Tachypnea or rapid respiration along with tachycardia and low blood pressure are present with severe dehydration. With severe dehydration, there will be decreased urine output. The body is trying to conserve volume. 3. Incorrect: The fontanels will be sunken rather than bulging. Bulging fontanels indicate brain swelling or fluid build up in the brain. Sunken fontanels are related to dehydration.

Which tasks are most appropriate for the hospice nurse to delegate to an unlicensed assistive personnel (UAP)? Select all that apply 1. Bathe the client. 2. Provide spiritual support 3. Listen to the client reminisce. 4. Administer routine medications. 5. Weigh the client. 6. Take vital signs

1., 3., 5., & 6. Correct: The UAP can bathe, listen to the client reminisce, weigh, and take the vital signs. These are within the scope of practice of the UAP. These assignments are routine and revolve around activities of daily living. 2. Incorrect: The task of providing spiritual support could best be delegated to the pastor or chaplain. 4. Incorrect: The nurse can not delegate routine medication administration to the UAP. This is not within the UAPs scope of practice. This is an LPN or RN responsibility.

The nurse is preparing a client for transport to the radiology department for a left lung tissue biopsy. Which actions should the nurse make certain have been completed? Select all that apply 1. The consent form is signed. 2. The operative site is prepped with a razor. 3. The most recent lab work is on the chart. 4. Any preoperative medication is given as prescribed. 5. Person performing the procedure has marked the site.

1., 3., 4. & 5. Correct: The nurse should ensure that the consent form is signed, the lab work is in order, and any prescribed preoperative medication is given. The operative site is marked by the person who is ultimately accountable for the procedure and will be present when the procedure is performed. 2. Incorrect: The site should be prepped with clippers as opposed to a razor, which can cause injury to the client. The goal of preoperative skin preparation is to decrease bacteria without injuring the skin.

A nurse is teaching a client who has frequent urinary tract infections how to prevent future infections. What statement by the client would indicate to the nurse that treatment has been successful? Select all that apply 1. "I will go to the bathroom as soon as the urge to void hits me." 2. "It is important for me to drink five to six 8 ounce glasses of water every day." 3. "I should eat foods such as plums and prunes to increase the acidity of my urine." 4. "Nylon underwear should be worn when I am free from infection." 5. "When I clean after voiding, I will discard toilet paper after each swipe."

1., 3., & 5. Correct: Holding urine can lead to stasis of urine and increasing the risk for infection. Foods such as eggs, cheese, meat and poultry, whole grains, cranberries, plums and prunes, and tomatoes tend to increase the acidity of urine. Acidic urine is less likely to allow for bacterial growth. Discarding toilet paper after each swipe will decrease exposure and accidental introduction of bacteria into the urinary meatus. 2. Incorrect: Emphasize the importance of drinking eight to ten 8 ounce glasses per day. Water helps flush bacteria from the urinary tract. 4. Incorrect: Cotton underwear is recommended. The natural fibers work to wick moisture away from the skin which discourages yeast growth.

What foods should the nurse teach a client who has been diagnosed with iron deficiency anemia to increase in the diet? Select all that apply 1. Chickpeas 2. Milk 3. Oysters 4. Raisins 5. Spinach 6. Tuna

1., 3., 4., 5., & 6. Correct: All of these are high in iron. 2. Incorrect: Milk is not high in iron and slows the absorption of iron.

Which signs/symptoms should the nurse assess for when caring for a client diagnosed with bulimia nervosa? Select all that apply 1. Increased thirst 2. Muscle cramps 3. Blurred vision 4. Tingling of lips 5. Constipation

2., 4., 5. Correct: The typical abnormalities associated with bulimia are hypokalemia and metabolic alkalosis because of the binging and purging process. This leads to muscle cramps, weakness, fatigue, constipation, and arrhythmias are all symptoms of this electrolyte and acid-base imbalance. Hypokalemia leads to metabolic alkalosis. 1. Incorrect: Increased thirst is a sign of hyperglycemia and would not be the concern with someone that is purging. This client would be more likely to be hypoglycemic instead. 3. Incorrect: Blurred vision is a sign of hyperglycemia because of the effect of too much glucose in the small vessels of the eye. Microvascular damage is one of the biggest concerns with hyperglycemia; the bulimic client would be hypoglycemic.

The nurse is caring for a client who has been receiving treatment for systolic heart failure. What assessment findings would indicate to the nurse that further treatment is necessary? Select all that apply 1. 3+ pedal edema 2. CVP of 6 mm Hg 3. One day weight loss of 2 pounds (0.9 kg) 4. Pursed-lip breathing 5. Pale nail beds 6. Urine output at 50 mL/hr

1., 4. & 5. Correct: These three findings would indicate that further treatment is needed. 3+ pedal edema, and pursed-lip breathing is seen when client is still short of breath. Pale conjunctiva, nail beds, buccal mucosa are signs of impaired gas exchange. 2. Incorrect: Normal CVP is 2-6 mm Hg. This CVP is within normal range so treatment is effective. 3. Incorrect: Weight loss indicates that fluid is being removed. 6. Incorrect: A urine output of 50 mL/hour indicates that renal perfusion is adequate.

A three year old weighing 13.6 kg is scheduled to receive a dose of digoxin elixir. The prescribed dose is 25 micrograms/kg How many mL will the nurse administer to the child? Round to 1 decimal place. Use numbers and decimals only. Exhibit Medicine Cart Drug Availability: Lanoxin (digoxin) Elixir Pediatric Each mL contain 50 mcg (0.05 mg)

Answer: 6.8 Rationale: Prescribed: Digoxin elixir 25 micrograms/kg Step 1: Determine how many micrograms per kg should be given. 25 mcg/kg x 13.6 kg = 340 mcg Step 2: Think: You will want to give less than 7 mL, since one mL is 50 mcg. Step 3: D/H x Q = 340 mcg/50 mcg x 1 mL = 34/5 = 6 4/5 = 6.8 mL

Which observations should the home health nurse discuss with the parents of a two year old regarding potential safety threats in the home? Select all that apply 1. Security gates at the stairs. 2. Cleaning supplies under sink cabinet. 3. No blinds on windows. 4. Use of space heaters. 5. Water heater temperature 140°F (60°C) 6. Use of tablecloths

2, 4, 5 & 6 Correct: Cleaning supplies should be placed high away from child's reach. Cabinets should have childproof locks. Space heaters need to be checked every year prior to use. Additionally, small children can be burned by space heaters if they get too close. A guard should be applied. Water heaters should be set at no higher than 120°F (48°C). Burns may occur with a 6 second exposure to 140°F water temperature. Children can pull on table cloths and spill hot food or break dishes which could lead to injury. 1. Incorrect: Placing security gates at the stairs will prevent falls. 3. Incorrect: This is not a concern for the child. If there are blinds, the string should be out of the child's reach.

What should the nurse teach the client following a right knee arthroscopy? Select all that apply 1. Apply ice to right knee continuously for the first 24 hours. 2. Elevate the right knee when sitting. 3. Notify the primary healthcare provider of tingling in the right leg. 4. Gradually start an exercise program to prevent scarring. 5. Place a plastic bag over wound when showering.

2., 3., 4. & 5. Correct: Elevating the joint for several days will reduce swelling and pain. Tingling to the extremity could mean nerves have been damaged. Exercise is gradually started to strengthen muscles surrounding the joint and prevent scarring of surrounding soft tissues. The client needs to keep the site as clean and dry as possible. 1. Incorrect: Continuous ice can cause tissue damage.

The nurse is caring for a client diagnosed with major depression post electroconvulsive therapy (ECT). What nursing interventions should be included in this immediate post-treatment period? Select all that apply 1. Monitor vital signs every hour for eight hours. 2. Position the client on their side. 3. Stay with the client until fully awake. 4. Provide flexibility in scheduling routine activities. 5. Encourage the client to ambulate in room and hall.

2. & 3. Correct: Positioning on the side will prevent aspiration. Stay with the client until they are fully awake, oriented, and able to perform self-care activities without assistance. Safety is priority. 1. Incorrect: Pulse, respirations, and blood pressure should be monitored every 15 minutes for the first hour. Vital signs every hour are too long immediately post-treatment. 4. Incorrect: The client needs a highly structured schedule of routine activities in order to minimize confusion. Also, immediately post-treatment is too soon to address routine activities. 5. Incorrect: The client should remain in bed during the immediate post-treatment period. The client needs to be fully awake prior to ambulation.

The nurse receives report about a client who is termed "a drug seeker." The nurse giving report states that the client does not need the pain medication and is just asking for medication because the client is "hooked on it." After receiving report, what actions should the nurse take? Select all that apply 1. Consult with the primary healthcare provider. 2. Assess the client. 3. Increase gradually the time between pain medication. 4. Encourage the client to wait longer before requesting the medication. 5. Utilize a pain scale to determine level of pain.

2. & 5. Correct: The nurse should carefully assess the client. The nurse must serve as an advocate for the client. A pain scale is used to determine level of pain. 1. Incorrect: The nurse must assess the client before consulting with the primary healthcare provider about the medication. 3. Incorrect: This action assumes that the client does not have pain, which does not take into consideration what is wrong with the client. 4. Incorrect: This action assumes that the client is a "drug seeker". The nurse must carefully assess the client.

The charge nurse is observing a new LPN preparing to irrigate a client's indwelling urinary catheter. The nurse must intervene when the LPN initiates what action? 1. Gathers all sterile equipment for procedure. 2. Opens bottle of sterile distilled water to flush. 3. Allows return flow to be achieved by gravity. 4. Uses gentle pressure when flushing catheter.

2. CORRECT. The charge nurse is observing the LPN's ability to complete this invasive procedure, monitoring for any action that could harm the client. The charge nurse should intervene to prevent the LPN from using sterile distilled water. Only sterile normal saline (NS) can be used to flush out the catheter to prevent an alteration in the pH balance of the bladder. 1. INCORRECT. This process is appropriate since all equipment should be gathered prior to any procedure. A nurse must be organized and prepared before any client procedure. The question is asking for something the LPN has done incorrectly. 3. INCORRECT. After a urinary catheter has been gently flushed with NS, the LPN would allow the fluid and any urine to return into the sterile basin by gravity flow. This correct action would not require the charge nurse to intervene during the process. 4. INCORRECT. Flushing a catheter is a delicate procedure, requiring slight, gentle pressure to instill the normal saline. Forcing fluid into the bladder can cause trauma or damaged to the tissue. The LPN completed this action correctly, requiring no intervention by the charge nurse.

While preparing to administer intravenous of chemotherapy the nurse accidently pulls the tubing apart, spilling the solution onto the floor. After clamping the tubing, what is the nurse's immediate action? 1. Use disposable towels to clean up the liquid. 2. Obtain spill kit specific to this type of solution. 3. Complete an incident report for supervisor. 4. Call housekeeping to help clean up the floor.

2. CORRECT: Chemotherapy spill kits are pre-packaged supplies specific to the type of cytotoxic drugs used and are kept in close proximity to the location the chemo is administered. These kits vary slightly but all follow the basic guidelines. Individuals cleaning up the spill must be completely covered head to toe to prevent any contact with the drug. This includes inhalation. This option contains the word solution, which also appears in the question. 1. INCORRECT: Disposable towels are not acceptable to clean up a chemotherapy spill. Although these towels are absorbent for kitchens and bathrooms, only special absorbent pads can be used to clean up cytotoxic drugs. 3. INCORRECT: While it is true that the nurse will need to complete an incident report regarding the chemotherapy spill, it is certainly not the nurse's immediate action. Focus on staff and client safety first. 4. INCORRECT: The responsibility for cleaning up cytotoxic drugs is for the nursing staff involved at the time. Special training and knowledge is required to handle this issue.

A new nurse is preparing to give a medication to a nine month old client. After checking a drug reference book, crushing the tablet and mixing it into 3 ounces of applesauce, the new nurse proceeds to the client's room. What priority action should the supervising nurse take? 1. Tell the new nurse to recheck the drug reference book before administering the medication. 2. Suggest the new nurse reconsider the client's developmental needs. 3. Check the prescription order and the client dose. 4. Observe the new nurse administer the medication.

2. Correct. Mixing medication with applesauce is appropriate in some circumstances, but the volume of 3 ounces is excessive for a nine month old. The nurse will want to make sure the client gets all of the medication. Additionally, applesauce may or may not have been introduced into the diet, and it is inappropriate to introduce a new food during an illness. 1. Incorrect. There is nothing in the stem about a problem with the medication dose or route. The drug reference book does not provide guidelines for meeting developmental needs when administering the medication. This is something that the nurse must look up if uncertain about developmental tasks. 3. Incorrect. There is nothing in the stem about a problem with the medication dose or route. Once the medication has been mixed in applesauce, the supervising nurse would not be able to compare the dose to the prescribed amount. Therefore, this would not be an appropriate action. It would not address the developmental task that is the underlying issue here. 4. Incorrect. This is an appropriate action. However, it is not the priority. The new nurse should be competent in medication administration but is needing guidance with the developmental considerations related to medication to a nine month old.

The nurse assesses a client post thyroidectomy for complications by performing which assessment? 1. Accucheck 2. Chvostek's 3. Ballottement 4. Ice water colonic

2. Correct: A positive Chvostek's and Trousseau's is indicative of tetany and low calcium. This can occur when a couple of parathyroids are accidently removed when the thyroid is removed. 1. Incorrect: Accucheck assesses for blood glucose levels, which is not the problem post thyroidectomy. 3. Incorrect: This assessment technique is used in examining the abdomen when ascites is present. It is done by palpating the abdomen to detect excessive amounts of fluid (ascites). 4. Incorrect: If you have never heard of it, no one else has either. The phrase implies using ice water to cleanse the colon and this would never be a good thing, especially for someone post thyroidectomy that would be intolerant to extremes in temperature.

After obtaining vital signs, which prescribed medication should the nurse hold when caring for a client on the cardiac unit? Exhibit Vital Signs: T - 98 F (36.7C) P - 74 R- 20 BP - 88/50 1. Rosuvastatin 2. Enalapril 3. Digoxin 4. Clopidogrel

2. Correct: Enalapril is an angiotensin converting enzyme (ACE) inhibitor. An ACE inhibitor will lower the client's blood pressure. The blood pressure in the stem's exhibit is low. Lowering the client's blood pressure more could have a negative effect on the client's condition. 1. Incorrect: Rosuvastatin is a lipid lowering medication. The client's blood pressure has no bearing on whether or not to administer the medication. 3. Incorrect: Digoxin is an antiarrhythmic/inotropic agent. It will slow the heart rate and increase the force of myocardial contraction. This action could actually increase the blood pressure. 4. Incorrect: Clopidogrel is an antiplatelet agent. The client's blood pressure would not have a bearing on whether or not to administer the medication.

A newly hired unlicensed assistive personnel (UAP) has consistently completed all assignments in a safe and timely manner. What is the most appropriate action by the charge nurse? 1. Assign more daily tasks to the UAP. 2. Provide positive feedback to the UAP. 3. Allow the UAP to work without supervision. 4. Teach the UAP to change surgical dressings.

2. Correct: Positive feedback is an effective communication tool that improves the workplace environment and encourages individual achievement, particularly in challenging situations. A new UAP is efficiently completing all daily assignments accurately and in a timely manner. This individual should be provided appropriate comments of appreciation for this accomplishment. 1. Incorrect: Just because the UAP is able to accomplish all daily assignments efficiently does not mean more work could be handled as effectively. It would not be appropriate to overload this new employee with extra work. 3. Incorrect: The scope of practice for the UAP encompasses basic personal care needs, ambulating, and taking vitals; however, the nurse must still verify that all tasks are accomplished in a safe manner. 4. Incorrect: A UAP may not remove and change surgical dressings, which would involve assessment and further education. The nurse cannot allow the UAP to perform advanced tasks.

Which nursing intervention represents secondary prevention level? 1. Teaching the effects of alcohol to elementary school children. 2. Providing care for abused women in a shelter. 3. Leading a group of adolescents in drug rehabilitation. 4. Ensuring medication compliance in a client with schizophrenia.

2. Correct: Providing care for clients in a shelter for abused women indicates that a problem has been identified and is being monitored to prevent the problem from getting worse. The focus of secondary prevention is early detection, use of referral services, and rapid initiation of treatment to stop the progress of the disease. 1. Incorrect: This is primary prevention which is aimed at reducing the incidence of mental or physical disorders within the population. 3. Incorrect: This is tertiary prevention which is designed to restore self-suffering and to limit complications and disabilities associated with a disease state, such as substance abuse or mental illness. 4. Incorrect: This is tertiary prevention which is designed to restore self-suffering and to limit complications and disabilities associated with a disease state, such as substance abuse or mental illness.

A community health nurse is presenting a seminar to teen parents on the topic of infant safety. What priority topic presented by the nurse represents the leading cause of injury or death among infants? 1. Monitoring the infant for food allergies. 2. Placing the infant in rear-facing, approved car seat. 3. Never propping bottle to feed when infant is alone. 4. Positioning infant prone when sleeping or napping.

2. Correct: The leading cause of death among infants under the age of one year is motor vehicle accidents. When instructing first time or young parents, it is vital to teach the need to have the infant snuggly restrained in an appropriately sized, approved infant car seat in the back seat and rear-facing. 1. Incorrect: While discussing the signs or symptoms of food allergies is an important topic for new parents, this is not the most vital information the nurse could present to the teen parents. 3. Incorrect: An infant should never be left unattended while feeding, and propping a bottle could lead to aspiration or respiratory distress. This is a dangerous practice that needs to be discussed by the nurse; however, there is another topic that is more urgent. 4. Incorrect: The research studies to date indicate the safest sleeping position for newborns and infants is supine, not prone. Positioning is always a nursing concern, and teaching new parents about the potential for sudden infant death syndrome (SIDS) would be crucial. However, another topic presents more important information.

Which action by two unlicensed nursing personnel (UAPs), while moving the client back up in bed, would require intervention by the nurse? 1. Lowers the side rails closest to them. 2. Places hands under client's axilla. 3. Lowers the head of bed. 4. Raises the height of the bed.

2. Correct: This action is not appropriate and requires intervention by the nurse. This could damage the brachial plexus nerves under the axilla. Use a draw sheet to prevent this from occurring. 1. Incorrect: This is a correct action. The UAPs will need to lower the side rails closest to them to safely move the client up in bed. Not lowering the rails could injury the UAPs back. 3. Incorrect: This action is correct. Moving the client upward with the head of the bed raised works against gravity, requires more force and can cause back strain. 4. Incorrect: This action is appropriate and would not require intervention by the nurse. Raising the height of the bed brings the client close to the UAPs center of gravity and decreases the chance of back injury.

The nurse is teaching a client, recovering from a myocardial infarction (MI), about the prescribed diet of low sodium, low saturated fat, and low cholesterol. Which statements, if made by the client, would indicate to the nurse that teaching has been successful? Select all that apply 1. "I should drink fruit juices rather than soft drinks." 2. "A good snack to eat would be unsalted popcorn." 3. "When making homemade tomato sauce, I should not add salt." 4. "I should use 2% milk when cooking." 5. "There is no restriction on egg white consumption."

2., 3., & 5. Correct: Popcorn without salt is a healthy snack choice for clients on a low sodium, low saturated fat, and low cholesterol diet. Homemade tomato sauce can be made without adding salt. The American Heart Association no longer makes recommendations on how many egg yolks to eat or not to eat. A good, general guideline is to eat no more than 1 egg yolk a day, up to 5 total a week. There is no restriction on egg whites (including those used in baking and cooking). 1. Incorrect: Consume whole vegetables and fruits rather than fruit juices. Fruit juices have added sugars and lack fiber. 4. Incorrect: Use skim or 1% milk rather than 2% or whole milk.

What should a nurse teach a group of teenage boys who admit to using smokeless tobacco? Select all that apply 1. Smokeless tobacco increases risk for lung cancer. 2. Inspect mouth frequently for lesions. 3. White patches in mouth should be reported to healthcare provider. 4. Risk for stomach cancer can be decreased by not swallowing smokeless tobacco juice. 5. Report decreased saliva to primary healthcare provider. 6. Smoking cessation.

2., 3., & 6. Correct: The mouth should be inspected frequently for painless lesions that do not heal. This may be a sign of oral cancer and should be reported to the primary health care provider. White patches (leukoplakia) is a sign of potential oral cancer as well. Nicotine is addictive and is found in smokeless tobacco. Clients using smokeless tobacco can benefit from smoking cessation information/classes. 1. Incorrect: Use of smokeless tobacco increases the risk developing of esophageal cancer, cancers of the mouth, throat, cheek, gums, lips, tongue, pancreatic cancer, stomach cancer, kidney cancer. 4. Incorrect: This is an incorrect statement. Some amount of tobacco juice will be swallowed and can lead to esophagus and stomach cancer. 5. Incorrect: Decreased saliva is not associated with oral cancer.

What dietary information should the nurse provide to a client diagnosed with Celiac disease? Select all that apply 1. "The most cost effective way to follow the lactose free diet is to eat more fruits and vegetables." 2. "Creamed based canned soups are a source of hidden wheat." 3. "You can eat foods containing fax, corn, or rice." 4. "Avoid foods and beverages that contain malt." 5. "Do not eat traditional wheat products such as pasta."

2., 3., 4., & 5. Correct: Soups and sauces are one of the biggest sources of hidden gluten, as many companies use wheat as a thickener. It is always a good idea to read the label of any pre-prepared or canned soups and sauces, paying special attention to those that are cream based. Grains that are naturally gluten free include rice, corn, potato, quinoa, kasha, flax, and nut flours. Malt flavoring or extract, which contains gluten may be found in cornflakes and puffed rice cereal. It is also found in beers, ales, and malt vinegars. As a rule, traditional wheat products such as pastas, breads, crackers, and other baked goods are not gluten-free. However, there are many gluten-free options available that use alternative flours and grains. 1. Incorrect: The client who has Celiac disease is prescribed a gluten free diet rather than a lactose free diet.

A client admitted to a long-term care facility is legally blind and partially deaf. How would the nurse best provide for the client's safety in the event of an emergency? Select all that apply 1. Have roommate lead client out of the room to safety area. 2. Assign a specific UAP every shift to escort client to safety. 3. Research established protocols utilized by emergency groups. 4. Discuss best communication methods with client and family. 5. Plan for the supervisor to be responsible for evacuating the client.

3 & 4. Correct: When faced with a new or challenging situation involving client safety, the nurse manager should employ the Nursing Process to assess needs and collect contributing data. Asking for input from emergency preparedness groups, such as the Red Cross or FEMA, could provide ideas about assisting individuals with sensory deficits. Secondly, the nurse should discuss the situation with both client and family to determine appropriate methods of communicating with client, particularly in an emergency situation. 1. Incorrect: The responsibility for client safety should never be placed on a roommate or even family members. Staff should be accountable for client safety at all times. 2. Incorrect: Assigning a specific UAP each shift to locate and escort client to a safe area would be confusing. Changing protocols every shift creates a hazardous situation for staff and clients. 5. Incorrect: The facility supervisor is responsible for all aspects of an emergency, including activation of alarms, coordinating evacuation of staff and clients, and initiating facility emergency protocols such as closing fire doors or turning off oxygen valves. It would not be safe for the supervisor to also be responsible for a single individual.

A home care nurse is assessing a client with a forearm cast recently applied for a displaced radial fracture. What client comment should the nurse consider the priority concern? 1. "The cast feels tight on my arm." 2. "There is an odd smell inside my cast." 3. "I can't open up my fingers this morning." 4. "The pain medicine is not relieving my pain."

3. CORRECT. All the reported problems have the potential to be serious and must be investigated; however, one problem has already occurred and could permanently impair the client's mobility. The inability to extend fingers, particularly in a casted extremity, is a contracture resulting from prolonged ischemia of muscle tissues. Swelling inside the cast causes muscles to shorten and scar, leading to deformities or contractures called Volkmann contractures. Mild cases may be treated with splinting and exercise but severe cases need surgical intervention and possible even transplanted tissues with no guarantee of restored dexterity or mobility. 1. INCORRECT. Obviously a 'really tight' sensation of the arm is of great concern, since swelling could be an early indication of compartment syndrome. However, the nurse should seek further clarification from the client regarding the "tightness" and its exact location. Another problem is of even greater concern. 2. INCORRECT. Many odors could emanate from casting material, from skin breakdown to the odor of drying cast material. The client may even have put something down inside the cast so the smell should definitely be investigated. However, the nurse has greater priority at the moment. 4. INCORRECT. There are many reasons pain medication may not relieve discomfort, including too low a dose or patient noncompliance with medication regime. On-going pain should certainly be investigated as a potential sign of greater problems but this is not the nurse's priority at this time.

A client arrives at the emergency room with active gastrointestinal bleeding. What is the most important nursing action? 1. Treat the cause of the bleeding. 2. Record the amount of blood loss. 3. Initiate an intravenous access line. 4. Prepare client for stat endoscopy.

3. CORRECT: The client has active gastrointestinal bleeding, which can quickly lead to hypovolemic shock. Active bleeding would be treated with fluids, and in certain cases, blood products. Establishing an IV site allows for immediate initiation of treatment before veins vasoconstrict and become too difficult to access secondary to shock. 1. INCORRECT: While it is crucial to find and treat the cause of the bleeding, diagnosing is the responsibility of the primary healthcare provider. Because this client is experiencing internal bleeding, further tests may be needed to determine the source of the hemorrhage. 2. INCORRECT: Gastrointestinal bleeding is very difficult to measure since there is no effective way to collect the fluid. The primary healthcare provider could order a hemoglobin and hematocrit but that does not precisely measure the actual amount of blood loss. 4. INCORRECT: An endoscopy is an internal examination of a portion of the gastric system. However, proper preparation requires the client to be NPO for hours in order to properly visualize that system. This is not the most important nursing action.

Which task would be appropriate for the nurse to assign to the unlicensed assistive personnel (UAP)? 1. Check the bladder for distention in the client who had a indwelling catheter removed 4 hours ago. 2. Obtain BP of client with syncope in the lying, sitting, and standing positions. 3. Prepare a sitz bath for a postpartum client. 4. Monitor for grimacing in the client who has had a stroke.

3. Correct. The UAP can assist clients with hygiene care, so it is within the scope of practice for the UAP to assist a client with a sitz bath for the postpartum client. 1. Incorrect. This is not within the scope of practice for the UAP. The nurse must assess and evaluate.Checking the bladder for distention is an assessment that requires the nurse's attention. 2. Incorrect. This client is not stable if having episodes of syncope that could be related to orthostatic hypotension. Since the client is not stable, the UAP should not obtain the client's BP. The nurse should assess the client. 4. Incorrect. The nurse cannot delegate an assessment or evaluation task to the UAP. This is beyond the scope of practice for the UAP.

The nurse manager on a medical-surgical unit receives official notification that staff overtime must be decreased as a cost-saving measure. In order to reorganize staffing, the nurse manager should initiate which action first? 1. Announce the new changes at the monthly staff meeting. 2. Ask for any staff objections to rearranging work hours. 3. Invite staff to contribute ideas on scheduling changes. 4. Explain administration is demanding a decreased overtime.

3. Correct: The nurse manager is aware that open communication with staff is vital to increase workplace satisfaction and staff retention. One important aspect is encouraging the flow of ideas between management and staff members. Open communication and brainstorming sessions in which staff can freely share thoughts or ideas creates a positive work environment while helping decrease dissatisfaction. 1. Incorrect: While it is true that the nurse manager is ultimately responsible for implementing and announcing new schedule changes, doing so without any staff input can create discontent in the work environment. When staff do not feel vested in any new process, there is a sense of underappreciation. This perceived lack of control can create distrust and frustration among personnel, ultimately impacting client care. 2. Incorrect: Although this action appears to be opening lines of communication, the nurse manager is actually fostering animosity in a situation where the outcome is already predetermined. Allowing staff to vent is acceptable but the nurse manager should focus on constructive methods of adjustment to the impending mandated changes. 4. Incorrect: The nurse manager is aware that health care facilities often face both political and financial issues that impact staff and clients simultaneously. The responsibility of the nurse manager is to implement change in a positive manner, while assisting staff adaptation even to unpopular modifications. Assigning blame for the changes to administration will not help staff adjust.

While the postpartum nurse was in report, four clients called the nurse's station for assistance. Which client should the nurse see first? 1. Client with three dime sized clots on her perineal pad. 2. Breastfeeding client who is reporting uterine cramping. 3. Client reporting blood running down legs upon standing. 4. Client who had an epidural and is now reporting a headache.

3. Correct: A new nurse should assess this client first because we are worried about hemorrhage. If the fundus is boggy, a fundal massage will need to be done. Assess vital signs for hemorrhage. 1. Incorrect: Clots smaller than a silver dollar are normal. However, do not ignore any bleeding. Always assess the client with any signs of bleeding to determine that the problem is significant. 2. Incorrect: Breastfeeding causes the release of endogenous oxytocin from the pituitary, which causes the uterus to contract. When the uterus contracts, the client may call this discomfort, cramping. This is a normal process necessary for the uterus to return to normal. 4. Incorrect: A post epidural headache can be an indication of inadvertent puncture of the dural membrane. This client will need to be positioned prone, push fluids, given caffeine and may need a blood patch to seal the dural leak.

After drawing up insulin for subcutaneous administration, the nurse receives a return phone call from a primary healthcare provider who wants to give prescription orders on a new admit. The nurse asks a new nurse to administer the insulin dose. What action should the new nurse take? 1. Administer the insulin dose to the client. 2. Consult with the charge nurse about administering the insulin dose to the client. 3. Tell the nurse that whoever draws up the medication has to administer that medication. 4. Offer to take the call from the primary healthcare provider so the nurse can administer the insulin.

3. Correct: A nurse can only administer medication that has been drawn up by that nurse. It is not acceptable practice to administer a medication drawn up by another nurse. 1. Incorrect: The nurse who gives this medication does not really know what was drawn up. It could be the wrong medication, the wrong dose, the wrong time. A nurse can only administer medication that has been drawn up by that nurse. 2. Incorrect: There is no need to consult the charge nurse because the new nurse should not administer the medication that has been drawn up by another nurse. 4. Incorrect: The nurse should first take the return phone call from the primary healthcare provider and then administer the insulin yourself.

Which action would the nurse need to perform to increase stability while initiating a client transfer? 1. Lift with the back. 2. Put on a back belt. 3. Spread feet to width of the shoulders. 4. Lean forward slightly.

3. Correct: In order to increase stability, the nurse will need to increase the base of support. This can be done by spreading the legs to the width of the shoulders. 1. Incorrect: Do not use your back to do heavy lifting. They are not your strongest muscles. Use your legs. 2. Incorrect: A back belt will not increase the base of support. 4. Incorrect: The nurse should not lean forward or backward. The ears, shoulders, hips and feet should be aligned.

Which task would be appropriate for the nurse to assign to an unlicensed assistive personnel (UAP)? 1. Encourage client to express grief related to loss of independence. 2. Irrigate a client's ear canal. 3. Disconnect client's nasogastric (NG) tube suction to allow ambulation. 4. Show client who has conjunctivitis how to clean the eyes.

3. Correct: Disconnecting NG tube suction is an appropriate task for the UAP. 1. Incorrect: Dealing with a client's emotional state requires a formative evaluation to gauge readiness and requires the knowledge of the RN. 2. Incorrect: Irrigating a client's ear canal is outside the UAP's scope of practice. It is not a routine task. 4. Incorrect: The nurse cannot assign teaching to the UAP.

A client diagnosed with hypothyroidism has received dietary education from the nurse. Which snack selection chosen by the client would indicate that education has been successful? 1. Cup of almonds 2. Cheese and crackers 3. Popcorn 4. Sweet potato fries

3. Correct: Hypothyroidism clients tend to have constipation due to decreased motility of the GI tract and need increased fiber and fluid intake. Popcorn is high in fiber. 1. Incorrect: People with hypothyroidism have a slow metabolism and do not need high protein but a well balanced diet. Almonds are high in protein. 2. Incorrect: Cheese and crackers are high in sodium. This client is at risk for CAD, so sodium should be limited. 4. Incorrect: This client does not need high potassium, which fried sweet potatoes have. The high potassium dietary approaches to stop hypertension (DASH) diet is only for healthy clients with hypertension.

A nurse is to administer a time release capsule to a client who has difficulty swallowing. Which intervention would be the best course of action for the nurse to take? 1. Open the capsule and sprinkle it on applesauce. 2. Melt the capsule in juice or water. 3. Call the primary healthcare provider to change the order. 4. Break the capsule in half using a pill splitter.

3. Correct: If the client has difficulty swallowing a capsule or tablet, ask the primary healthcare provider to substitute a liquid medication if possible. 1. Incorrect: Sprinkling the medication over applesauce or pudding may be the only option the nurse has if there is no other form, but since this medication is time-released, the best answer and priority would be to get a liquid form, if available, for the drug. 2. Incorrect: Never melt a time release capsule or tablet as this would release the medication all at once. 4. Incorrect: Breaking or splitting would also release the medication in boluses and could cause harm to the client.

Which response by the nurse is appropriate when responding to a client who reports eliminating all dairy foods from their diet because of lactose intolerance? 1. "Take calcium tablets since they can be used as a total supplement for dairy products." 2. "You can take lactose enzymes which will eliminate the effects of lactose intolerance." 3. "Valuable nutrients found in milk include calcium and protein." 4. "Consume more leafy green vegetables to maintain calcium levels."

3. Correct: Milk contains both calcium and high-quality protein. 1. Incorrect: Dairy products provide for both calcium and protein. 2. Incorrect: Lactose enzymes may help but will not eliminate the problem. 4. Incorrect: People generally do not eat enough green leafy vegetables to get enough protein.

When planning post procedure care for a client who is having a barium enema, what must the nurse include? 1. Cardiac monitoring for potential arrhythmias 2. Monitoring urinary output 3. Administration of a laxative or enema after the procedure 4. Reordering the client's diet

3. Correct: The client must expel the barium post procedure. If the barium is not eliminated, it can harden in the colon and cause an obstruction. 1. Incorrect: It is not standard practice to place clients on a cardiac monitor after a barium enema. 2. Incorrect: Monitoring urine output has nothing to do with this procedure and does not answer the specific question related to this diagnostic procedure. 4. Incorrect: Reordering the client's diet is important but is not as life-threatening as a bowel obstruction.

A recently hired nurse is distressed that the facility's documentation system has been upgraded to a more challenging process, including the use of laptops in client rooms. The new nurse expresses concerns to the nurse manager, indicating an inability to learn the new process. What comment by the nurse manager is most appropriate? 1. "Of course you can do this, and I will help you! " 2. "Why does this new system upset you so much?" 3. "It is hard to deal with so many changes at once." 4. "This is so easy, even a child can learn how to do it."

3. Correct: This open-ended statement provides the opportunity for the nurse to express feelings not just about the new system, but also regarding any additional concerns. This option focuses on the new nurse's emotions rather the just the computer changes. 1. Incorrect: Although this response by the nurse manager may seem encouraging, particularly with the offer to help, it dismisses the new nurse's emotions. This is a closed-ended response that does not encourage the further expression of feelings. 2. Incorrect: Anytime a question demands an explanation, the situation is non-therapeutic. The nurse manager is making the assumption it is only the new system which is upsetting the nurse, and is focusing on the charting rather than the new nurse's feelings. 4. Incorrect: Although the nurse manager may have meant to be encouraging, this statement is rather demeaning. This does not create a positive learning environment and may discourage the new nurse even further.

A nurse is evaluating an unlicensed assistive personnel (UAP) for proper body mechanics while lifting a heavy object off of the floor. What action by the UAP would indicate a need for further instruction by the nurse? 1. Testing the weight to determine if additional assistance is needed. 2. Keeping the feet shoulder width apart. 3. Bending from the waist to pick up the object. 4. Holding the object close to the body upon rising.

3. Correct: You should not bend at the waist. This will injure your back. Lower your knees, and stay close to the object to use thigh muscles. 1. Incorrect: This is correct. If it is too heavy do not attempt to lift alone. 2. Incorrect: This is correct and will help maintain balance as you lower yourself to the floor. 4. Incorrect: This is correct. Holding close to the center of gravity will help prevent injury to your back and arms. Body mechanics describes the ways we move as we go about our daily routine. Poor body mechanics are often the cause of back problems. When we don't move correctly and safely, the spine is subjected to abnormal stresses that over time can lead to degeneration of spinal structures. The process of lifting places perhaps the greatest loads on the low back and therefore, has the highest risk of injury. Use of proper lifting mechanics and posture is critical to prevent injury. How to lift safely: Place the load immediately in front of you. Bend the knees to a full squat or lunge position. Bring the load towards your chest. Assume a neutral position with your back. Tighten the lumbar and buttocks muscles to "lock" the back. Lift now from the legs to the standing position. DO NOT Lift from a twisted / sideways position. DO NOT Lift from a forward stooped / imbalanced position.

What should the nurse emphasize when teaching clients how to decrease the risk of chronic obstructive pulmonary disease? Select all that apply 1. Avoid exposure to individuals with respiratory infections. 2. Increase intake of Vitamin C. 3. Eliminate exposure to second hand smoke. 4. Avoid prolonged exposure to occupational dusts and chemicals. 5. Get a yearly influenza and pneumococcal vaccination.

3., & 4. Correct: The most important environmental risk factor for COPD is cigarette smoking. Second hand smoking also contributes to COPD. Risk factors for COPD include prolonged and intense exposure to occupational dust and chemicals as well as indoor and outdoor air pollution. 1. Incorrect: Exposure to individuals with respiratory infections does not increase risk of chronic obstructive pulmonary disease. Respiratory infections may cause an acute exacerbation in a client with existing COPD. 2. Incorrect: Increasing intake of vitamin C does not decrease risk of obstructive pulmonary disease. 5. Incorrect: Clients should get the influenza vaccine annually in autumn. The pneumococcal vaccine should be administered every 5 years, rather than yearly.

The nurse is evaluating an elderly bedridden client for possible fecal impaction. What sign/symptom should the nurse report as most indicative for a fecal impaction? 1. Rigid, board-like abdomen 2. Absence of any bowel sounds 3. Diarrhea with severe cramping 4. Constipation with liquid seepage

4. CORRECT. A client may have several symptoms with a fecal impaction; however, the most classic symptom which the nurse should report is the presence of constipation in the presence of liquid fecal seepage from the rectum. The client may also report abdominal distention with a feeling of fullness, cramping or even painful defecation. 1. INCORRECT. Although the client may report abdominal pain, a rigid, board-like abdomen is indicative of peritoneal inflammation such as peritonitis. As pain increase, the abdominal muscles respond by becoming rigid, hence giving the board-like appearance. 2. INCORRECT. A complete absence of bowel sounds is indicative of other bowel issues, such as a paralytic ileus, which occurs when peristalsis has completely stopped. That is not the case with an impaction. 3. INCORRECT. Diarrhea, especially with cramping, may be attributed to a variety of illnesses, including gastroenteritis, Crohn's disease or even ulcerative colitis. Additionally, diarrhea is not the same as liquid seepage.

The nurse is caring for a client with tuberculosis receiving isoniazid therapy. Because of the possible peripheral neuropathy that can occur, which supplementary nutritional agents would the nurse expect to administer? 1. Cyanocobalamin 2. Vitamin D 3. Ascorbic acid 4. Pyridoxine

4. Correct: Isoniazid interferes with vitamin B6 (pyridoxine) metabolism by inhibiting the formation of the active form of vitamin B6. This interference often results in peripheral neuropathy. 1. Incorrect: Vitamin B12 (Cyanocobalamin) is not given to prevent peripheral neuropathy caused from isoniazid therapy. It is used to treat vitamin B12 deficiency often caused by pernicious anemia. It may be given in client's with peripheral neuropathy, but is not beneficial in clients whose neuropathy is due to isoniazid therapy. 2. Incorrect: Vitamin D is not given to prevent peripheral neuropathy. It is used in the treatment of weak bones, bone pain and/or bone loss. 3. Incorrect: Vitamin C is not given to prevent peripheral neuropathy cause from isoniazid therapy. It's use can be beneficial in clients with diabetic peripheral neuropathy.

The pediatric nurse is assessing a child following an appendectomy. What is the nurse's main priority following surgery? 1. Obtain vital signs every four hours. 2. Assess the need for pain medication. 3. Tally intake and output every eight hours. 4. Auscultate lung sounds every four hours.

4. CORRECT: No matter what type of surgery, recall that the effects of anesthesia and intubation, if performed, can lead to complications, particularly in children. The potential for atelectasis and pneumonia follows surgery; therefore the client is encouraged to cough and deep breathe to minimize these risks. Auscultating lung sounds frequently post-op is crucial. 1. INCORRECT: Although vital signs are important, initially the nurse should check vitals every half hour to one hour. Despite the frequency, another assessment is even more important. 2. INCORRECT: It is crucial to medicate a postoperative client; however, pain medications should never be administered until after the initial assessment as pain medication will alter important symptoms the nurse needs to determine any complications. 3. INCORRECT: Standard intake and output is tallied once a shift, or every eight hours. Though this information is vital to determine hydration and function of the kidneys, it is not the nurse's main priority.

The nurse is reviewing morning laboratory results on four clients. Which lab finding should the nurse report to the primary healthcare provider immediately? Exhibit Client Lab Values: Client diagnosed with deep vein thrombosis who is receiving a heparin infusion - aPTT 85 sec. Client diagnosed with possible appendicitis - WBC 18,000 per mm3 Client diagnosed with rheumatoid arthritis - Sed rate 100 mm/hr Client diagnosed with congestive heart failure receiving furosemide - K + 2.9 mEq/L 1. aPTT 2. WBC 3. Sed rate 4. K+

4. CORRECT: Notice that all laboratory results are abnormal, based on the disease process of each client. However, the potassium level for the cardiac client is way below normal, most likely secondary to the furosemide. Levels that low can result in premature ventricular contractions (PVCs) or other arrhythmias, placing the client at risk for sudden onset of CHF. 1. INCORRECT: The client is on the heparin infusion for a diagnosed deep vein thrombosis (DVT). While normal aPTT levels should be between 20 - 36 seconds, the therapeutic levels of heparin are usually 2 ½ to 3 times normal to keep the blood thin. This result is expected and not alarming at this point. 2. INCORRECT: Appendicitis is a serious infection that is treated with either antibiotics or surgery to remove the organ. While normal WBC values are 5,000 to 10,000, this elevated result is not unexpected for an infection. 3. INCORRECT: A Sed rate or sedimentation rate, reveals inflammatory activity in the body and can be used to diagnose or monitor the status of an inflammatory disease process. The blood cells affected (erythrocytes) will settle to the bottom of a blood tube and that speed indicates the severity of the inflammatory process in the body. This is not an unexpected result in clients with rheumatoid arthritis.

Which nursing intervention should the nurse include when caring for a client with Alzheimer's disease being admitted to a long term care facility? 1. Offer multiple environmental stimuli at the same time to provide distraction. 2. Encourage the client to participate in activities such as board games. 3. Restrain the client in a chair to prevent falls when sundowning occurs. 4. Involve the client in supervised walking as a routine.

4. Correct: A regular routine and physical activity help client's with Alzheimer's disease maintain abilities for a longer period of time. Physical activities promote strength, agility and balance. The client's walking should be supervised for client safety issues. 1. Incorrect: Environmental stimuli should be limited with clients with Alzheimer's Disease. The client can become agitated and/or more disoriented with an increase in environmental stimuli. 2. Incorrect: Board games would not be appropriate due to the client's cognitive and memory impairment. Board games require complex cognitive actions. 3. Incorrect: Restraints should be avoided because they increase agitation. The client may become agitated by the restriction of he restraints. Also the client may perceive the restraints as a threat.

The nurse is caring for a client on the oncology unit. The client asks, "Why do I need this LifePort to receive my chemotherapy?" What evidence should the nurse consider when answering? 1. IV infusions can be more rapidly administered via an implantable IV port 2. Implantable IV ports are kept sterile and therefore do not become infected 3. Chemotherapeutic agents are more readily absorbed from implantable IV ports 4. Implantable ports are beneficial when long-term and/or multiple IV therapy is indicated.

4. Correct: Clients requiring long-term and/or multiple IV therapy benefit from implantable ports, because they reduces the number of IV sticks, preserve the integrity of peripheral veins, and provide a vessel with adequate blood flow. The part allows chemotherapy agents to be given in a larger vein, decreasing risk of tissue damage that can occur with peripheral administration. 1. Incorrect: Rate of administration is not an indicator for an implantable port, and chemotherapeutic agents are administered at a slower rate than most IV medications. Chemo agents should be given at the prescribed rate. 2. Incorrect: Infection is a concern for any implantable device. Sterile technique is used when accessing port. Inspection of the site is essential, in addition to monitoring vital signs and WBCs. 3. Incorrect: Rate of absorption is not affected by the type of central line or implantable IV port. Implantable ports promote safety and reduce problems during medication administration.

A paralyzed adolescent admitted for decubiti debridement has brought multiple personal electronics, including a laptop, cell phone and video game unit. The nurse notes the family has used extension cords to provide enough electrical outlets. What action by the nurse is most appropriate? 1. Inform family some of the electronics must be taken home. 2. Explain that extension cords are not permitted in a hospital. 3. Notify maintenance to install more outlets in the client room. 4. Ask client to have staff switch equipment in outlets as needed.

4. Correct: Extension cords are considered a safety hazard in the hospital setting, especially when provided by the family. The nurse has provided an alternative in order for the client to use personal equipment. Staff will assist the client to switch equipment when requested. 1. Incorrect: The nurse is violating the client's right to keep personal belongings while providing no alternative suggestion for the client. Additionally, the nurse has not addressed the use of extension cords in the hospital. 2. Incorrect: Though the nurse is making an accurate statement, no alternative plan has been offered to help meet the client's needs. 3. Incorrect: It is not possible to have maintenance perform construction in a room currently occupied by a client. Installing outlets for one client is not feasible.

How does the nurse identify the correct size of crutches for a client? 1. Turn the crutches upside down and measure from the heel to the shoulder. 2. Obtain a set of crutches and adjust the height until the client can stand comfortably while resting the axilla on the crutch pad. 3. Measure the client while standing upright from the axilla to the heel then adjust the crutches so that the elbow flexion is a 30-degree angle. 4. Measure the client from 2 inches below the axilla to 6 inches lateral to the client's heel.

4. Correct: Measuring the client from 2 inches below the axilla to 6 inches lateral to the client's heel correctly measures a client for crutches. This is the correct size while a client is standing. 1. Incorrect: This is not the correct way to choose the correct size crutches. Without the proper fit safety is a concern. 2. Incorrect: This is not how to choose the correct size of crutches. The client should not rest their weight on the crutch pad as this can cause damage to the brachial plexus nerve. 3. Incorrect: This is not how to choose the correct size of crutches. The shoulders should be relaxed, the hand piece should be adjusted to provide a 20°- 30° elbow flexion. The 2 inch drop below the axilla allows the weight to be pressed against the sides and the hands absorb the weight. The crutch should not be placed against the axilla or the brachial plexus nerve could be damaged.

A nurse enters a client's room to find the client on the floor having a seizure. Which nursing action is appropriate for this client? 1. Hold the client's arms and legs. 2. Insert a padded tongue blade in the client's mouth. 3. Assist the client back into the bed. 4. Place a rolled towel under the client's head.

4. Correct: Placing a rolled towel under the client's head prevents further injury to the client. 1. Incorrect: Restraining the client may cause further injury to the client. 2. Incorrect: Forcing an object into the client's mouth can result in choking the client or injuring the client's teeth and mouth. 3. Incorrect: Lifting the client may cause injury to the nurse and client.

What task would be appropriate for a nurse caring for a client diagnosed with gastroesophageal reflux to delegate to an unlicensed assistive personnel (UAP)? 1. Inform the client of the need to avoid irritants such as carbonated beverages. 2. Ask client if they are eating small, frequent meals. 3. Monitor for GI upset 30 minutes after meals. 4. Remind the client to avoid tight fitting clothes.

4. Correct: The UAP can remind the client to do something that has already been taught by the nurse. 1. Incorrect: Informing is the same thing as teaching. Although this is a correct thing to inform the client, this teaching should be done by the RN and not delegated to the UAP. 2. Incorrect: The RN is responsible for collecting data. 3. Incorrect: The RN is responsible for assessment and evaluation.

A client has been on the nursing unit for two hours following a retropubic prostatectomy for the treatment of prostate cancer. The client is receiving a continuous bladder irrigation of normal saline infusing at 1000 mL/hr. The client's urine output for the past two hours is 410 mL. What is the nurse's first action? 1. Inspect the catheter tubing for obstruction. 2. Irrigate the catheter with a large piston syringe. 3. Notify the primary healthcare provider. 4. Stop the irrigation flow.

4. Correct: The catheter output should be at least the volume of irrigation input plus the client's actual urine. A severe decrease in output indicates obstruction in the drainage system. The first action is to stop the irrigation flow to prevent further bladder distention. Bladder distention is one of the main causes of hemorrhage in the fresh post op period. 1. Incorrect: The next action is to check the external system for kinks or obstruction to assess if this is the cause of the decreased urine output. Obstruction of the catheter tubing can also cause bladder distention. 2. Incorrect: After the external system is checked for kinks or obstruction, and the client's urine output doesn't change, then the catheter is irrigated with 30 to 50 mL of normal saline using a large piston syringe. However, irrigating a new post-op client is not the safest or first action for the nurse. 3. Incorrect: Of the options listed here, this is the last intervention. If the obstruction is not resolved after irrigating the system, the primary healthcare provider must be notified.

An infant has been prescribed Bryant's traction for a diagnosis of developmental dislocated hips (DDH). At what degree of hip flexion should the nurse maintain the infant's hip for proper traction alignment? 1. 15 2. 30 3. 45 4. 90

4. Correct: Bryant's traction is used for DDH. The child's body and the weights are used as tension to keep the end of the femur in the hip socket. Traction helps position the top of the femur into the hip socket correctly. This is accomplished with 90 degrees of hip flexion. 1. Incorrect: Fifteen degrees of flexion is not adequate to keep the femur end in the hip socket. 2. Incorrect: Thirty degrees of flexion is not adequate to keep the femur end in the hip socket. 3. Incorrect: Forty-five degrees of flexion is not adequate to keep the femur end in the hip socket.

Which medication should the nurse administer first after receiving the morning shift report? 1. Levothyroxine to the client with hypothyroidism and a thyroid stimulating hormone (TSH) level of 2.8 mU/L 2. Amlodipine to the client with hypertension and a blood pressure of 150/86 3. Regular insulin sliding scale dose to the client with diabetes and a 210 blood glucose level. 4. Cefotaxime intravenous piggyback to the newly admitted client with a diagnosis of pneumonia and a white blood cell count (WBC) of 12,000mm3

4. Correct: The first dose of intravenous antibiotic medication is the priority since the WBCs are elevated and the antibiotic should be administered first. 1. Incorrect: The TSH is normal so the thyroid medication is not the priority. 2. Incorrect: Amlodipine is for high blood pressure and is important but the antibiotic is the priority. 3. Incorrect: It is important to administer the regular insulin but it is not priority over initiating the intravenous antibiotic medication.

A client states, "I really do not want to go through open heart surgery. I have told my children this, but they still want me to go through with the surgery. I don't know what to do." What is the best response for the nurse as client advocate? 1. Your children are correct. The open heart surgery is the best thing for your health. 2. You feel as if your children are not addressing your concerns. You and your family will need to resolve this before you go to surgery. 3. I can contact your primary healthcare provider so that you can discuss your concerns regarding open heart surgery. 4. You have some genuine concerns about the open heart surgery, and you feel as if your children are not addressing your concerns.

4. Correct: The nurse has a duty to advocate for the client if there is a discrepancy between the care or proposed care and the client's wishes regarding treatment. It is important to acknowledge the client's feelings, and to demonstrate compassion and a willingness to understand. This presents an opportunity for additional communication to help answer some of the client's questions, or set up a client-family conference with the client, the client's family, and the primary healthcare provider. 1. Incorrect: When the nurse agrees with the client's children, the nurse ignores the client's feelings and does not address the issue of the client's treatment wishes. 2. Incorrect: When the nurse restates the client's comment without investigating the client's concerns, the issue goes unresolved. 3. Incorrect: Offering only to contact the primary healthcare provider is an incomplete solution and hints of the nurse not taking responsibility to investigate the client's concerns. The client may be uncomfortable addressing concerns with the primary healthcare provider before resolving the issue of treatment wishes with family members.

A client is scheduled for a colonoscopy with biopsy of a large tumor that is completely blocking the large intestine in the morning. Which preoperative prescription should the nurse question? 1. Administer tap water enemas until clear at 6 AM. 2. Nothing by mouth (NPO). 3. IV of D5 ½ NS at 75 mL/hour with a 20 gauge catheter. 4. Give magnesium citrate 296 mL at 3 PM today.

4. Correct: Did you notice the hint? Complete blockage of the large intestine. If you give the magnesium citrate, which is a laxative, what will happen? Nothing will get passed the complete blockage. The client would develop severe cramping. This could cause a medical emergency. 1. Incorrect: The client does need to be cleaned out below the tumor so that the primary care provider can see the area of concern and complete the biopsy. Since the enema would clean below the obstruction, the client would be able to expel the enema and any feces in this part of the colon. 2. Incorrect: Since this client has an obstruction, anything the client eats will not be able to come out. This is an appropriate prescription. 3. Incorrect: The client will need IV access for sedation during the procedure and will need the fluid for hydration since the client is NPO.

A client is to be discharged following cataract removal with lens implantation. What statement by the client indicates to the nurse that teaching has been successful? 1. "I must keep both eyes covered till my check-up." 2. "I should only have pain for about two days." 3. "I will no longer have to wear reading glasses." 4. "My vision will be blurry for a couple weeks."

4. Correct: Following cataract removal, a new lens is sutured in place, which slightly alters the corneal curve. Newer surgical approaches involve the use of a "suture-less glue" but that method is less common. Although the client's vision will eventually improve and stabilize, minor blurring may exist during the 6 to 12 week healing period. After that time, any remaining visual issues can be corrected with glasses. 1. Incorrect: Only the operative eye is protected by an eye patch during the healing process. The primary healthcare provider will remove that covering at the first post-operative checkup. Covering both eyes would pose a greater safety risk and decrease the client's self care abilities. 2. Incorrect: Pain following cataract surgery is the sign of a serious complication and should be reported to the surgeon immediately. Clients may experience a small amount of serous drainage or scratchy sensation, but should not have pain. 3. Incorrect: Implantation of a new lens causes a mild astigmatism that will be permanent. The client may still need to use corrective lenses, even if just for reading.

A child is admitted to the emergency department due to suspected ruptured appendicitis with perforation. What would be the priority nursing assessment for this client? 1. Monitor for the Rovsing sign. 2. Assess for an increase in temperature. 3. Check for rebound tenderness at McBurney's point. 4. Monitor for increasing pain and rigidity of the abdomen.

4. Correct: Increasing pain and rigid, board-like abdomen are signs that the appendix may have ruptured, with resulting peritonitis developing. 1. Incorrect: The Rovsing Sign results in RLQ pain that occurs with palpation of the LLQ. This suggests peritoneal irritation due to palpation of a remote location and would indicate appendicitis. 2. Incorrect: Although children with appendicitis may have an elevated temperature, the priority would be assessing for the signs of peritonitis which include increasing pain and rigidity of the abdomen. Children can have an increased temperature with many different types of inflammation and infections. 3. Incorrect: Although rebound tenderness at McBurney's point is indicative of appendicitis, the nurse should not check for this due to the possibility of rupturing the appendix.

The nurse is preparing to hang an IV bag of Heparin after receiving a prescription from a client's primary healthcare provider: Heparin IV to infuse at 1000 U/h. What flow rate should the nurse set the IV infusion pump rate at? Round to the nearest whole number. Exhibit IV Heparin Infusion Label: Heparin Sodium 25,000 USP Units Added to 0.45% Sodium Chloride 250 mL Bag (100 USP units/mL)

Answer: 10 Rationale: Prescription: Heparin IV to infuse at 1000 U/h. Available: Heparin 25,000 units in NS 250 mL Use ratio-proportion to calculate flow rate in mL/h which will administer 1000 units/hour. 25,000 units/250 mL = 1000 units/X mL 25,000X = 250,000 X = 10 mL/hr

In what order will the nurse provide instructions to a client on using a cane? Advance weaker leg forward toward the cane. Move cane forward 6-10 inches (15 - 25 cm). With cane on stronger side of body, support body weight with both legs. Advance stronger leg forward toward cane. Drag and Drop the items from one box to the other

First, with cane on stronger side of body, support body weight with both legs. This will support the even distribution of weight away from the weaker side to promote a normal gait. Second, move cane forward 6-10 inches (15-25 cm). Moving the cane the approximate distance of a normal gait helps with stability. Third, advance weaker leg forward toward the cane. This allows the weight to be supported by the cane and the stronger leg. Fourth, advance stronger leg forward toward the cane. This allows the weight to be supported by the can and weaker leg.

A newly hired nurse has been instructed by the preceptor nurse on burn dressing techniques. The nurse knows teaching has been effective when the new nurse performs wound care in what order? Wash hands and apply clean gloves. Remove the old dressing and discard. Set up sterile field and open packages. Clean burn and place sterile dressing. Wash hands and apply sterile gloves. Medicate client with pain medication. Drag and Drop the items from one box to the other

The correct protocol for changing burn dressings consistently follows a specific pattern. The client must first be medicated for this painful procedure, and at least 30 minutes in advance so the drug has time to work. The nurse must then wash hands thoroughly and apply the clean (non-sterile) gloves. Depending on the type and extent of burns, the nurse may also need a gown to prevent contaminating the client. Although not mentioned here, the nurse would most certainly explain the procedure to the client, which could actually be accomplished while the nurse sets up the sterile field, opens sterile packages and pours the ordered cleansing fluids. Once properly set up, the nurse will gently remove the old dressing and discard along with the non-sterile gloves, per facility protocols. After washing hands a second time, the nurse will apply the sterile gloves to complete care. The burned area is cleaned, prescribed antibiotic cream is applied, and a new sterile dressing placed over the burn.

Four clients arrive for their appointment at a diabetic clinic. In what order should the nurse see the clients? Client eating a simple-carb snack due to weakness. Client reporting a headache and has a fruity breath. Client scheduled for a dressing change to foot ulcer. Client to receive dietary education. Drag and Drop the items from one box to the other

The first client needing the nurse's attention is the one reporting a headache and has a fruity odor to their breath. Remember, pick the killer answer first! This client is likely in metabolic acidosis due to diabetic ketoacidosis (DKA). What was the hint? Fruity breath. The second client that needs to be seen by the nurse is the client having weakness, a sign of hypoglycemia. This is a diabetic clinic. This client is eating a simple carb snack, but the nurse needs to check the client's blood glucose level to see if the snack has helped. The third client would be the one needing a dressing change. Nothing life threatening, but an assessment needs to be made regarding the ulcer. The last client would be the one needing dietary education. Nothing life threatening. This client can wait until the others are treated.

The nurse is working with a new unlicensed assistive personnel (UAP) on a post-operative unit. The nurse received a client following surgery 8 hours ago. The first vital sign check was performed by the nurse. As the evening progressed, the unit tasks became very demanding and the nurse had to delegate several actions to the UAP. In planning care for the post-operative client, the nurse has decided to retain the task of vital sign assessment. What was the rationale for this plan? 1. The nurse did not trust the new UAP. 2. The nurse prefers to check all vital signs on all clients. 3. The nurse is responsible for the assessment of all vital signs of post-op clients. 4. The nurse does not know the skills of the new UAP.

4. Correct: The nurse has not been able to determine the skill of vital sign assessment for this new UAP. When the licensed person cannot determine this, the task should not be delegated. This determination is needed to assure client safety is being considered. 1. Incorrect: The nurse may trust the UAP; however, the nurse has not been able to determine the competency of the new staff member. Since this is a postoperative client, It is important that the vital sign measurement is accurate to detect any changes or possible complications. 2. Incorrect: When a unit is very busy, the nurse should rely on the UAP if the person is competent to perform the tasks. In this situation, it is not a matter of the nurse preferring to take all the vital signs, but the nurse needs to know the competency level of the UAP before delegating this task. 3. Incorrect: The nurse can measure vital signs; however, agency policy usually states that UAP can perform this task also. If the client is unstable, the nurse would retain the role of measuring the vital signs. Once the client is stable, the UAP could perform this task. However, since the new UAP's competency level is not known, the nurse does not delegate this task for the safety of the client.

Labetalol has been prescribed for a client in the emergency room. Prior to administering this medication, what assessment should the nurse perform? 1. Listen to the client's breath sounds. 2. Check the client's temperature. 3. Monitor for peripheral edema. 4. Auscultate the apical pulse rate.

4. Correct: The therapeutic effect of labetalol, which is a beta blocker, is to lower the blood pressure and decrease the heart rate. Apical pulse should be assessed for 1 full minute. If pulse is less than 60 the medication is held and the healthcare provider should be notified. 1. Incorrect: Indirectly a beta blocker could affect breath sounds but assessing breath sounds is not as important as taking the client's apical pulse. Beta blockers should be used cautiously in clients with a history of COPD or asthma these could cause airways to constrict. 2. Incorrect: Labetalol does not affect the client's temperature. This is not a side effect of labetalol. 3. Incorrect: Indirectly a beta blocker could affect the amount of peripheral edema, however, assessing for peripheral edema is not as important as taking the client's apical pulse.

The nurse is assisting a new mother with breastfeeding her newborn baby. The mother verbalizes concern that the baby is not getting adequate milk. Which observations by the nurse indicate adequate fluid intake? Select all that apply 1. Birth weight regained in 14 days 2. Fontanels soft and depressed 3. Pulse rate of 135/min 4. Six to eight wet diapers a day 5. Baby appears satisfied after feedings

1., 3., 4. & 5. Correct: Are all indicators of adequate fluid intake in a newborn. Gaining weight, a heart rate between 70 to 190 beats per minute (BMP), six to eight wet diapers a day and periods of contentment after feedings alternate with periods of wakefulness indicate adequate breastfeeding. 2. Incorrect: Fontanels should be soft, firm and flat. A depressed or sunken fontanel may indicate dehydration. Dehydration is one of the major causes of sunken fontanels.

What action should the nurse take after mistakenly administering the wrong medication? Select all that apply 1. Notify the nursing supervisor. 2. Inform the primary healthcare provider. 3. Complete an incident (variance) report. 4. Document client assessment and response to medication. 5. Document medication error and incident (variance) report in nurse's notes.

1., 2., 3., & 4. Correct: Nurses must immediately report all client care issues, concerns or problems to the supervising nurse, the primary healthcare provider and/or the performance improvement or risk management department. A written report of the incident is completed by the nurse and turned into the appropriate person (generally the performance improvement department). Documentation of what occurred, and the client's assessment is required in the nurse's notes. 5. Incorrect: Do not document that an error was made or that an incident (variance) report was completed. Document what medication was given, the client's assessment, the notification of the nursing supervisor, and primary healthcare provider, and any prescriptions received.

A client rescued from a house fire is being treated for burns to both arms and suspected inhalation injury. What data collected by the nurse has the highest priority? 1. Estimation of total surface burn area 2. Characteristics of cough and sputum 3. Calculation of client weight and age 4. Extent of edema to arms

2. CORRECT: A client rescued from a burning house is presumed to have inhaled superheated air during that process. Though calculating fluid replacement is vital to the client's survival, the ABCs dictate the highest priority is airway. Noting any cough or sputum can help determine whether prophylactic intubation may be necessary. 1. INCORRECT: The total amount of body surface burned is crucial information needed to determine fluid replacement using the Parkland Formula. However, though IV fluids are necessary, calculating the burn percentage is not the highest priority. 3. INCORRECT: The client's age is not an immediate priority, although a complete health history will be essential to the final outcome. The client's weight will be used to calculate fluid replacement; however, there is a higher initial priority. 4. INCORRECT:. A burn causes cellular damage that leads to edema. Depending on the location and extent of that edema, circulation could be greatly impaired. However, when monitoring a burned client, the ABCs place circulation third on the priority list.

The nurse observes an unlicensed assistive personnel (UAP) performing AM care for a client with a plaster leg cast applied 12 hours ago. Which action by the UAP should the nurse intervene? 1. Lifting the affected leg with the palms of the hands 2. Covering the affected leg with a blanket to avoid chills 3. Placing plastic over the entire cast prior to bathing 4. Elevating the casted leg on two pillows

2. Correct: Plaster cast take 24-48 hours to completely dry. During this time they release heat. The new cast should not be covered so that heat from the cast can evaporate. If the heat is not allowed to dissipate, drying will take longer. 1. Incorrect: Plaster cast take 24-48 hours to completely dry. During this time the cast should be handled carefully as to avoid indentations. Handling the cast carefully with the palms and not the fingertips will prevent indentations. Indentations in the cast could cause skin breakdown inside the cast. 3. Incorrect: Yes, will keep cast from getting wet. Plaster casts should never get wet. The plaster cast does not hold up well in water. Wet casts can also irritate the skin underneath. 4. Incorrect: Yes, may elevate the leg. Fractures are prone to swelling. Elevating fractures while casted is a common occurrence. Elevation prevents swelling.

A client diagnosed with hypertension has been prescribed metoprolol. Which statement by the client indicates that the client's medication instruction for metoprolol has been effective? 1. "I should not stop taking this drug immediately." 2. "I will need to rinse my mouth with water 3 times a day." 3. "I can decrease my aerobic exercises from 3 to 2 times per week." 4. "I will report irregular heartbeats, if they continue for more than 3 days."

1. Correct: Metoprolol, a beta-adrenergic antagonist, should not be discontinued abruptly. This action may have the serious result of precipitating angina. Metoprolol should be gradually discontinued. 2. Incorrect: Dry mouth is not a side effect of metoprolol. This drug does not stimulate anticholinergics to block acetylcholine from binding to its receptors on certain nerve cells. 3. Incorrect: Lifestyle modifications by the client should be continued. The client should not reduce the number of aerobic exercises after metoprolol has been prescribed. 4. Incorrect: The client should monitor their pulse rate, quality and rhythm daily. If changes in the quality and rhythm of the pulse occur, the primary healthcare provider should be notified immediately. A cardiovascular side of effect of metoprolol is bradycardia.

The nurse receives new primary healthcare provider prescriptions on a client diagnosed with Addison's disease. What prescription should the nurse question? Select all that apply 1. Weigh QD 2. IV of Normal Saline at 125 mL/hr 3. MRI of pituitary gland 4. Fludrocortisone acetate 0.1 mg by mouth T.I.W. 5. Dehydroepiandrosterone (DHEA) 5 mg by mouth every other day

1., & 4. Correct: QD is listed on the Joint Commission on Accreditation of Healthcare Organizations (JCAHO) official "do not use" list of abbreviations. This should be prescribed as "daily" instead of "QD". The abbreviation T.I.W. stands for three times a week; however, it is an unapproved abbreviation. Use "three times a week". 2. Incorrect: This is a correct action, for a client with Addison's disease, and it is written properly. 3. Incorrect: The primary healthcare provider may suggest a MRI scan of the pituitary gland if testing indicates the client might have secondary adrenal insufficiency. This is an approved abbreviation. 5. Incorrect: This is written correctly and may be given to women to treat androgen deficiency in cases such as this client with Addison's disease.

A client who was diagnosed with amyotropic lateral sclerosis (ALS) has been immobile for 2 weeks. Which of the nursing interventions would the nurse implement? Select all that apply 1. Explore diversional activities. 2. Perform range of motion exercises. 3. Maintain the feet in dorsiflexion position. 4. Assess pressure points for skin changes. 5. Encourage a fluid intake of 1500 mL/24 hours.

1., 2., 3., & 4. Correct: The client's immobility may lead to apathy and isolation. The nurse should explore diversional activities which can reduce the frustration and depression of being immobile. Immobility will result in muscle weakness and decreased venous return. The client is encouraged to perform active range of motion exercises. Also passive range of motion exercises should be performed if the client cannot perform the active exercises themselves. Due to the client's decreased movement of the ankles, the client's feet should be positioned in the dorsiflexion position to prevent plantar flexion contractures. A bed board should be positioned to the foot of the bed. Active and passive range of motion exercises to the ankle and foot will promote proper joint movement. An immobile client's skin is affected by extrinsic, intrinsic, and shear forces. A decrease in the client's perfusion and peripheral circulation are intrinsic factors. The immobile client is experiencing the extrinsic factor of increased skin temperature at the skin pressure points. Moving the immobile client may result in a shearing force on the skin. 5. Incorrect: The effects of immobility on the urinary system may include urinary retention, renal calculi, and urinary tract infections. Also the immobile client may experience constipation if the fluid intake decreases. The fluid intake for a healthy adult is recommended at 2200 - 2700 mL per 24 hours.

A client with a history of alcoholism arrives at the clinic reporting severe abdominal pain with nausea and vomiting. What additional findings would make the nurse suspect the client may have pancreatitis? Select all that apply 1. Afebrile 2. Cullen's Sign 3. Pain relieved after eating 4. Positive Chvostek's sign 5. Tachycardia.

2 and 5. CORRECT: Whether the client is experiencing acute or chronic pancreatitis, symptoms are severe and distinct. Bruising around the umbilicus is referred to as "Cullen's Sign", indicating internal bleeding. Because of inflammation in the pancreas, the client generally becomes febrile and has pain. Both causing tachycardia. 1. INCORRECT: Because of inflammation in the pancreas, the client generally becomes febrile. 3. INCORRECT: Pain is worse with eating. 4. INCORRECT: A positive Chvostek's sign is an indication of low levels of calcium in hypocalcemia, occurring as a twitching occurs when the nurse taps the client's cheek. This sign does not relate to pancreatitis.

A client diagnosed with advanced cirrhosis is admitted with dehydration and elevated ammonia levels. While discussing dietary issues, the client requests larger portions of meat with meals. Which response by the nurse provides the most accurate information to the client? 1. I will ask the dietician to add more meat with dinner. 2. Protein must be limited because of elevated ammonia levels. 3. You need to drink more fluids because of your dehydration. 4. We can ask for between meal snacks with more carbohydrates.

2. Correct: Normally, protein is broken down into ammonia, which the liver converts into urea, and the kidneys then easily excrete. However, in a diseased liver, this conversion is not possible, and ammonia continues to build up in the body, ultimately affecting the brain. The nurse would be aware that additional protein would be harmful for this client. 1. Incorrect: Increasing meat at mealtimes would be detrimental to the client's health. When protein is taken into the body, a healthy liver will convert this into urea that is then excreted by the kidneys. However, this client's impaired liver is not able to make that conversion; therefore, the ammonia levels would continue to increase. The nurse can discuss with the client other foods that might safely be added to meals. 3. Incorrect: While it is true this client is dehydrated, the issue is that the client wants to increase the amount of meat at mealtimes. This response does not address the client's request nor does it provide any teaching that would help the client once discharged. 4. Incorrect: Although this response indicates that the nurse is focusing on the client's issue with food, this reply does not address the request for more meat with meals. This would be the appropriate opportunity to educate the client on the need to limit daily protein in the diet.

Which nursing intervention should the nurse implement when administering a medication through a nasogastric (NG) tube? 1. Place the client in a high-Fowler's position for medication administration. 2. Flush the tubing between administering medications 3. Turn the client onto their left side after medication administration. 4. Mix the medication directly into the tube feeding

2. Correct: The NG tube should be flushed with appropriate facility approved amount of fluid between medications. The amount of the flushing solution should be added to the intake amount. 1. Incorrect: Semi-Fowler's position is the position of choice for administering tube feedings. This position helps prevent aspiration and promotes digestion. The volume of fluid administered with medication administration is usually much smaller than with tube feedings, so high-Fowler's is not required. 3. Incorrect: The left side position slows gastric emptying, which could lead to aspiration. The right side is the position that best promotes gastric emptying. 4. Incorrect: Do not mix medications in the enteral feeding solution. The tube feeding rate may be prescribed at different rates or the tube feeding can be held for a designated time. The proper administration of the medication could not be determined.

Several clients have reported to the charge nurse that they are not receiving pain relief when a certain RN administers their pain medication. The charge nurse has noticed that the RN has been looking unkempt in appearance and seems to be in a daze much of the time. What is the most appropriate action for the charge nurse to take? 1. Lessen the nurse's client assignment to see if things improve. 2. Discuss the concerns directly with the nurse. 3. Give the nurse a 6 month period to be observed. 4. Avoid confronting the nurse so that the client's care will not be jeopardized.

2. Correct: This may be a situation in which the charge nurse must address the issue of an impaired nurse. All nurses should be aware of the signs and symptoms of substance abuse. The best way for the charge nurse to deal with these suspicions initially is to directly discuss the concerns with the nurse. Intervention may be needed immediately to protect the safety of the clients. If deemed appropriate, the charge nurse may encourage the nurse to seek help independently. 1. Incorrect: This action will not do anything to help an impaired nurse. In fact, this could potentially put the few clients being cared for by this nurse at risk of harmful actions, and it could create an unsafe workload on the other nurses who would be picking up additional clients that this nurse was no longer caring for. 3. Incorrect: Oh my! Would we really consider allowing this impaired nurse to continue to care for clients for 6 months without intervention while we "observe" the actions of the nurse? This would not be a very good idea. Keep in mind that impaired nurses can lose their usual ability to provide safe, competent client care. Although nurses may be working under the influence of a substance, they retain accountability for their actions and cannot use impairment as a legal defense if harm occurs to a client. So we should certainly not allow the nurse to continue working without investigation and/or intervention. 4. Incorrect: I know that not jeopardizing client care sounded nice, but avoiding confrontation with the nurse will not help to fix this problem. Although it can be very difficult to suspect a co-worker of being impaired or abusing substances, especially when fear of retaliation may be present, nurses should know that they have a responsibility to report any suspicion of such activity to nursing management.

A client is transported to the emergency department following a 20 foot fall from a ski lift. The nurse records initial assessment findings on the chart. Based on that data, what actions should the nurse implement immediately? Exhibit: Vitals: BP 90/40; HR 125; RR 30 and labored; + jugular venous distention (JVD) with subcutaneous emphysema noted to right shoulder area. Select all that apply 1. Apply occlusive dressing to chest. 2. Initiate large gauge IV line. 3. Prepare for chest tube placement. 4. Administer high flow oxygen. 5. Position client on right side.

2., 3. & 4. Correct: Based on the assessment data recorded by the nurse, the client most likely has a tension pneumothorax secondary to blunt force trauma from the fall. Immediate actions must focus on preventing tracheal deviation and a fatal outcome. The need for intravenous fluids and medications in any trauma requires at least one large bore IV line or more. This client will need immediate chest tube placement to relieve increasing intrathoracic pressure. While preparing the client for this procedure, high-flow oxygen should be administered via non rebreather mask because of the client's respiratory distress. 1. Incorrect: There is no indication in the question of an open chest wound, or that a dressing is needed. The occlusive chest dressing will be placed over the insertion site of the chest tube after placement is completed. 5. Incorrect: This trauma client will be secured to a backboard, most likely with a cervical collar in place, until x-rays confirm there has not been a cervical spine injury. Placing the client on the right side is counterproductive and in fact could further impair respiratory efforts.

A client with a history of myasthenia gravis (MG) has been discharged from the hospital following a thymectomy. When teaching the client how to prevent complications, the home care nurse emphasizes what daily actions are most important? Select all that apply 1. Include daily weight lifting exercises. 2. Practice stress reduction techniques. 3. Complete chores early in the day. 4. Take medications on time and prior to meals. 5. Eat three large meals daily.

2., 3. & 4. Correct: Myasthenia gravis is a chronic autoimmune disorder characterized by progressive muscle weakening and chronic fatigue. Clients become weaker throughout the day, contributing to the potential for complications. Stress reduction techniques are important since stress can contribute to a myasthenic crisis, a severe respiratory emergency. Daily tasks, including ADL's, should be completed early in the day when the client has the most energy. Medications for MG, including neostigmine and pyridostigmine, must be taken on time and prior to meals. 1. Incorrect: Clients with myasthenia gravis are instructed to include gentle daily exercise combined with periods of rest throughout the day. Weight lifting would be too strenuous and would quickly tire this client, possibly leading to a myasthenia crisis. 5. Incorrect: Because of the difficulty in chewing or swallowing, multiple small meals throughout the day are safer and more beneficial to a client with myasthenia gravis. Medications are timed in relation to meals, so consistent but smaller meals would be more beneficial for the client.

The staff nurse is caring for a 3-month old client receiving potassium IV therapy. Which actions indicate to the charge nurse that the staff nurse understands IV management? Select all that apply 1. Uses a 15 gtt factor drip chamber when changing the IV tubing. 2. Applies elbow restraints to prevent dislodgement of the IV catheter. 3. Checks the IV site for blood return hourly. 4. Instructs unlicensed assistive personnel (UAP) to count drip rate hourly. 5. Attaches a volume-controlled IV administration set to IV bag prior to beginning IV therapy.

2., 3. & 5. Correct: Young children and infants usually must be restrained to some degree to prevent accidental dislodging of the needle. Elbow restraints can prevent an infant with a scalp IV from rubbing or touching the IV site. When a foot, leg, or arm is used, limb motion must be limited. IV potassium is an irritant. When the fluid being infused is a known irritant or vesicant, the nurse should check the IV site for blood return and possible infiltration hourly. Infants and young children have a narrow range of normal fluid volume, and the risk for fluid overload is great, especially in an infant. Always use a volume-controlled IV administration set with an infant or small child. These sets hold no more than 100-150 mL of fluid, so the maximum amount that could accidentally be infused is limited. 1. Incorrect: Always use microdrip tubing which is a 60 gtt chamber. Micro Drip chambers are used for children and for clients who can not tolerate a fast infusion rate or large volumes. 4. Incorrect: This intervention is beyond the scope of a UAP. The UAP may assist with activities of daily living and bedside care under the supervision of a registered nurse or other healthcare professional. The nurse is responsible for monitoring the IV flow rate.

A client diagnosed with human immunodeficiency virus (HIV) is to be sent home today. The nurse has initiated discharge instructions on the proper handling of blood and body fluid at home. The nurse knows the teaching is successful when the client makes what statement? 1. "As long as it's my home, I can use normal cleaning methods." 2. "I must scrub with hot, soapy water and allow it to air dry." 3. "I should clean area with a 10% mixture of bleach and water." 4. "I must sterilize with isopropyl alcohol and rinse with ammonia."

3. CORRECT: The proper method to clean spills of blood or body fluids at home is to use a 10% solution of household bleach, which means 9 parts of water to 1 part bleach. It is recommended to leave the bleach solution on the contaminated area for 10 to 20 minutes, and then rinse with hot water. Any towels or cloths used to clean the area should be double bagged and discarded. 1. INCORRECT: This demonstrates a false sense of security. Even in a home environment, visitors and family could become contaminated with the HIV virus. Microscopic amounts of blood or body fluids could contaminate others, and therefore proper cleaning methods must be followed even at home. 2. INCORRECT: Hot, soapy water will not kill the HIV virus on hard surfaces, regardless of the type of soap or the temperature of the water. Additionally, air drying will not decrease the virulence of the virus. 4. INCORRECT: Isopropyl alcohol, or rubbing alcohol, does not inactivate the HIV virus, even if rinsed with ammonia. Also, mixing household chemical cleaners, such as bleach with ammonia, can create dangerous fumes that are toxic to humans.

The nurse has received the change-of-shift report. What client should the nurse assess first? 1. A client with fibromyalgia reporting generalized pain of 7 out of 10. 2. A client diagnosed with rheumatoid arthritis needing discharge teaching. 3. A client with a fractured right humerus who reports the cast is too tight. 4. A client with an above the knee amputation reporting phantom pain.

3. Correct: The clue that should be picked up on here is that the client is now reporting that the cast has become too tight. Compartment syndrome could be developing which can impede circulation and cause nerve damage. This situation requires an immediate neurovascular check to determine if intervention is needed to relieve the pressure and restore circulation. 1. Incorrect: The client with fibromyalgia is reporting a pain level that needs to be addressed and the client will likely require pain medications. However, this client would not need to be seen prior to the client with potential neurovascular compromise from a cast that is too tight. 2. Incorrect: The client who was diagnosed with rheumatoid arthritis will need discharge teaching and may be wanting to go home quickly, but this client would not take precedence over the client with the cast that has become too tight. You would be jeopardizing the limb of this client to take the time to do discharge teaching for the client waiting to go home. 4. Incorrect: Is phantom pain something that is unexpected with above the knee amputations? No! However, it remains true pain for this client and the client would need intervention to help manage this pain. This client would not be a priority to be seen before assessing the client with the cast that is too tight who may be developing compartment syndrome.

A school nurse is caring for a child who fell on the playground. Upon examination of the child, the nurse notes multiple bruises in various stages of healing. What is the nurse's initial intervention? 1. Ask the parents who hit the child on the back. 2. Notify the child's primary healthcare provider. 3. Contact the Department of Health and Human Services. 4. Document the findings and observe the child over the next week.

3. Correct: Unless there is a policy to direct otherwise, the nurse who suspects child abuse is obligated to report it to the Department of Health and Human Services (DHS). 1. Incorrect: This is confrontational and will warn the parents that you suspect abuse. This may lead to greater harm for the child. 2. Incorrect: Unless there is a policy to direct otherwise, the nurse who suspects child abuse is obligated to report it to the Department of Health and Human Services. DHS, rather than the primary healthcare provider can intervene to maintain the child's safety. 4. Incorrect: This is delaying care. If the child is being abused, not reporting it could lead to serious injury or even death.

A client with a history of deep vein thrombosis (DVTs) is being instructed on how to apply compression stockings prior to discharge. What statement alerts the nurse the client may be noncompliant when at home? 1. "I will follow the special diet in order to lose weight." 2. "I should walk a little every few hours after sitting." 3. "My husband can help remind me not to cross my legs." 4. "The stockings are too difficult to put on every morning."

4. CORRECT: Compression stockings are used to prevent the formation of blood clots, reduce the diameter of distended veins and decrease stasis. Usually these stockings are ordered to be applied upon rising in the morning and removed at night, depending on the disease process. The client's comment suggests the difficulty of putting the hose on may lead to not wearing the stockings consistently. 1. INCORRECT: This statement by the client indicates a positive attitude about the need to lose weight and the intention of following the prescribed diet. Obesity is one of several main factors that can lead to the development of DVTs. 2. INCORRECT: Prolonged sitting, or even lying down, can increase the incidence of blood clots or DVTs. If the client does a lot of sitting during the day, it is advisable to walk around every few hours to reduce stasis. The client is acknowledging the need to increase mobility regularly, which is an indication of compliance. 3. INCORRECT: Placing pressure directly on vessels by crossing the legs compresses both veins and arteries, thus increasing the potential for blood clots or dislodging an unknown clot. The client has acknowledged the need to keep legs uncrossed and the benefit of having family provide reminders.

The nurse is presenting a seminar to expectant teen parents regarding infant car seat safety. What statement from a teen parent indicates to the nurse that teaching was successful? 1. "It's okay to place the car seat up front as long as it faces backwards." 2. "The baby has to stay rear facing until at least 40 pounds or 40 inches." 3. "Regular seat belts can be used if the child does not like the booster seat." 4. "An infant must stay in the backseat, facing backward, till at least a year old."

4. CORRECT: The nurse is looking for a statement that indicates the teen parents understand the proper use of infant car seats. Although there are some variations from state to state, the National Safety Council advises that infants should be in a rear-facing car seat in the back seat of a vehicle until at least age one year. This comment indicates the parents understand the teaching clearly. 1. INCORRECT: An infant or child car seat can never be placed in the front seat at any time, regardless of what direction it may face. Further teaching is definitely indicated. 2. INCORRECT: A child of 40 pounds or forty inches is of pre-school age, usually around 3 to 4 years old. This is too old for a rear-facing car seat. The issue of height and weight is more useful when determining whether a child can safely move from a car seat to a booster seat. The parents did not understand the instruction. 3. INCORRECT: The choice of booster seat versus regular car seat belts is not based on whether the child likes, or is comfortable, in using either type of restraints. The most accepted guideline for child safety is that children under the age of 8 years old should be in either a child's car seat or booster seat. Further teaching is needed.

A medical-surgical LPN has been sent to a short-staffed pediatric unit. The charge nurse knows what client would be most appropriate for this LPN? 1. 3 month old child with nonorganic failure to thrive. 2. 14 year old with exacerbation of cystic fibrosis. 3. 5 year old newly admitted with epiglottitis. 4. 10 year old with type 1 diabetes mellitus.

4. Correct: A medical-surgical LPN would likely have seen and cared for diabetics on the floor, including checking fingerstick blood sugars and injecting insulin. A 10 year old school-age child would also be more cooperative, making it easier for the LPN to interact with that client. 1. Incorrect: This client is only 3 months old, which would require specialized skills to evaluate developmental needs. Additionally, nonorganic failure to thrive is a serious situation in which the infant is not getting appropriate nutrition. There could be economic factors, resulting in a lack of food or poor-quality breast milk. Parental beliefs or negligence could also contribute to the situation; therefore, an RN should be assigned to this infant. 2. Incorrect: Although this client is an adolescent, an exacerbation of cystic fibrosis would require careful and frequent respiratory assessments with possible chest physiotherapy. This client would be more appropriate for an RN. 3. Incorrect: A new admission is not appropriate for a nurse sent from the medical surgical unit to the pediatric unit, particularly an LPN, because of the need for initial and frequent assessments. Epiglottitis is a respiratory illness that also impacts the airway. This child should be assigned to an RN.

The charge nurse is orienting a new nurse to the pediatric unit. Which teaching related to assessment is appropriate? 1. One assessment should be done daily on each client by the charge nurse. 2. An assessment should be done daily on each client at the beginning of the shift. 3. Assessments of clients should be updated as the nurse provides care to clients. 4. Assessments of clients should be done at the beginning of the shift and updated as nursing care is provided.

4. Correct: Assessment is ongoing; however, for each shift a baseline assessment should be done so the nurse can verify or make judgment regarding other findings throughout the 24 hour day. It is best to get the baseline as soon as possible once the shift begins, and update or reevaluate during the shift. This option actually incorporates the other 3 options making it the correct option. 1. Incorrect: Must include ongoing updated assessments, not just one assessment. These can be done by the assigned RN, not the charge nurse. 2. Incorrect: Must include ongoing updated assessments, not just at beginning of shift. 3. Incorrect: Must include initial beginning of shift assessment.

A client with a rare disorder has been admitted to a teaching hospital. The primary healthcare provider includes this client in medical students' morning rounds without notifying the client. When the angry client reports this to the charge nurse, what response by the nurse would be most appropriate? 1. "Consent is implied because this is a teaching hospital." 2. "These students will provide excellent care for you." 3. "I will call your primary healthcare provider to report how upset you are." 4. "You can refuse to be part of the students' study."

4. Correct: Clients' rights (still referred to in a hospital setting as the" Patient Bill of Rights") is a written code of ethical behavior describing the relationship that exists between the client and any facility to which they are admitted, including mental health units and hospice care. These guidelines provide the client a specified level of expectations regarding, for example, access to care, confidentiality and personal dignity. Regardless of the circumstances of the disease or location of treatment, clients have the right to refuse care from any professional personnel, including medical and nursing students. 1. Incorrect: Implied consent is an inferred agreement in which medical interventions are provided when the client cannot formally agree, as in the case of unconsciousness or incompetence. However, this client is clearly conscious and able to choose whether care by students is acceptable. The fact that the facility is a teaching hospital in no way deprives this client of the right to refuse student involvement. 2. Incorrect: The issue is the client's rights were violated when medical students were allowed involvement in this case without express consent or acknowledgement by the client. This response by the nurse ignores the client's rights or feelings by focusing on student abilities to provide care. It is demeaning to the client and does not address the client's concerns or provide alternatives. 3. Incorrect: Alerting the primary healthcare provider will be one component needed to resolve this situation. However, this initial response by the nurse is inappropriate for two reasons; first, this process transfers care of the client away from the nurse. Secondly, it does not provide the client with specific information about rights or resolutions.

In which situation should the nurse consult the client's advanced directive? Select all that apply 1. Client scheduled for breast reconstruction after mastectomy. 2. Client with a T-5 spinal cord injury beginning rehabilitation therapy. 3. Client diagnosed with Guillain-Barre' who is receiving ventilator support. 4. Comatose client with end stage chronic obstructive pulmonary disease. 5. Client diagnosed with inoperative brain tumor who is confused.

4., & 5. Correct: The client must have lost decision-making capacity because of a condition that is not reversible or must be in a condition that is specified under state law, such as a terminal, persistent vegetative state, irreversible coma, or as specified in the Advanced Directive. Both of these clients are terminal. 1. Incorrect: A client scheduled for surgery after a mastectomy is still able to make decisions. The option does not say the client is terminal, in a vegetative state, or in a coma. 2. Incorrect: A client who has a spinal cord injury and is in rehabilitation is still alert and able to make decisions. 3. Incorrect: A client diagnosed with Guillain-Barre' is mentally competent and being on a ventilator does not indicate that the client has lost decision-making capacity.

During night time rounds, the nurse finds a client has cigarettes in bed and the room is filled with smoke. In what order should the nurse perform the following actions? Remove client from room. Notify hospital operator. Get the fire extinguisher. Close the client's door. Pull the fire alarm handle. Drag and Drop the items from one box to the other

Anytime an internal disaster is suspected, client safety is always the first concern. National Fire Safety codes refer to the mnemonic "R-A-C-E" (rescue -alarm-contain-extinguish). If the area is safe for the nurse to enter, removing the client from that environment would be the first action. Secondly the nurse must activate the EMS alarm system so that emergency personnel are en route. Additionally, the hospital must be alerted by contacting the hospital operator to activate appropriate internal alarm systems. Closing the client's door will help contain any fire or smoke. Finally, the nurse should obtain the closest fire extinguisher appropriate for the type of fire.

The nurse is caring for a client immediately following a bilateral salpingo-oophorectomy. Which position would be best for this client? 1. Fowler's 2. Modified Sims 3. Side-lying 4. Supine

3. Correct: We want to position for comfort with the knees flexed and on the side for airway. 1. Incorrect: Avoided to prevent pooling and edema in pelvis. 2. Incorrect: Partial lying on stomach is going to be painful. 4. Incorrect: Stretching out straight puts pressure on the abdomen and should be avoided.

What clients could safely be delegated to the LPN/LVN? Select all that apply 1. A client two days post appendectomy needing to ambulate. 2. A client with bronchitis receiving nebulizer treatments. 3. A newly diagnosed diabetic client awaiting discharge home. 4. A client newly admitted with exacerbation of myasthenia gravis. 5. A client admitted yesterday for observation following a fall. 6. A client with a nasogastric tube (NG) hooked to low suction.

1, 2, 5 & 6. Correct: These clients are appropriate and stable enough for the LPN/LVN's scope of practice. While an LPN/LVN cannot be assigned a fresh post-op, the first client had an appendectomy two days ago. The LPN/LVN could even delegate ambulating this client to unlicensed assistive personnel (UAP). A client with bronchitis will need a respiratory assessment by the RN at some point, but the LPN/LVN is definitely qualified to administer aerosol treatments. The third client was admitted for observation following a fall a day ago, indicating no injuries serious enough for a full admission. PNs can insert and monitor NG tubes. 3. Incorrect: This client is a newly diagnosed diabetic who will require extensive teaching about self care at home. Additionally, discharging a client always involves teaching, which cannot be initiated by an LPN/LVN. This option does not indicate that any teaching had been presented, so the client is not an appropriate assignment for the LPN/LVN. 4. Incorrect: Myasthenia Gravis is a progressive weakening of the neuromuscular system placing the greatest risk on the respiratory system. Although this client is on a medical-surgical floor, there is a need for close monitoring and frequent assessment of the respiratory system, requiring an RN.

When inspecting the equipment in a client's room, what would the nurse recognize as electrical safety hazard(s)? Select all that apply 1. Flickering overhead light 2. Ground-fault circuit interrupter electrical sockets 3. Hospital labeled UL power strip 4. Bent electrical bed cord 5. Cracked electrical socket

1, 4, & 5. CORRECT. Dim or flickering lights are indications that there is a possible electrical wiring problem. Use of a damaged electrical cord or socket increases the risk of an electrical fire, shock, or burn. 2. INCORRECT. Ground-fault circuit interrupter (GFCI) electrical sockets should be in place in hospital and healthcare facilities. A GFCI socket will immediately cut off power if it detects someone receives a shock, helping prevent serious injury. 3. INCORRECT. While power strips are not ideal in the hospital setting, power strips that have been inspected and tagged with a hospital label may be used when multiple electrical outlets are required. Additionally, only power strips with the Underwriters Laboratories (UL) seal should be used.

Which observation of denture care by the unlicensed assistive personnel (UAP) would require the nurse to intervene? Select all that apply 1. Soaking the dentures in hot water 2. Donning gloves and using a gauze pad to grasp and remove dentures 3. Moistening the dentures prior to inserting them 4. Wrapping the dentures in tissue while the client sleeps 5. Placing a washcloth in the bathroom sink prior to cleaning

1. & 4. Correct: Hot water may damage dentures so intervention is needed. Dentures should be stored in a denture cup. 2. Incorrect: Gloves should be worn to remove dentures and a gauze used to grasp the dentures. 3. Incorrect: Moistening the dentures will ease insertion. 5. Incorrect: The wash cloth is placed in the sink to prevent the dentures from breaking if they are dropped.

The nurse is reviewing sequential lab results on a newly admitted client with multiple health issues. Critical changes in which body system require the nurse to immediately notify the primary healthcare provider? Exhibit Metabolic Profile: Date: 1/1, Sodium: 143 mEq/L (143 mmol/L), Potassium: 4.2 mEq/L (4.2 mmol/L), Chloride: 100 mEq (100 mmol/L), Glucose: 99 mg/dL (5.7 mmol/L), Blood Urea Nitrogen: 16 mg/dL (5.7 mmol/L), Creatinine: 1.3 mg/dL (99 mmol/L), Venous Carbon Dioxide: 24 mEq/L (24 mmol/L) Date: 1/2, Sodium: 138 mEq/L (138 mmol/L), Potassium: 4.6 mEq/L (4.6 mmol/L), Chloride: 102 mEq (102 mmol/L), Glucose: 102 mg/dL (5.6 mmol/L), Blood Urea Nitrogen: 19 mg/dL (6.8 mmol/L), Creatinine: 1.6 mg/dL (122 mmol/L), Venous Carbon Dioxide: 26 mEq/L (26 mmol/L) Date: 1/3, Sodium: 137 mEq/L (137 mmol/L), Potassium: 5.0 mEq/L (5.0 mmol/L), Chloride: 104 mEq (104 mmol/L), Glucose: 104 mg/dL (5.8 mmol/L), Blood Urea Nitrogen: 22 mg/dL (7.9 mmol/L), Creatinine: 2.0 mg/dL (153 mmol/L), Venous Carbon Dioxide: 27 mEq/L (27 mmol/L) 1. Renal 2. Endocrine 3. Pulmonary 4. Cardiovascular

1. Correct: All lab values are fluctuating, but those most significantly outside of normal range are the BUN and Creatinine levels, reflecting possible renal failure. The nurse would need to immediately notify the primary healthcare provider of possible complications in the client's renal system. The sodium, potassium, and glucose are within normal limits. 2. Incorrect: Several lab readings could relate to the endocrine system, but most specifically are glucose and chloride. Both these electrolytes have fluctuating levels but remain well within normal limits. Therefore, the endocrine system is not the nurse's concern at this time. 3. Incorrect: The carbon dioxide levels listed reflect venous, NOT arterial, blood. Norms for venous carbon dioxide are 23 to 29 mEq/L (milliequivalent units per liter of blood), indicating these results are all within normal levels. Although chloride could also reflect the pulmonary system, there are no irregular results in chloride levels. 4. Incorrect: Many of these elements could affect the cardiovascular system, but most specifically sodium and potassium. At present, these levels are all within normal limits, although the potassium has risen to the uppermost levels of normal. If those levels continue to climb, this could become a concern; however, this would not require a call to the primary healthcare provider at this time.

Which action by the nurse administering intravenous ciprofloxacin would require intervention by the charge nurse? 1. Sets IV pump to administer ciprofloxacin over a period of 30 minutes. 2. Educates client that medication may cause dizziness. 3. Instructs client to notify nurse for any tendon pain. 4. Administers ciprofloxacin through 20 gauge catheter into the cephalic vein.

1. Correct: Cipro IV should be administered to by intravenous infusion over a period of 60 minutes. Slow infusion of a dilute solution into a larger vein will minimize client discomfort and reduce the risk of venous irritation. 2. Incorrect: This action does not require intervention by the charge nurse as dizziness is a side effect of this medication. 3. Incorrect: This is a correct action. Fluoroquinolones, including Cipro IV, are associated with an increased risk of tendinitis and tendon rupture in all ages. This adverse reaction most frequently involves the Achilles tendon, and rupture of the Achilles tendon may require surgical repair. Tendinitis and tendon rupture in the rotator cuff (the shoulder), the hand, the biceps, the thumb, and other tendon sites have also been reported. 4. Incorrect: Slow infusion of a dilute solution into a larger vein will minimize client discomfort and reduce the risk of venous irritation.

A client has been admitted with folic acid deficiency anemia. Which referral would most likely be appropriate for the nurse to make? 1. Alcoholic Anonymous 2. American Sickle Cell Anemia Association 3. Pernicious Anemia Society 4. Aplastic Anemia Support Group

1. Correct: Clients diagnoses with folic acid anemia typically have developed the anemia from chronic alcohol abuse. Alcohol consumption increases the use of folates, and the alcoholic diet is usually deficient in folic acid. This referral would be appropriate. 2. Incorrect: Sickle cell anemia is not caused by folic acid deficiency, so this client would not need a referral to this society. It is an inherited form of anemia, a condition in which there aren't enough healthy red blood cells to carry adequate oxygen throughout the body. Normally, red blood cells are flexible and round, moving easily through blood vessels. In sickle cell anemia, the red blood cells become rigid and sticky and are shaped like sickles or crescent moons. These irregularly shaped cells can get stuck in small blood vessels, which can slow or block blood flow and oxygen to parts of the body. 3. Incorrect: Pernicious anemia is a decrease in red blood cells that occurs when the intestines cannot properly absorb vitamin B12. The body needs vitamin B12 to make red blood cells. You get this vitamin from eating foods such as meat, poultry, shellfish, eggs, and dairy products. A special protein, called intrinsic factor (IF), binds vitamin B12 so that it can be absorbed in the intestines. This protein is released by cells in the stomach. When the stomach does not make enough intrinsic factor, the intestine cannot properly absorb vitamin B12. 4. Incorrect: Aplastic anemia is a blood disorder in which the body's bone marrow doesn't make enough new blood cells. This may result in several health problems including arrhythmias, an enlarged heart, heart failure, infections and bleeding. Aplastic anemia is a rare but serious condition. It can develop suddenly or slowly and tends to worsen with time, unless the cause is found and treated.

What room assignment by the charge nurse is most appropriate for a client who is being admitted with poor appetite, malaise, and temperature of 101.5ºF (38.6ºC)? 1. Private room. 2. Room with a client who has biliary colic. 3. Room with a client who is 3 days post operative hip replacement. 4. Room with a client who is in skeletal traction due to broken femur.

1. Correct: In this particular situation, a private room is best due to the elevated temperature. This could mean the client has an infection and is contagious. All of the often clients do not need to be exposed to this client with fever of unknown cause. 2. Incorrect: Does not need to be exposed to infection. Biliary colic is pain due to a gallstone blocking the bile duct. The client may need surgery and definitely should not be exposed to infection. 3. Incorrect: Post op client already at risk for infection. This is not the most appropriate client to room with the new admit. 4. Incorrect: Does not need to be exposed to infection. The client is already at risk for infection due to the skeletal traction. Complications of skeletal traction include risk for bone infection due to a screw being placed in a bone.

A client with type II diabetes reports normal blood glucose levels at bedtime and high blood glucose levels in the morning for the past week. What instruction would the nurse give the client? 1. Monitor blood sugar around 2am. 2. Decrease bedtime snacking. 3. Decrease intermediate acting insulin. 4. Increase intermediate acting insulin.

1. Correct: Morning hyperglycemia may be the result of dawn's phenomenon or the Somogyi effect. The client must take their blood sugar between two and three o'clock in the morning for several days to determine the cause of morning hyperglycemia. If the client has decreased blood sugar between two and three o'clock in the morning, suspect Somogyi effect. 2. Incorrect: This is an intervention; assessment should come first. The nurse must determine the cause of morning hyperglycemia in order to treat the condition appropriately. 3. Incorrect: This is an intervention; assessment should come first. The nurse must determine the cause of hyperglycemia in order to treat the condition appropriately. An appropriate intervention for a client with Somogyi effect would be to decrease the evening dose of intermediate acting insulin, however, the nurse must first determine that the client is in fact experiencing the Somogyi effect. 4. Incorrect: This is an intervention; assessment should come first. Increasing the intermediate acting insulin would not be appropriate action for a client experiencing Somogyi effect.

Which ethical principle is involved when a nurse reports a medication error to the primary healthcare provider? 1. Nonmaleficence 2. Beneficence 3. Justice 4. Fidelity

1. Correct: Nonmaleficence is best illustrated with the nurse's action, as the goal is to do no harm to the client. With timely reporting of an error, further complications may be prevented. 2. Incorrect: Beneficence refers to doing good. This may include compassion and kindness. 3. Incorrect: Justice refers to equitable distribution of resources. Triage in the ED is one action that illustrates justice. 4. Incorrect: Fidelity refers to truth-telling. If the client were to ask if a medication error was made, the nurse would answer yes to the question as a way of demonstrating fidelity.

The nurse is caring for a Puerto Rican client. The client has several injuries from a car accident and is experiencing pain. Which behavior is likely to be noted? 1. Loud crying with pain. 2. Enduring the pain in order to bring honor. 3. Quiet and stoic responses to pain. 4. Refusing pain medication because it is God's will.

1. Correct: Puerto Rican clients tend to cope with pain by loud and outspoken reports of pain. This is consistent with Puerto Rican culture and their response to pain. 2. Incorrect: Quietly enduring pain is consistent with the Japanese culture. This is consistent with the Asian culture and brings honor. 3. Incorrect: Stoic responses are consistent with Asian culture. The client is likely to be quiet about the pain thinking that complaints of pain will bring dishonor to the family. 4. Incorrect: Filipino clients tend to view pain as God's will. They may refuse medication to relieve the pain.

Which client should the nurse see first? 1. 53 year old client with chest pain scheduled for a stress test today 2. 62 year old client with mild shortness of breath and chronic obstructive pulmonary disease 3. 66 year old client with angina scheduled for a cardiac catheterization this AM 4. 78 year old client who had a left hemispheric stroke 4 days ago

1. Correct: The client may be experiencing a myocardial infarction and requires further assessment. Therefore, this client would not be a priority over a client who may be experiencing a MI. 2. Incorrect: Dyspnea is one of the three (chronic cough, sputum production, and dyspnea) primary symptoms characteristic of chronic obstructive pulmonary disease. 3. Incorrect: The client is scheduled for the procedure needed for further assessment of angina. This client would be considered more stable than the client who may be having a MI. 4. Incorrect: After a stroke has occurred, medical management is aimed at preventing a second stroke from occurring and rehabilitation. This client may have significant sequelae related to the stroke, but would not be considered acute nor a priority over the client possibly having a MI.

What is the best method for the nurse to verify correct nasogastric (NG) tube placement after insertion? 1. X-ray of the upper GI 2. Gastric aspiration and pH testing 3. Auscultation of air instilled into the stomach 4. Visualization of the tube markings

1. Correct: The gold standard for nasogastric feeding tube placement is radiographic confirmation with X-ray. This is the most reliable method! 2. Incorrect: Both respiratory and gastrointestinal aspirates may be similar in color and may be misinterpreted. 3. Incorrect: This method cannot differentiate tube placement in the stomach or lung. The practitioner may still hear a rush of air. 4. Incorrect: Visualization of tube markings does not provide a reliable verification that the tube is in the stomach. This has never been a reliable way of verifying placement.

The nurse is supervising the care of a client on bedrest with a skull fracture from head trauma. Which action, when performed by an unlicensed assistive personnel (UAP), should the nurse interrupt? 1. Assisting with turn, cough, and deep breathing (TCDB) 2. Elevating the head of the bed to 30 degrees. 3. Measuring urinary output every hour. 4. Turning off room lights.

1. Correct: The nurse should interrupt the UAP assisting with TCDB because this may increase intracranial pressure (ICP). TCDB increases intrathoracic pressure which then increases ICP. 2. Incorrect: Maintain client with head trauma in the head up position. This position promotes drainage from the head and decreases vascular congestion. 3. Incorrect: This is an acceptable action and one the UAP can do. 4. Incorrect: You want to decrease stimulation and turning off room lights will provide restful environment in an effort to decrease ICP.

A client receiving 50 mL/hr of enteral feedings has a gastric residual volume of 200 mL and is reporting nausea. What is the appropriate nursing intervention? 1. Stop the feeding and assess gastric residual volume in 1 hour. 2. Reduce the infusion rate to 25 mL/ hour and reevaluate residual volume in 4 hours. 3. Change the feeding schedule from continuous to intermittent delivery. 4. Discard the 200 mL and continue the feedings at the same rate.

1. Correct: The safest response is to STOP the feedings and re-assess in 1 hour. Nausea may be a sign of intolerance. Continuing the feeding may also result in vomiting with possible aspiration. 2. Incorrect: Reducing the rate requires a primary healthcare provider's prescription and does not fix the problem. In this answer, the NCLEX people are giving you a scope of practice question. If you select this answer, you are telling the people who write the test that you are going to write prescriptions for your clients. 3. Incorrect: Changing the feeding schedule requires a primary healthcare provider's prescription and does not fix the problem. Again, with this answer, the NCLEX people are giving you a scope of practice answer. If you choose this answer, you are telling the people at NCLEX that you are going to write prescriptions for your clients. 4. Incorrect: Do not discard residual volumes. Discarding residual volumes can disrupt a client's fluid and electrolyte balance. Standard practice is to give it back. Discarding the residual requires a prescription. Continuing at the same rate is not safe when you have high residuals. The feedings should be stopped.

The nurse is discharging the client after removing sutures from an abdominal wound. Which instructions should the nurse give the client at the time of discharge to reduce the risk of complications? 1. inspect the wound daily for any changes 2. Resume normal activities when you go home. 3. Keep the incision covered at all times. 4. Follow up with primary healthcare provider when scheduled.

1. Correct: The wound should be inspected daily for any signs of infection once the client goes home. Healing has only just begun by discharge. Signs of wound infection include: Increased pain, swelling, redness, or warmth around the affected area; Red streaks extending from the affected area; Drainage of pus from the area; Fever. 2. Incorrect: The client may be restricted in some activities, such as lifting, that would place undue strain on the suture line. 3. Incorrect: It is likely that the incision can be uncovered, but the primary healthcare provider prescription would apply here. Look for words like "all" which generally make the option wrong. Things are not that definite. 4. Incorrect: This is true; however, the signs and symptoms of infection should be given to the client. If signs/symptoms develop, the primary healthcare provider should be notified prior to the next appointment.

The nurse is caring for a burn victim with a skin graft to the hand. The area is pale and mottled but has good capillary refill. What is the nurse's best action at this time? 1. Warm the room. 2. Submerge the hand in warm water. 3. Order a K pad and apply to hand. 4. Have the client exercise the fingers to increase blood flow.

1. Correct: When caring for clients with skin grafts, we want good circulation, so warm that room up.2. Incorrect: This will not improve circulation and can lead to infection. 3. Incorrect: This will not improve circulation. Someone who has a skin graft doesn't have good sensation so there is risk of another burn to the graft with this. 4. Incorrect: Working those stiff, cold fingers will further imbalance the oxygen supply. This will not help, particularly if the environment remains cool.

A nurse on the unit has had a disagreement with the family of a client regarding the client's dressing change. What is the best action by the nurse manager? 1. Meet with the family member and the RN to discuss the disagreement. 2. Assure the family member that the nurse followed the hospital procedure. 3. Discuss the dressing change procedure with the RN and compare to a current textbook. 4. Report the argument to the hospital administrator.

1. Correct: When conflict occurs, meet with both parties together to discuss the problem. Each party can hear what the other is saying and the nurse manager is not caught in the middle. They will be able to come up with solutions together or the manager can mediate. 2. Incorrect: It is ok to clarify that the nurse followed hospital procedure. However, the nurse is sing the nontherapeutic communication technique of blocking. The family member may still believe that there is another procedure that could have been initiated. 3. Incorrect: You may want to do this as well, but it will not address the conflict. The conflict is that the family member disagrees with the nurse's procedure for dressing change. 4. Incorrect: The nurse manager must try to resolve the conflict between the family member and the nurse first. If the conflict cannot be resolved the nurse manager would notify the person that is next in the chain of command.

The nurse has been teaching the parents of a child taking methylphenidate for the treatment of attention deficit hyperactivity disorder (ADHD). Which comments by the parents indicate adequate understanding of the important considerations for methylphenidate? Select all that apply 1. "I know that I need to monitor weight." 2. "I am supposed to call if my child has decreased attentiveness." 3. "This medication may cause increased drowsiness." 4. "I know that I need to monitor my childs height." 5. "If my child can't sleep, the dosage may need to be increased."

1., 2. & 4. Correct: Continued use of the medication may cause delays in growth and loss of appetite. Lack of appetite may cause weight loss. This drug may affect child's growth rate. The child's attentiveness should increase with this medication and if there is no improvement in attentiveness with this medication then notify the primary healthcare provider. 3. Incorrect: The medication is more likely to cause insomnia especially if administered late in the day. If this medication can cause insomnia. 5. Incorrect: If the client cannot sleep, it is likely that the afternoon dose will be decreased or omitted.

A palliative care client is suffering from persistent diarrhea. What foods should the nurse suggest? Select all that apply 1. Applesauce 2. Rice 3. Bananas 4. Tea 5. Yogurt

1., 2., & 3. Correct: The BRAT diet is recommended for clients with persistent diarrhea. This diet consists of bananas, rice, applesauce, and toast. Rice and potatoes help to reduce diarrhea. Bananas will help replace potassium. Once the diarrhea subsides, the client can add easily digestible foods like eggs. 4. Incorrect: Avoid coffee and tea because caffeine containing beverages may have a laxative effect. Caffeine is a stimulant and will increase the peristalsis even more. 5. Incorrect: Dairy products may make the diarrhea worse. Avoid these until the diarrhea subsides.

Which assessment findings would the nurse expect to see in a client diagnosed with idiopathic thrombocytopenic purpura (ITP)? Select all that apply 1. Ecchymosis 2. Bleeding gums 3. Palpable spleen 4. Pain 5. Petechiae

1., 2., 3., & 5. Correct: The word thrombocytopenia means low platelets. Any client with low platelets is at risk for bleeding, which is indicated by ecchymosis (bruising), bleeding gums, and petechiae (red to purple dots on the skin, 1-3 mm in size). Spleen and liver are often slightly palpable. 4. Incorrect: Pain is not associated with ITP unless there are other associated problems. However, the stem of the question gave no indication that other problems exist.

A client who has been taking phenytoin for several years arrives to the clinic for follow-up care. During the nurse's history and physical of the client, which findings indicate a possible side effect to the phenytoin? Select all that apply 1. Skin rash 2. Reports fatigue 3. Dyspnea on exertion 4. Pale conjunctiva 5. Heart rate 60/min

1., 2., 3.,& 4. Correct: An adverse effect of phenytoin is aplastic anemia. Phenytoin is an anticonvulsant. Aplastic anemia is a blood disorder where not enough new blood cells are produced in the bone marrow. The blood cells include red blood cells, white blood cells and platelets. The most common symptom of decreased RBC's is fatigue and dyspnea upon exertion because RBC's are responsible for oxygen transport throughout the body. A common sign/symptom of aplastic anemia is also skin rashes. Collectively, these are signs/symptoms of aplastic anemia caused by this medication. 5. Incorrect: This is a normal heart rate, and there is no concern for vital signs within normal limits.

The nurse is planning to teach a group of senior citizens about modifiable risk factors for developing a stroke. Which factors should the nurse include? Select all that apply 1. Diabetes mellitus 2. Hypertension 3. Hispanic ethnicity 4. Atrial fibrillation 5. Sleep apnea 6. Smoking

1., 2., 4., 5., & 6. Correct: These are all modifiable risk factors that can be managed through lifestyle changes or medical treatment. 3. Incorrect: Hispanics, African Americans, Native Americans, and Asian Americans have a higher incidence of strokes than whites. You cannot change your race or ethnicity so this is a non-modifiable risk factor for stroke.

A nurse works in the operating room (OR) as a circulator. Which actions should the nurse perform to help prevent surgical-site infections? Select all that apply 1. Keep the OR doors closed during a surgical case. 2. Minimize traffic in the OR. 3. Ensure the room has negative air flow. 4. Monitor the sterile field at all times. 5. Immediately discard any object that becomes contaminated.

1., 2., 4. & 5. Correct: Positive pressure can be used in rooms adjacent to a negative pressure room. The purpose of positive pressure is to ensure that airborne pathogens do not contaminate the patient or supplies in that room. Positive pressure could be used in an operating room to protect the patient and sterile medical and surgical supplies. Positively pressurized rooms are typically considered the cleanest rooms in the hospital. Keeping the doors open impedes the air exchange system in the OR. The air exchange system is designed to decrease airborne contaminants in the OR. Limiting the traffic in the room decreases the amount of bacterial shedding, minimizes harmful air turbulence, and prevents accidental contamination of the sterile field. Contaminated objects break sterile field. 3. Incorrect: Negative air flow has not been proven to prevent surgical-site infections. Negative air flow refers to an airborne infection isolation room. A negative pressure room in a hospital is used to contain airborne contaminants within the room. Harmful airborne pathogens including bacteria, viruses, fungi, yeasts, molds, pollens, gases, VOC's (volatile organic compounds), small particles and chemicals are part of a larger list of airborne pathogens you can find in a hospital. A negative pressure isolation room is commonly used for clients with airborne infections. For example, a client with active tuberculosis, a disease caused by the bacteria Mycobacterium tuberculosis, will be placed in a negatively pressurized room because the tuberculosis bacterium is spread in the air from one person to another. When the client with active tuberculosis sneezes or coughs, other people may become infected when they inhale. However, by using a negative pressure room you can better contain the bacterium within the room.

A client diagnosed with terminal cancer wants information about an Advanced Directive for end-of-life care. What information should the nurse include? Select all that apply 1. An Advance Directive includes a Living Will and a Medical Power of Attorney. 2. A person can be designated to make medical decision in the event the client cannot. 3. The spouse can rescind the Advance Directive if the client becomes unresponsive. 4. Anyone over age 18 can have an Advanced directive. 5. The client can indicate desire for Do Not Resuscitate (DNR).

1., 2., 4., & 5. Correct: Advance directives do consist of two types of legal documents: Power of Attorney and a Living Will. A Medical Power of Attorney is a type of Advance Directive that appoints a health care agent to make decisions on the client's behalf when the client is unable to do so. An adult (18 years or older) can create an advanced directive. A person can indicate they wish to be a DNR client if their heart stops beating or they stop breathing. 3. Incorrect: An Advance Directive is a written, legal document regarding preferences for medical care should a person become unable to make medical decisions. Family cannot withdraw the Advance Directive and make decisions that go against the client's wishes made within the document.

Which client would be appropriate for the RN to assign to the LPN? Select all that apply 1. Client with cast to right leg requiring pain medication. 2. Client with chronic emphysema experiencing mild shortness of breath. 3. Client one day post kidney transplant. 4. Client two days post percutaneous endoscopic gastrostomy (PEG) placement. 5. Client prescribed antibiotics for cystitis.

1., 2., 4., & 5. Correct: The client who has a cast and requires pain medication is a stable and predictable client. This client needs ongoing monitoring which is within the scope of practice for the LPN. The client with chronic emphysema has expected shortness of breath. This client is stable and predictable. The client post PEG placement is stable. The LPN can monitor the wound and provide care to the PEG insertion site. The client with cystitis is stable and has a predictable outcome. It is within the LPNs scope of practice to administer antibiotics. 3. Incorrect: This client does not have a predictable outcome. There is a possibility of rejection, which means close assessments and evaluations are needed by the RN. This client will also need a lot of education regarding anti-rejection medications.

Which tasks would be appropriate for the nurse to delegate to an unlicensed assistive personnel (UAP)? Select all that apply 1. Prepare a client's room for return from surgery. 2. Observe for pain relief in a client after receiving acetaminophen with codeine. 3. Assist a client with perineal care after having diarrhea. 4. Clean nares around a client's nasogastric (NG) tube. 5. Pour a can of tube feeding into a client's percutaneous endoscopic gastrostomy (PEG).

1., 3, & 4. Correct. These are appropriate tasks for an UAP to complete. The UAP can provide hygiene needs to a client such as perineal care and cleaning of the nares. Also, making a surgical bed for the client returning from surgery is a basic procedure. 2. Incorrect. The UAP cannot assess or evaluate or even monitor the effectiveness of pain medication. That is what you are asking the UAP to do here. The client has received a narcotic and you have asked the UAP to evaluate the effectiveness of the medication. 5. Incorrect. Administering tube feeding into a PEG tube is beyond the scope of practice for the UAP. This is a procedure which requires a licensed personnel. Catheter placement must be confirmed, client identity checked, tube site flushed with water or sterile water and flow rate determined.

A nurse is planning a teaching session for a group of clients diagnosed with irritable bowel syndrome. What points should the nurse include to help the clients control symptom flare-ups? Select all that apply 1. If you are constipated, try to make sure you have breakfast. 2. Avoid low fat foods. 3. If you think a certain food is a problem, try cutting it out of your diet for about 12 weeks. 4. Drinks containing caffeine are likely to contribute to symptoms. 5. Foods such as broccoli and cabbage are good sources of fiber.

1., 3. & 4. Correct: If you are constipated, try to make sure you eat breakfast, as this is the meal that is most likely to stimulate the colon and give you a bowel movement. If you think a certain food is a problem, try cutting it out of your diet for about 12 weeks. (If you suspect more than one, cut out one at a time so you know which one causes you problems.) If there's no change, go back to eating it. The foods most likely to cause problems are: Insoluble (cereal) fiber; Coffee/caffeine; Chocolate; Nuts. 2. Incorrect: Avoid meals that over-stimulate the gut, like large meals or high fat foods. 5. Incorrect: Broccoli and cabbage are common gas-producing foods that can cause abdominal distention and flatulence.

Which discharge instruction should the nurse implement for a client diagnosed with insomnia? Select all that apply 1. Eliminate chocolate in the evening. 2. Drink a glass of red wine 1 hour prior to bedtime. 3. Perform progressive relaxation techniques at bedtime. 4. Take acetaminophen/diphenhydramine 2 tablets at bedtime. 5. Leisurely walk 3 hours prior to bedtime. 6. Increase the airflow on the continuous positive airway pressure (CPAP) machine.

1., 3., & 5. Correct: Consuming chocolate in the evening may cause insomnia. Chocolate contains caffeine and xanthines which are stimulates. The chemicals will suppress melatonin and increase the time to fall asleep. Progressive relaxation techniques are recommended to reduce insomnia. This exercise is a systematic relaxation and tensing of the muscle groups of the body. Insomnia is reduced by increasing muscle relaxation and decreasing the stress level of the client. Non Strenuous exercises such as a leisure walk performed within 3 hours of bedtime promotes the reduction of the client's stress level. 2. Incorrect: Consuming alcohol prior to bedtime is not recommended. Alcohol consumption increases the start of sleep but reduces rapid eye movement (REM) sleep. The side effect of the alcohol may also cause the client to awaken during night and have difficulty returning to sleep. 4. Incorrect: Diphenhydramine is not recommended for insomnia. The action of the diphenhydramine may cause the client to feel drowsy but provides only temporary increase in quantity of sleep. The hypnotic effect of diphenhydramine will cause client to experience decrease energy levels the next morning. 6. Incorrect: A CPAP is prescribed for a client with obstructive sleep apnea not insomnia. The CPAP machine delivers a constant air pressure to the lungs. The constant air flow will keep the airway open during sleep.

The nurse is preparing a client for transport to the radiology department for a left lung tissue biopsy. Which actions should the nurse make certain have been completed? 1. The consent form is signed. 2. The operative site is prepped with a razor. 3. The most recent lab work is on the chart. 4. Any preoperative medication is given as prescribed. 5. Person performing the procedure has marked the site.

1., 3., 4. & 5. Correct: The nurse should ensure that the consent form is signed, the lab work is in order, and any prescribed preoperative medication is given. The operative site is marked by the person who is ultimately accountable for the procedure and will be present when the procedure is performed. 2. Incorrect: The site should be prepped with clippers as opposed to a razor, which can cause injury to the client. The goal of preoperative skin preparation is to decrease bacteria without injuring the skin.

Which nursing action represents measures taken to protect the client from a mode of infection transmission in the chain of infection? Select all that apply 1. Donning personal protection equipment. 2. Administering the Haemophilus influenzae type B (HIB) immunization to a child. 3. Disposing of soiled gloves in the appropriate receptacle. 4. Wearing gloves when coming into contact with client's secretions. 5. Teaching importance of long pants and sleeves and insect repellent to reduce the risk of West Nile Virus. 6. Performing hand hygiene after removal of soiled gloves.

1., 3., 4., & 6. Correct: In this question, can you identify the nursing actions that represent prevention of the spread of infection to other clients at the point of mode of transmission on the chain of infection? The first one identified is the donning of personal protection equipment. This prevents the infectious agent from coming into contact with the nurse's hands that could then spread the infection to other clients. Next, disposing of soiled gloves in the appropriate receptacle assures that the infectious agents are not carried outside of the infectious client's room and then transmitted to other clients. Gloves should always be worn when there is a possibility that the nurse could come into contact with the client's secretions. Hand hygiene is a crucial part of infection control. Hand hygiene by washing the hands and/or using alcohol based sanitizer before and after glove removal reduces the risk of the spread of infection. Both the use of gloves when secretions are present and proper hand hygiene help to prevent the nurse's hands from becoming a mode of infection transmission to other clients. 2. Incorrect: Immunization of a child against Haemophilus influenzae type B (HIB) is an example of a nursing action to prevent infection transmission by disrupting the susceptible host link in the chain of infection. This is accomplished by increasing the resistance of the host to the infectious agent, which in this case is HIB. 5. Incorrect: Teaching clients about the importance of wearing long pants and long sleeves, as well as using insect repellent is an example of nursing action aimed at reducing the risk of West Nile Virus by breaking the chain of infection at the portal of entry link. By wearing the protective clothing and using insect repellent, the vector (mode of transmission) is not as likely to be able to access a portal of entry on the host.

Which task would be appropriate for the nurse to assign to an unlicensed assistive personnel (UAP)? Select all that apply 1. Assist client to brush and floss teeth. 2. Administer sodium polystyrene sulfonate enema. 3. Evaluate pain relief after narcotic administration. 4. Measure urine output when client voids. 5. Gather supplies to prepare room for isolation. 6. Monitor client for pain while assisting with ambulation.

1., 4., & 5. Correct: Assisting clients with activities of daily living are within the UAPs scope of practice. So, the UAP can assist a client to brush and floss teeth. UAPs can assist with elimination and are taught how to measure output. This would be an acceptable task to assign to the UAP. Gathering needed equipment and supplies is within the scope of duties for the UAP. 2. Incorrect: It is out of the UAP's scope of practice to administer medication. This includes medication enemas. Only a plain enema or soap enema can be given by the UAP. 3. Incorrect: The nurse is responsible for evaluating a client. This would be out of the UAP's scope of practice. The nurse cannot assign assessment and evaluation of the nursing process to the UAP. 6. Incorrect: The nurse is responsible for monitoring a client. This would be out of the UAP's scope of practice. The UAP can ambulate the client and can report to the nurse if the client states that pain is occurring but cannot monitor or collect data.

A client is admitted with a hip fracture after falling. Based on these lab values, what is the nurse's priority nursing intervention? Exhibit: Lab Values: Na+ 147 mEq/L (147 mmol/L) Specific gravity 1.030 Hct 55% 1. Provide foods high in iron 2. Increase fluid intake 3. Obtain a urine for culture 4. Measure intake and output

2. Correct: We already know that the question is about what life threatening complication? A pulmonary embolism. And these lab values say that the client is what? Dehydrated! So the only thing that is going to fix that is....... Increasing fluids. 1. Incorrect: This will not prevent pulmonary embolism. The problem is dehydration. Do something to fix the problem. Foods high in iron will not fix the problem. 3. Incorrect: This will not prevent pulmonary embolism. How will obtaining a urine sample for culture fix dehydration? It won't. This client needs to increase fluid intake. 4. Incorrect: We do want to monitor intake and output to see how the client is doing, however, this will not fix the problem. Hydrating the client will help the problem.

The following clients arrive to the emergency department (ED) at the same time. The triage nurse gives priority to which client? 1. A client with a possible fracture of the tibia 45 minutes ago. 2. A client with left hemiparesis and aphasia beginning 1 hour ago. 3. A client smelling of alcohol and reporting of severe abdominal pain. 4. A client involved in a motor vehicle accident (MVA) with a possible fractured pelvis.

2. Correct: The client who is started experiencing hemiparesis and aphasia 1 hour ago is likely having a stroke. The window for treatment with fibrinolytics is 3 hours, thus taking priority over the other clients. Time is brain! 1. Incorrect: This client has a possible fracture of the tibia. This is not a large bone, which would be at risk for hemorrhage. Splinting and ice packs could be used until after seeing the client having a stroke. 3. Incorrect: With this client, you would worry about pancreatitis. This client needs to be seen soon but not prior to the client having a stroke. 4. Incorrect: The MVA client could have bleeding from a fractured pelvis. This client is high on the admit list, but after the client having a stroke.

The nurse has been assigned four clients. Who should the nurse see first? 1. A client with diabetes admitted for debridement of a foot ulcer. 2. A client with epilepsy reporting an odd smell in the room. 3. A client with exacerbation of COPD reporting dyspnea. 4. An adolescent client post appendectomy reporting pain.

2. CORRECT: The client is potentially experiencing symptoms of an impending seizure, which can include seeing halos around lights or detecting odd smells. The nurse should immediately assess this client, implement seizure precautions and remain with client for safety. 1. INCORRECT: Although the vascular status of the foot will need to be assessed, there is no indication if the debridement has been completed yet. This client is not the nurse's first priority. 3. INCORRECT: Clients with COPD are always short of breath and dyspnea is an expected finding during an exacerbation. The client will need to be assessed, but there is no specific indication the respiratory status is presently compromised. 4. INCORRECT: There is no information regarding how recent was the surgery or the degree of pain being experienced. Post-surgical pain is expected and without further parameters, no determination can be made regarding this client. The nurse has another priority.

A newborn is admitted to the nursery with a diagnosis of rule out cytomegalovirus (CMV). Which of the following RNs should not be assigned to this baby? 1. A nurse just back from maternity leave. 2. A nurse who is 10 weeks pregnant. 3. A nurse who is breastfeeding her 4 month old. 4. A nurse who is on hormone replacement therapy.

2. Correct: Cytomegalovirus is a viral infection that can be devastating to a fetus, especially in the first trimester. Assigning this nurse to the newborn with CMV would put her unborn baby at high risk for life-long defects and even death. 1. Incorrect: Most adults have already been exposed to the virus and are not at risk for adverse effects of the infection. Even though this nurse just had a baby, there is no risk of her transmitting this virus to her child. 3. Incorrect: Most adults have already been exposed to the virus and are not at risk for adverse effects of the infection. Even though this nurse just had a baby, there is no risk of her transmitting this virus to her child. 4. Incorrect: Most adults have already been exposed to the virus and are not at risk for adverse effects of the infection. Hormone replacement does not affect the immune system and, therefore, this nurse is not at risk for infection from CMV exposure.

Which meal option should the client diagnosed with gout select? 1. Tuna salad on bed of lettuce, apple slices, coffee 2. Vegetable soup, whole wheat toast, skim milk 3. Roast beef with gravy sandwich, baked chips, diet coke 4. Spinach salad with chickpeas and asparagus, apple, tea

2. Correct: Gout is pain and inflammation that occurs when too much uric acid crystallizes and deposits in the joints. This is a good choose as it is low in purine and fat. Purines are broken down into uric acid. A diet rich in purines can raise uric acid levels. Meat and seafood increase the risk of gout. Dairy products may lower risk for gout. 1. Incorrect: The client should not eat tuna, which is high in purine. 3. Incorrect: Gravy is a high purine food and should be avoided. Also avoid artificial sweeteners. 4. Incorrect: Although spinach, and asparagus can be consumed in moderation, they still contain purines, so it is not as good of a choice as the vegetable soup, toast and skim milk.

The nurse cares for a client who is scheduled for an upper GI series. The nurse teaches the client about the test. Which statement by the client indicates an understanding of the nurse's teaching? 1. I'll have to take a strong laxative the morning of the test. 2. I'll have to drink contrast while x-rays are taken. 3. I'll have a CT scan after I'm injected with a radiopaque contrast dye. 4. I'll have an instrument passed through my mouth to my stomach.

2. Correct: In an upper GI series (sometimes called a barium swallow test), the client swallows barium contrast while x-rays are taken. 1. Incorrect: Laxatives are taken the night before a colonoscopy to ensure stool is cleared from the colon. Waiting to take the laxative the morning of the test would be ineffective and uncomfortable for the client. 3. Incorrect: Radiopaque dye injected before a CT (computed tomography) scan is not part of a GI series. This would be a totally different diagnostic test from the upper GI. 4. Incorrect: In a gastroscopy (sometimes called a gastric endoscopy), a scope is passed through the mouth to the stomach to visualize the inner lining of the upper GI tract.

The charge nurse is observing a nurse perform a dressing change on a client with a Stage III pressure ulcer. What observation by the charge nurse would indicate a need for further teaching? 1. Irrigates the pressure ulcer with 50 mL normal saline. 2. Irrigates the pressure ulcer with half-strength hydrogen peroxide. 3. Packs the wound with sterile gauze soaked in normal saline. 4. Applies a hydrocolloid dressing over the wound after cleansing.

2. Correct: Pressure ulcers should not be cleaned with substances that are cytotoxic such as hydrogen peroxide, betadine or Dakin's solution. Cytotoxic means toxic to cells, or cell-killing. Any agent or process that kills cells. These solutions can kill or damage cells, especially fibroblasts. Dakin's solution is a type of hydrochlorite solution. It is made from bleach that has been diluted and treated to decrease irritation. Chlorine is the active ingredient in Dakin's solution. 1. Incorrect: Normal saline is an appropriate solution and is used to clean pressure ulcers. This does not kill or damage cells. 3. Incorrect: Normal saline is an appropriate solution and pressure ulcers may be packed with sterile gauze. This helps remove necrotic tissue. 4. Incorrect: The wound should be covered with an appropriate dressing after cleaning. Hydrocolloid dressings support healing in clean granulating wounds and autolytically debride necrotic wounds. Hydrocolloid dressings are occlusive, so they provide a moist healing environment, autolytic debridement, and insulation.

A client in the third trimester of pregnancy arrives at the emergency room reporting general illness. The client is noted to have a blood glucose level of 390 mg/dL and is diagnosed with gestational diabetes. The primary healthcare provider prescribes 30 units of NPH insulin subcutaneously stat. What is the nurse's priority action? 1. Administer the dose of insulin immediately. 2. Question the type of insulin prescribed. 3. Insert an IV for an insulin infusion. 4. Question the dose of the insulin.

2. Correct: The client's blood glucose is extremely high and needs to be quickly reduced. The prescription given by the primary healthcare provider is for 30 units of NPH insulin, an intermediate acting insulin whose onset is about 1 ½ hours. That is too long to wait to start reducing this elevated glucose. This client should have been prescribed regular insulin. 1. Incorrect: While this client should indeed receive insulin immediately to start reducing the blood glucose, there is a problem with the prescription that the nurse must address before implementing. 3. Incorrect: Even though the primary healthcare provider has not prescribed an insulin drip, an IV would be an important intervention with a pregnant female whose blood sugar is very high. However, there is another problem that takes priority first. 4. Incorrect: Thirty units of insulin is not an unreasonable dose for a blood glucose level of 390 mg/dL. However, the amount of insulin is not the problem here. There is another issue of greater concern for the nurse.

The nurse manager is performing a chart audit for clients who were restrained. For which client would the side rails in the up position be considered a restraint? 1. The client who requests that the rails be placed in the up position. 2. The client who is confused and wanders about the unit. 3. The client who is ambulatory and places the side rails up without staff assistance. 4. The client who asks the family to place all the rails up before leaving.

2. Correct: The intent of the side rails in the up position is to limit movement; therefore, they are considered a restraint. The nurse cannot restrain or limit a client's movement without a primary healthcare provider prescription. 1. Incorrect: The client may request that side rails be raised at any time. 3. Incorrect: The ambulatory client can put his/her own side rails up if that increases feelings of security. 4. Incorrect: The family may place the rails up at the request of the client. That action would not be considered a restraint.

The nurse is caring for a client with a fibula fracture. The primary healthcare provider makes rounds and writes prescriptions. What is the nurse's best action? Exhibit MSO4 8 mg IM now Advance diet as tolerated Hgb and Hct in AM 1. Check the prescription prior to sending it to the pharmacy. 2. Clarify the prescription with the primary healthcare provider. 3. Notify the pharmacy that the prescription is needed immediately. 4. Gather the supplies needed for an injection.

2. Correct: The nurse should notify the primary healthcare provider, because MSO4 is an unapproved abbreviation that presents safety concerns. MSO4 is the abbreviation for morphine sulfate. MgSO4​ ​is the abbreviation for magnesium sulfate. Notifying the primary healthcare provider to clarify the prescription will prevent a medication error. 1. Incorrect: The prescription should not be sent to the pharmacy until after it is clarified with the primary healthcare provider. The Institute for Safe Medication Practices (ISMP) and The Joint Commission (TJC) recommend using the complete names for morphine and magnesium to eliminate confusion. 3. Incorrect: MSO4 is not an approved abbreviation. Before notifying the pharmacy, make sure you know what the prescription is for. The complete drug name should be written out. 4. Incorrect: You might be making a medication error if you assume you know what you are giving. Always seek clarification when in doubt.

The nurse enters a client's room to administer morning medications and notes that the client is praying aloud. What would be the nurse's best action? 1. Interrupt the client to administer the medications. 2. Wait quietly until the prayer is finished. 3. Join the client for the prayer. 4. Ask the client if you can provide a directed prayer.

2. Correct: This is the best action by the nurse as this is a private spiritual moment for the client. Prayer is a self-care strategy that provides comfort, increases hope, and promotes healing and psychological well-being. The nurse could either leave and return later or wait quietly for the client to finish. 1. Incorrect: Administering the medications can wait until the client finishes the prayer. 3. Incorrect: Do not assume that the client wants others to join in the prayer. This is a private moment for the client. 4. Incorrect: Do not assume that the client wants others to join in the prayer. Don't interrupt the client while praying.

A teenage client with asthma reports becoming very anxious and fearful each time an asthma attack occurs. What would be the nurse's best response to the client? 1. "I understand that you feel anxious. But you must stop this behavior." 2. "The feelings that you described can occur in individuals with asthma. You may find that learning relaxation exercises may help." 3. "I am concerned that feeling anxious during an asthma attack means you need more education about asthma." 4. "Everyone with asthma experiences tough times with their symptoms. You are learning to manage your asthma."

2. Correct: This statement acknowledges the client's feelings and then provides a suggested strategy that has been found to be useful in clients with anxiety and fear associated with asthma. 1. Incorrect: The nurse states understanding but then tells the client to stop the behavior without providing any helpful suggestions. 3. Incorrect: This response is disagreeing with the client's feelings and psychosocial response by stating that more education about asthma will prevent anxiety during an asthma attack. 4. Incorrect: This response dismisses and belittles the client's feelings and psychosocial response associated with asthma. By stating "everyone with asthma", the nurse is making a stereotypical response. This does not promote expressions of feelings by the client.

A client has just had a bone marrow biopsy. What is the nurse's priority intervention post procedure? 1. Apply ice pack to needle site. 2. Hold pressure on needle site for at least 5 minutes. 3. Observe needle insertion site every 2 hours. 4. Advise client to avoid activities that may result in trauma to the site for 48 hours.

2. Correct: When you see the word priority, you need to think: "What is the MOST important thing I can do for my client?" If you can only pick one answer, pick the life threatening answer or the answer that will decrease the risk for harm to the client. Here, that answer is hold pressure on the site to prevent bleeding. 1. Incorrect: There is nothing wrong with applying ice to decrease swelling, however, it is not the priority. Bleeding takes priority over swelling. 3. Incorrect: Yes, you want to monitor the client's needle insertion site at least every 2 hours, but you better make sure the bleeding stops immediately after the procedure first. 4. Incorrect: Nurses must teach about potential complications and ways to avoid them, but that is not the priority here.

A child is being discharged home following a bone marrow transplant. When providing discharge instructions to the parents, what information is most important for the nurse to include? 1. Clean toothbrush weekly with alcohol. 2. Avoid eating raw fruits and vegetables. 3. Drink bottled water the day. 4. Apply heating pad to bruised areas of the skin.

2. Correct:The greatest risk to clients following a transplant is the chance of infection from any source since the client is severely immunocompromised for an extended period of time. There are numerous precautions necessary to avoid bacteria, but one area of concern is food storage, preparation, and consumption. Raw fruits with no skin to peel, such as strawberries, and raw vegetables like broccoli and cauliflower, present a serious risk for bacterial contamination and should not be consumed by new transplant clients. 1. Incorrect: Precise mouth care is vital following a bone marrow transplant; however, rinsing a toothbrush in alcohol is unsafe. Any residual alcohol would cause irritation and trauma to gum tissue, placing the client at risk for mouth inflammation and infection. Clients are instructed to brush teeth twice daily with a soft bristle brush, using a fluoride toothpaste. Some clients are instructed to soak the toothbrush once weekly in a special bleach solution, then rinse in hot water, while others need to replace the toothbrush weekly, based on lab test results. 3. Incorrect: Standing water of any type quickly builds up bacteria, including flower vases and vaporizers. Although bottled water may seem a safe choice, after that bottle is opened, bacteria begins to quickly build up, even if the bottle is recapped. Any water standing more than 15 minutes is considered old and must be thrown out. 4. Incorrect: With bone marrow transplant clients, it will be months before the body begins to stabilize and produce normal blood cells. Bruising and low platelet counts are to be expected for a period of time. When clients develop bruising, the approved treatment is cold compresses or ice packs applied for 15 minutes a couple times per day, and never a heating pad. Additionally, the healthcare provider should be notified so that a current platelet count can be obtained.

The charge nurse is observing a new nurse administer a Mantoux test. The new nurse demonstrates accurate knowledge of the procedure by completing what steps? Select all that apply 1. Administers 0.1 ml of PPD to upper outer arm. 2. Inserts needle under dermis with the bevel up. 3. Uses tuberculin syringe with 27-gauge needle. 4. Wraps site with gauze to prevent leaking. 5. Assesses the injection site after 48 hours.

2., 3., & 5. Correct: The Mantoux test is standardly used to test individuals for immunity to tuberculosis by giving an intradermal injection of tuberculin. This intradermal test uses 0.1 millimeter of solution given with a tuberculin syringe and 27 gauge needle, injected with bevel pointed upward into the inner surface of the forearm. The test must be read between 48 and 72 hours for accuracy. 1. Incorrect: PPD stands for "purified protein derivative", which is the solution injected for this test. The upper outer surface of the arm is just below the deltoid muscle area and is not suitable for an intradermal injection. 4. Incorrect: If the solution is injected properly, a small wheal, or bubble, will appear on the skin which should not be compressed in anyway, particularly with gauze wrap.

The nurse is caring for a client on the psychiatric unit. The client is prescribed fluphenazine 10 mg. The drug is available as an elixir: 2.5 mg / 5 mL. How many mL will the nurse give to the client? ______mL. Round answer to the nearest whole number.

2.5 mg : 5 mL : 10 mg : x mL 2.5 mg/x mL = 50 mg/mL 2.5 mg/x mL = 50 mg/mL x = 20 mL

The RN, LPN, and unlicensed assistive personnel (UAP) are providing care for clients on the nursing unit. Which tasks could be completed only by the RN? Select all that apply 1. Administration of routine medications. 2. Dressing changes. 3. Assessment of newly admitted clients. 4. Calling the primary healthcare provider about lab results. 5. Teaching the diabetic client foot care.

3. & 5. Correct: Admission assessments and teaching must be performed by the RN. The nursing process, along with teaching are outside the scope of practice of the LPN. These are tasks that must be performed by the RN. The LPN can reinforce teaching. 1. Incorrect: Medication administration is within the LPN scope of practice and can be completed by the LPN. 2. Incorrect: Dressing changes may be delegated to the LPN as this is within the LPN scope of practice. 4. Incorrect: The LPN may call lab results to the primary healthcare provider because this is within the scope of practice for the LPN. If any additional prescriptions are required, the LPN can take these prescriptions over the phone.

A client with a new colostomy is learning to perform a colostomy irrigation. The nurse knows the teaching was successful when the client makes what statement? 1. "My spouse can verbalize all the steps in order." 2. "I have attended all the sessions on ostomy care." 3. "I can do the irrigation if I refer to the instructions." 4. "I don't need to irrigate if the ostomy is making stool."

3. CORRECT: The true test of learning is for the client to be able to actually complete a self-care task independently. There is nothing wrong with the client referring to written instructions to complete the task. 1. INCORRECT: While it is beneficial for another family member to be familiar with the process of ostomy irrigation, having the spouse recite the steps does not ensure the client has learned successfully. 2. INCORRECT: Though the client has attended all the teaching sessions presented on performing self-ostomy care, that fact does not guarantee the client could actually successfully complete the task. 4. INCORRECT: A surgeon generally will order daily irrigation of a new ostomy to help establish a consistent bowel pattern. Only the surgeon can determine when the client may discontinue ostomy irrigation.

A newly hired nurse in a long term care facility has been asked to assist with revising old policies regarding family visitation schedules. The nurse considers various ideas submitted by team members. What proposal would the nurse determine to best meet the needs of families and clients in long term care? 1. Plan all care to be completed in early morning to allow afternoon for visitation. 2. Schedule visiting times in two-hour increments so clients are not overwhelmed. 3. Encourage clients and families to develop mutually appropriate visitation times. 4. Allow families unlimited visitation around the clock to meet their schedules.

3. Correct: A long term care facility is considered a client's "home environment", and families are encouraged to visit often. Because facilities generally prefer some type of consistent schedule for staffing purposes, older visitation policies were often very restrictive. Flexible hours allow clients and families to spend more quality time together, increasing positive outcomes and satisfaction. 1. Incorrect: This is not completely practical for everyone. Though it may benefit staff to have one particular goal, some clients cannot tolerate to have everything performed at one time, and instead need short rest periods during personal care. This schedule may leave some clients too exhausted to visit with family. 2. Incorrect: Restricting visitation to two hours is not appropriate, particularly for families traveling long distances to visit a client. Those residing in long term care facilities benefit greatly from time spent with family or even older friends. A two-hour limit on visits discourages quality time. 4. Incorrect: This option would create total chaos, interrupting sleep patterns and staffing schedules. Some general guidelines are necessary, with input from clients and family to individualize any special requests with assistance from the facility.

A 70 year-old client reports not sleeping well at night, having trouble staying asleep, and awakening about 4:00 a.m. What should the nurse teach the client about sleep patterns in the elderly? 1. Don't worry about a few hours of lost sleep. 2. Elders need as much sleep as younger adults. 3. Caffeine and some medications may interfere with sleep. 4. Elders sleep more than younger adults.

3. Correct: Caffeine and some medications may interfere with sleep. 1. Incorrect. The client is concerned about the sleep problem, and the nurse should address the client's concerns. Sleep disturbances can also indicate depression. This option is denying their concerns. 2. Incorrect. Elders actually require less sleep because they are less active. Elderly do not need as much sleep. 4. Incorrect. Elders are likely to have more disturbed sleep. They usually do not need more sleep.

The nurse is caring for a client undergoing electroconvulsive therapy (ECT) for major depression. What is the nurse's most important intervention during the treatment? 1. Monitor vital signs and cardiac functioning. 2. Provide support to the client's arms and legs. 3. Provide suctioning as needed. 4. Place electrodes on temples.

3. Correct: Ensuring patency of the airway is the nurse's first priority. The client should also be NPO for 6-8 hours prior to the procedure. 1. Incorrect: This is an intervention, but does not come before airway. Vital signs do need to be monitored but the client's breathing is a higher priority. 2. Incorrect: This is an intervention, but does not come before airway. Support the extremities due to the seizure activity but highest priority remains airway. 4. Incorrect: This is done prior to initiation of the procedure to deliver the electrical stimulation. The question asks for the most important intervention during the treatment.

After making initial assessment rounds on assigned clients in the morning, the RN tells the charge nurse that the clients are too difficult. The RN requests reassigning at least one of the clients to another nurse. What is the best response by the charge nurse? 1. Offer to take one of the clients. 2. Notify the nursing supervisor of the situation. 3. Ask the RN why the assignment is too heavy. 4. Explain to the RN that all the nurses have the same number of clients.

3. Correct: It would be best to explore the reason the RN thinks the assignment is too heavy. The charge nurse needs additional information to make a decision. This will allow the charge nurse to analyze the situation to make a better decision as to whether the assignment should be changed. 1. Incorrect: Volunteering to take a client would add more work to the charge nurse when this might not be necessary. The charge nurse's best response is to first obtain the needed information to make the best decision. 2. Incorrect: The charge nurse should first obtained the needed information and then decide whether to notify the nursing supervisor. The situation should be explored before bringing the supervisor in on the situation. 4. Incorrect: It is important to hear what the nurse is saying and not to dismiss the request by refusing to reassign the clients. Something new could have occurred with the clients, making the assignments too heavy. The charge nurse might not have realized all the responsibilities of taking this team of clients. Client assignments are based on client acuity and nurses do not necessarily have the same number of clients.

Which side effect of vincristine should the nurse immediately report to the primary healthcare provider? 1. Nausea 2. Fatigue 3. Paresthesia 4. Anorexia

3. Correct: Paresthesia is a side effect of some chemotherapeutic medications and if it occurs, the primary healthcare provider needs to modify the dosage or discontinue. 1. Incorrect: Nausea and vomiting are common side effects of many chemotherapeutic medications. 2. Incorrect: Fatigue is a common side effect of many chemotherapeutic medications. 4. Incorrect: Anorexia is a common side effect of many chemotherapeutic medications.

A client has been admitted with a diagnosis of portosystemic encephalopathy secondary to Laennec's cirrhosis. The primary healthcare provider writes prescriptions based on the lab values. The nurse would monitor the effectiveness of medications by observing for what specific neurologic changes in the client? Exhibit: Lab Results: Sodium: 129 meq/dl Potassium: 3.0 meq/dl Albumin: 2.0 gm/dl Ammonia: 80 mcg/dl Bilirubin: 2.0 gm/dl BUN: 32 mg/dl Creatinine: 2.0 mg/dl BP: 100/60 Pulse: 110 Resp: 28 Medication: Furosemide (Lasix) 60 mg IV every 12 hours Lactulose 30 mg by mouth every 4 hours K-Dur 40 meq by mouth twice daily Albumin 25% 100 mL IV twice daily 1. Increased urination and improved memory. 2. Increased blood pressure and lower pulse. 3. Frequent diarrhea with orientation x three. 4. Clear speech and +2 pitting edema to BLE.

3. Correct: Neurologic deterioration in clients with cirrhosis is secondary to increased ammonia levels in the body and brain, resulting in development of encephalopathy. Frequent diarrhea, secondary to the use of lactulose, helps rid the body of ammonia, allowing the client's orientation to improve to normal. 1. Incorrect: Although increased urination is expected because of the furosemide, this medication would not impact the client's memory. Additionally, there is no indication whether the improvement reflects changes in long-term or short-term memory. 2. Incorrect: As the client slowly improves, vital signs should begin to stabilize, with the blood pressure increasing and the pulse decreasing toward the normal range of 60-100. However, neither of these changes would relate to specific changes in the neurologic status. 4. Incorrect: Though the client's speech is now clear, this does not indicate improvements in either orientation or alertness. The client's speech could be clear even with disorientation. The +2 edema in BLE is decreasing but does not indicate neurologic improvement.

What task would be most appropriate to assign to the UAP when caring for a client with ulcerative colitis? 1. Sharing successful anxiety reduction measures. 2. Encouraging the client to express concerns about an ileostomy. 3. Reminding the client to avoid cold foods and smoking. 4. Explaining the rationale for needing a low residue diet.

3. Correct: Reminding clients to follow through on teaching performed by the RN such as to avoid cold foods and smoking would be an appropriate task for the UAP. 1. Incorrect: Sharing successful anxiety reduction measures is teaching. This is the role of the RN and would not be appropriate to delegate to the UAP. 2. Incorrect: Although encouraging a client to express concerns about the possibility of having an ileostomy sounds like something that could be assigned to the UAP, this would require assessment of the client's concerns and should be performed by the RN. 4. Incorrect: Explaining the rationale for needing a low residue diet is teaching. This is outside the scope of practice for the UAP. The RN should retain all tasks related to teaching.

The nurse is caring for a client following a transurethral resection of the prostate (TURP). The client has a 3 way irrigation catheter in place. Which observation would indicate the need to slow the irrigation? 1. Clots in urine 2. Bladder pressure 3. Clear urine 4. Bladder spasms

3. Correct: The irrigation is regulated so that the urine is free of clots and slightly pink tinged. When it becomes clear after surgery, the fluid is going too fast and not clearing any blood clots effectively. 1. Incorrect: The irrigation should be increased if you see clots in order to keep the catheter patent. 2. Incorrect: Bladder pressure may mean that the indwelling urinary catheter is obstructed. Either increase flow or manually irrigate catheter to ensure patency and no retention of fluid in the bladder. 4. Incorrect: Bladder spasms occur with clots so you do not want to slow the irrigation if this happens. This would indicate the need for increasing the irrigation fluid rate.

The nurse is providing care for an elderly client who has a percutaneous endoscopic gastrostomy (PEG) feeding tube and is receiving continuous feeding. Which interventions should the nurse include when providing care? Select all that apply 1. Add medications to enteral feeding formula. 2. Change dressing around insertion site weekly. 3. Flush feeding tube with 30 mL warm tap water every 4 hours. 4. Maintain head of bed at 30 degree elevation. 5. Monitor for hypoglycemia.

3., & 4. Correct: All enteral feedings require flushing. Flush feeding tubes in adults with 30 mL of warm tap water every 4 hours during continuous feedings or before and after each intermittent feeding. To prevent aspiration, elevate the head of bed to a minimum of 30 degrees, but preferably 45 degrees. 1. Incorrect: Do not add medications to the enteral feeding formula. Medications can interact with the formula and may case the feeding tube to clog. 2. Incorrect: The dressing around the insertion site should be changed at least daily. 5. Incorrect: The elderly client is more likely to experience hyperglycemia rather than hypoglycemia. This is due to the high carbohydrate load in some enteral feeding formulas.

The nurse is making an initial home visit to a client newly diagnosed with diverticulitis. The client had been on a liquid diet but is now to begin solid foods appropriate for the disease process. The nurse knows dietary teaching has been successful when the client selects which meal? 1. Hamburger on sesame roll, macaroni and cheese, tossed salad 2. Lamb chop with brown rice, cooked broccoli, baked potato 3. Pork with sauerkraut, baked beans, and coconut cake 4. Spaghetti with meatballs, fruit cocktail, garlic bread

4. CORRECT. Diverticulitis is an inflammation within the small, outpouching which can develop in the colon. A low residue/low fiber diet limits the amount of food waste passing through the large intestine, allowing the intestinal tract to rest and heal. Cooked pasta, along with ground, well cooked meatballs, is tolerated well. Canned fruits like fruit cocktail are far better than fresh fruit to control diarrhea and cramping. Garlic bread is also acceptable. 1. INCORRECT. A hamburger is well ground, cooked meat, which is acceptable for this client, but not when served on the sesame roll. Seeds and nuts tend to lodge in the diverticula, leading to pain or infection. Macaroni and cheese is a great menu item for the client. However, fresh vegetables, though healthy, do not breakdown easily, resulting in large amounts of undigested material passing through, or getting stuck, in the large intestine. 2. INCORRECT. Lamb is an acceptable as part of a low residue diet, but clients are instructed to eat white rice rather than the whole grain brown rice. Vegetables which create gas even when cooked, such as broccoli, could lead to a serious exacerbation of diverticulitis. Well cooked potatoes are permitted, but not if prepared with the skin, such as the baked potato. 3. INCORRECT. Several parts of this menu selection present a major problem for the client; in fact, only the pork is acceptable. Sauerkraut is prepared cabbage, and even when cooked, causes digestive and gas issues. Baked beans should be avoided for the same digestive reasons, and coconut is in the category with nuts or seeds.

A client with a history of angina has returned to the unit following a cardiac catheterization. What nursing action has the highest priority? 1. Obtain vital signs every thirty minutes. 2. Assess pedal pulses every ten minutes. 3. Place the call bell within client's reach. 4. Keep affected extremity immobilized for 6 hours.

4. CORRECT: The greatest risk following a cardiac catheterization is the potential for hemorrhage, most often from the insertion site. Therefore, the affected extremity must remain straight and immobilized for 4-6 hours after the procedure. 1. INCORRECT: The frequency of vital signs is determined by facility protocol, but generally vital signs are obtained every ten minutes for the first half hour, then every fifteen minutes for another half hour. While vital signs provide valuable information to compare to baseline, another action is more important. 2. INCORRECT: It is vital to assess pedal pulses in order to verify circulation following a catherization. The frequency is based on facility protocol. However, this action is not the highest priority. 3. INCORRECT: Because the client is on bed rest, it is crucial for the client to be able to summon staff when needed. Despite the importance of this action, there is an even more important action.

The nurse discovers that a client was given the wrong medication. After verifying the client is stable, an incident report is completed. What is the proper disposition of the report? 1. Send a copy of the report to the primary healthcare provider. 2. Notify the State Board of Nursing about the incident report. 3. Document that a report was completed on the client's chart. 4. Give the report to the hospital's risk management team.

4. Corect: The purpose of an incident report is to describe and document a particular event, injury, medication error, or other occurrence that affects a client or staff member. This report is then sent directly to the hospital risk management team for the express purpose of developing a plan or protocols to prevent a repeat occurrence. 1. Incorrect: Although the primary healthcare provider will need to be informed of the medication error and the client's current status, including vital signs, a copy of the incident report is not provided. 2. Incorrect: The State Board of Nursing is rarely notified about medication errors or the existence of an incident report. 3. Incorrect: The information documented on the main chart includes the client's current status and assessment specifics. It should also be documented that the primary healthcare provider was notified. However, there should not be any mention of the incident report on the client's chart.

The nurse asks if the client has an advance directive. The client responds by saying, "I have heard of advance directives, but I do not have one. What is an advance directive?" Which response by the nurse is appropriate? 1. Specifies your wishes regarding your personal effects and finances should you become unable to make decisions. 2. Specifies your wishes regarding healthcare and your finances should you become incapacitated. 3. Similar to a will, it specifies your wishes for burial should you die during hospitalization. 4. A form of a living will. It specifies your wishes regarding healthcare and treatment options should you become incapacitated.

4. Correct: An advance directive is a legal document prepared by a competent individual that specifies what treatments, if any, the client desires should the client become incapacitated or unable to make informed healthcare decisions in the future. The document includes wishes regarding resuscitation measures, withdrawing treatment and life support, and end-of-life care. 1. Incorrect: An advance directive does not address client personal effects and finances. These might be included in a last will and testament, but are not part of an advance directive. 2. Incorrect: An advance directive does not address client personal effects and finances. These might be included in a last will and testament, but are not part of an advance directive. 3. Incorrect: An advance directive does not address burial wishes.

The hospice nurse has been assigned a new client who is being cared for at home by family members. Based upon the client's physical assessment, the nurse is aware that the client's death is imminent. What is the nurse's most important role in the care of the family at this time? 1. Providing care for the client, allowing the family to rest. 2. Providing education regarding the symptoms the client will likely experience. 3. Allowing the family to express their feelings and actively listening. 4. Communicating the client's impending death to the family while they are together.

4. Correct: Communicating news of the client's impending death to the family while they are together. The nurse's most important role in the care of the family is compassionate communication. The family needs to be informed about the situation so that they are prepared for the client's death and can provide support to one another. 1. Incorrect: Providing respite time when death is imminent is not a priority. Family should be allowed to spend time with the client. They will, more than likely, want to be with the client in the last hours. 2. Incorrect: When death is imminent, education of what to expect is appropriate, but does not take priority over compassionate communication. Compassionate communication is most important at this time. 3. Incorrect: Silence and listening sends a message of acceptance and comfort. Although important, allowing for expression of feelings is not more important than preparing the client for the imminent death.

Which intervention should the nurse recommend to the adult child who is caring for an elderly parent diagnosed with Alzheimer's disease (AD)? 1. Give parent a small dog for company and comfort. 2. Reset the water heater to 125 degrees Fahrenheit (51.67 degree Celsius) to prevent burns. 3. Place mirrors in multiple locations so parent sees images of self. 4. Make floors and walls different colors.

4. Correct: People with Alzheimer's disease (AD) get more confused over time. They also may not see, smell, touch, hear and/or taste things they once did. By creating a contrast in color between the floors and walls makes it easier for the person with AD to see. 1. Incorrect: Be careful with small pets. The person with AD may not see the pet and trip over it. This is a safety issue. A fall could cause a major injury to the client. 2. Incorrect: The water heater should be set below 120 degrees Fahrenheit (48.8 degrees Celsius) to prevent burns. 3. Incorrect: Limit the size and number of mirrors in the home. Mirror images may confuse the person with AD. They may not recognize self and may see the person as a stranger.

While preparing an information sheet for a client diagnosed with a vancomycin-resistant enterococcus (VRE) urinary tract infection (UTI), the home health nurse should include which instructions? 1. Wash hands with hot water and soap when hands are soiled. 2. Gloves are not needed in the home since contamination with VRE has already occurred. 3. Wash hands before using the bathroom and after preparing food. 4. Clean the bathroom and kitchen with warm water and bleach.

4. Correct: The bathroom and kitchen should be cleaned with warm water and bleach to decrease contamination. The client should wash hands after using the bathroom and before preparing food. 1. Incorrect: Instructing the client and family to wash with hot water can cause drying and cracking of the skin. Hands should be washed with all contacts. Washing hands is the single most important thing to do to prevent infection. 2. Incorrect: Gloves are needed with VRE to prevent spread of infection. Gloves are especially needed if contact with blood or other infectious materials is anticipated. 3. Incorrect: Hands should be washed after using the bathroom and prior to handling or preparing food.

A client with diabetes is hospitalized for debridement of a non-healing foot ulcer. Following the procedure, the nurse notes that the client has become confused and combative. The family expresses concern with the behavioral changes and requests that the client be restrained in bed. What is the nurse's priority action? 1. Notify the primary healthcare provider. 2. Apply a vest restraint as requested by family. 3. Move client to a room near the nurse's desk. 4. Obtain a finger-stick blood glucose level.

4. Correct: The client's behavior has negatively changed following the ulcer debridement procedure. The nurse's priority is to determine the cause of the client's confusion. The nurse is correct to investigate other possible causes for the behavior changes, including an abnormal glucose level in this diabetic client. 1. Incorrect: The nurse will indeed have to contact the primary healthcare provider about the client's change in behavior. However, the first priority would be to assess the client and collect data prior to placing that phone call. 2. Incorrect: The nurse understands that restraints cannot be applied by family request. Additionally, applying a restraint can often increase negative behavior while ignoring the actual cause. 3. Incorrect: Although assigning confused clients to a room near the nurses' station is an accepted practice, this does not determine the cause for the changing behavior and is not a priority at this time. The nursing priority is to assess the client for possible factors causing the behavior changes.

The charge nurse identifies that three admissions were received during the night shift, one nurse has called in sick, and the clients on the unit have high acuity levels. What action should the nurse implement first to ensure client safety? 1. Take report on the most critical clients first. 2. Encourage the staff to help each other. 3. Assign one additional client to each nurse. 4. Call the nursing supervisor to request additional staff immediately.

4. Correct: The hospital nurse to client staffing ratio should reflect the complexity of nursing care for high acuity clients. The nurse should call for immediate help so that a safe care environment is maintained for all clients. The charge nurse should notify the nursing supervisor who will seek additional staff. The nursing supervisor may be able to assist with client care until another nurse can come in to work. 1. Incorrect: The critical clients are important, but all clients must be considered. The charge nurse must evaluate each client's status and needs to assign the appropriate staff to care for them. The safety of each client must be reviewed. 2. Incorrect: The charge nurse may encourage the staff to work together. This is a positive action but the priority for the charge nurse is to ask for additional staff to maintain safe nursing care. 3. Incorrect: Each nurse may have to increase his/her client load until adequate staffing can be obtained. However, calling the nursing supervisor to request help is the first action.

A client with a history of schizophrenia is currently being treated in a mental health facility. The client wants to vote in an upcoming election. The nurse understands what is true about the legality of this action? 1. Primary healthcare provider can decide if client may vote. 2. Psychiatric clients cannot vote if taking medication. 3. A lawyer must approve the finished ballot. 4. An absentee ballot from the polling place can be obtained.

4. Correct: There are very few reasons that a United States citizen would lose the right to vote in any election, and those few are mostly legal violations. A client who is hospitalized, whether in a medical or psychiatric facility, still retains the right to vote. The nurse, or facility designee, must advocate for this client by obtaining an absentee ballot, following the laws of that state, and is required to provide privacy for the client to complete that ballot. 1. Incorrect: The primary healthcare provider has no authority over the client's ability to cast a vote. Regardless of any mental health diagnosis, this client still retains the legal right to vote in any election. In fact, notifying the primary healthcare provider of the client's intent to vote violates the client's privacy. 2. Incorrect: Whether a client takes medication does not affect the client's right to cast a ballot in any election. Refusing this client, the right to vote based on medication use would be considered discriminatory. 3. Incorrect: A lawyer is not required to approve either the client's voting rights, or the completed ballot. In fact, having anyone else look at the client's ballot would be a violation and is definitely illegal. A client's ballot is private and protected by both state and federal law.

The nurse is caring for a client prescribed ondansetron due to postoperative nausea. Which side effect is the nurse most worried about the client experiencing with administration of this medication? 1. Respiratory depression 2. Hyperglycemia 3. Malignant hypertension 4. Torsades de pointes

4. Correct: Torsades de pointes is a life threatening dysrhythmia which can occur with administration of ondansetron. Clients who are at increased risk for Torsades de pointes are those with underlying heart conditions and those with hypomagnesemia or hypokalemia. 1. Incorrect: Respiratory depression is not a common side effect of ondansetron. Headache and drowsiness are more common. 2. Incorrect: Hyperglycemia is also not a side effect of ondansetron. Hyperglycemia is high blood sugar and may produce symptoms of urinary frequency, increased thirst and increased appetite. Hyperglycemia is not related to ondansetron. 3. Incorrect: Malignant hypertension is extremely high blood pressure that develops rapidly and causes some type of organ damage. Although it is a serious condition this is not a side effect of ondansetron.

How should the nurse interpret the arterial blood gas (ABG) results of a client admitted with dehydration? Exhibit: ABGs: pH - 7.49 PaO2 - 99% PaCO2 - 29 HCO3 - 23 Select all that apply 1. Metabolic acidosis 2. Respiratory acidosis 3. Metabolic alkalosis 4. Respiratory alkalosis 5. Uncompensated 6. Partially compensated 7. Fully compensated

4., & 5. Correct: The blood gases confirm respiratory alkalosis. Why? The pH is 7.49 (normal 7.35-7.45). This pH indicates alkalosis since it is high. Which other chemical says alkalosis? The PaCO2 of 29 (normal 35-45) is low which indicates alkalosis. The HCO3 is normal. This means that the client is in uncompensated respiratory alkalosis. 1. Incorrect: The blood gases confirm respiratory alkalosis. The Bicarb is normal, so the problem is not metabolic. 2. Incorrect: The blood gases confirm respiratory alkalosis. The PaCO2 of 29 (normal 35-45) is low which indicates alkalosis. For this client to be in respiratory acidosis, the PaCO2 would be greater than 45. 3. Incorrect: The blood gases confirm respiratory alkalosis. The Bicarb is normal, so the problem is not metabolic. 6. Incorrect: Compensation has not begun because the bicarb is normal. To compensate the bicarb would need to decrease to bring the pH down to normal. 7. Incorrect: Fully compensated would occur if the pH is normal with abnormal CO2 and bicarb.

The nurse is preparing to hang an IV bottle of fat emulsions 20% on a client. At what rate should the nurse set the IV infusion pump? Answer in numbers only. Exhibit Primary Healthcare Provider Prescription: 200 mL fat emulsion 20% IV at 17 mL/hour. Fat Emulsion 20%: Fat Emulsions 20% 200 mL

Answer: 17 Rationale: Prescription: 200 mL fat emulsions 20% IV at 17 mL/hour. Think, when an infusion pump is used, the flow rate is prescribed by the primary healthcare provider and programmed by the nurse by setting the device for milliliters per hour (mL/h). Rule - To regulate an IV by infusion pump, Total mL prescribed/Total hr prescribed = mL/hr If you forgot that, then work the formula for gtts/min - mL/hr x drop factor/time in minutes = gtts/min The drop factor for the infusion pump is 60. 17 mL x 60 / 60 minutes = 17 Therefore, the pump should be set at 17 mL per hour

A cardiac step down unit has requested float staff because of multiple impending admissions. The supervisor can only send one LPN/LVN to the floor. Which clients would be appropriate assignments for the LPN/LVN? Select all that apply 1. A client with COPD complaining of shortness of breath on exertion. 2. A post-cardiac catheterization needing assistance with bedpan. 3. A client receiving heparin injections for deep vein thrombosis. 4. A client with atrial fibrillation currently on a diltiazem drip. 5. A client receiving a blood transfusion that requires monitoring. 6. A client post pacemaker insertion, awaiting discharge instructions.

1, 3 & 5. Correct: The LPN is being floated to a specialty floor and appropriate assignments would include clients who are stable. Client #1 has COPD, and, although complaining of shortness of breath, that is not unusual for clients with this diagnosis. Client #3 is receiving heparin sub-q for deep-vein thrombosis, and sub-q injections are within the LPN's scope of practice. Client #5 -It is considered within the scope of practice for an LPN/LVN to monitor a transfusion of a blood product. 2. Incorrect: This client is post cardiac catheterization and remains on bedrest; therefore, the affected leg must be kept straight to prevent femoral hemorrhaging. Because positioning on a bedpan requires rolling of the client, an RN should be assigned to assess the insertion site and monitor for the presence of bleeding. 4. Incorrect: Atrial fibrillation places the client at risk for blood clots. Diltiazem is a calcium channel blocker that has been ordered as a titrated drip to slow heart rate and restore a regular rhythm. Assessing this client and titrating the diltiazem requires the skills of an RN. 6. Incorrect: Discharging a client includes teaching and a review of medications to be taken at home. These areas require the expertise of an RN and would not be appropriate for an LPN/LVN.

Following a large hurricane, multiple clients arrive at the emergency room for treatment. The charge nurse must triage and assign clients to appropriate staff. Which clients could be assigned to an LPN? Select all that apply 1. Child with superficial burns on both upper arms. 2. Adolescent with bruising to left upper quadrant. 3. Crying toddler missing both upper front teeth. 4. Adult reporting headache and blurred vision. 5. Elderly adult reporting nausea and heartburn.

1. & 3. Correct: An LPN should be assigned clients with predictable outcomes. Even though the client is a child, superficial burns require only dry sterile dressings and possibly oral pain medication, both tasks which are within the scope of practice for an LPN. The crying toddler has missing front teeth, but there is no indication this was the result of the hurricane. However, providing care for missing teeth would also be within the LPN scope of practice. 2. Incorrect: Bruising of the left upper quadrant is often indicative of a ruptured spleen and internal bleeding. This adolescent will require further tests, such as CT scan, and possibly emergency surgery. Because of the complexity of the situation, an RN should be assigned this client. 4. Incorrect: Since these clients were injured during the hurricane, the charge nurse must assume the worst. This client is reporting headache and diplopia; therefore, a safe nurse would consider the possibility of head trauma with brain swelling accounting for the blurred vision. Such potential makes this client serious to critical, and as such, should be assigned to an RN for on-going neurologic assessment. 5. Incorrect: While the trauma of a hurricane could adversely affect the digestive system, the charge nurse would assume the worst and suspect the likelihood the client is having a myocardial infarction. Only an RN can complete the appropriate assessment, testing, and other needs expected with an M.I. client.

A client arrives by ambulance after being thrown from a horse. The client is pale, clammy and tachycardic with bruising over left upper abdominal quadrant. The nurse is aware what prescription by the primary healthcare provider takes priority? 1. Obtain blood for type and crossmatch. 2. Administer hydromorphone IV for pain. 3. Increase Lactated Ringers to 150 mL/hour. 4. Send client to radiology for stat CAT scan.

1. CORRECT: The signs and symptoms displayed by the client suggest a ruptured spleen and shock. The greatest concern in this situation is internal bleeding and possible emergency surgery. The client will need blood; therefore, the nurse should immediately obtain blood for type and crossmatch. 2. INCORRECT: There is no indication in the scenario the client has pain. Pain medication should never be administered while the client is still being assessed or is in shock. 3. INCORRECT: Fluids are crucial for clients in shock and increasing the Lactated Ringers to 150 mL/hr. is important to help maintain blood pressure. However, this is not the nurse's priority action. 4. INCORRECT: A CAT scan is often prescribed prior to surgery to verify the extent of splenic injury and the amount of blood in the abdominal cavity. Though the order is written as 'stat', this is not the nurse's priority. Transporting an unstable client to another department requires preparation.

A client returns to the nursing unit post-thoracotomy with two chest tubes in place connected to a drainage device. The client's spouse asks the nurse about the reason for having two chest tubes. The nurse's response is based on the knowledge that the upper chest tube is placed to do what? 1. Remove air from the pleural space 2. Create access for irrigating the chest cavity 3. Evacuate secretions from the bronchioles and alveoli 4. Drain blood and fluid from the pleural space

1. Correct: A chest tube placed in the upper chest is to remove air from the pleural space. Remember air rises and fluid settles down low. 2. Incorrect: Chest tubes are placed in the pleural space to get rid of air, blood, fluid, or exudate so that the lung can re-expand. The purpose is not to create an access for irrigating the chest cavity. 3. Incorrect: The chest tube is inserted into the pleural space because the lung has collapsed due to air, blood, fluid, or exudate. The chest tube does not go into the lung so secretions can not be removed from the bronchioles and alveoli by way of the chest tube. 4. Incorrect: You have to know the purpose of the upper chest tube. Fluid drains down, so the lower one is for fluid.

The nurse walks into a client's room and finds the client exposed while the unlicensed assistive personnel (UAP) is giving the bath. After covering the client with a sheet, what should the nurse do first? 1. Tell the UAP to keep the client covered at all times. 2. Talk with the UAP about providing appropriate care for all clients. 3. Provide teaching to the UAP about privacy for clients. 4. Use the call light to ask for additional assistance in the room.

1. Correct: A comment should be made about keeping the client covered. This instruction is the first action after covering the client. 2. Incorrect: The nurse should talk with the UAP but the discussion should focus specifically about providing privacy for clients. 3. Incorrect: The nurse may want to provide teaching, but this is not first action. Teaching would require allowing enough time to give instructions and then arranging time for return demonstration. 4. Incorrect: The UAP should be allowed to finish the bath. Additional assistance is not needed.

A new nurse is preparing an injection from an ampule. What action by the new nurse would require the precepting nurse to intervene? 1. Snaps the neck of the ampule gently towards the body. 2. Uses a filter needle when drawing up the ampule contents. 3. Folds gauze around the ampule neck before snapping open. 4. Avoids touching edges of the ampule when inserting needle.

1. Correct: An ampule is a glass vial with a narrow, scored neck that must be snapped off to open. Even if the neck of the ampule is covered with gauze, the proper procedure is to snap the top away from the body, not toward the body. If the new nurse attempts to snap the top of the ampule toward the body, the charge nurse would need to intervene immediately. 2. Incorrect: This is a correct action. When a glass container is broken, there is the potential for tiny glass shards to fall into the solution and subsequently be infused into the client. To avoid this situation, a filter needle must be utilized to draw up the solution from the ampule. Once drawn up, the filter needle is removed and a regular needle utilized to inject the solution into the client. This is a correct action. 3. Incorrect: The use of an alcohol wipe or small gauze sponge, wrapped around the neck of the ampule prior to snapping the top open is crucial to prevent injury to the nurse. Exposure to the jagged glass top could easily cut a thumb or finger while holding the vial. No intervention needed here. 4. Incorrect: The scenario asks for an incorrect action requiring intervention by the charge nurse. However, this action is appropriate. It is always important to avoid touching the edges of the opened ampule when inserting the needle to prevent possible contamination of the solution.

After reviewing the client assignments, the LPN/LVN tells the RN the assignment is very unfair and requests that some of the clients be redistributed to the other staff. What should the RN do first? 1. Ask the LPN/LVN how the client assignment should be adjusted. 2. Assign one of the LPN/LVN's clients to another nurse. 3. Encourage the LPN/LVN to use teamwork skills in caring for the clients. 4. Develop a strategic plan to assist with client assignments.

1. Correct: Explore her concerns; this is most therapeutic and helpful response. Finding out what are LPN/LVN's concerns first will help the RN address the LPN/LVN's request and build trust in the healthcare team relationship. 2. Incorrect: This statement does not help the RN understand the LPN/LVN's concern about the assignment, an negates the confidence in the LPN/LVN's abilities and skills. 3. Incorrect: This answer does not acknowledge the LPN/VN's concern. 4. Incorrect: This action will not help address the LPN/LVN's immediate concern with the assignment and makes resolution of the issue much more complicated than it should be.

A client with an acute exacerbation of systemic lupus erythematosus (SLE) is hospitalized with incapacitating fatigue and fever. A urinalysis reveals proteinuria and hematuria. The primary healthcare provider prescribes corticosteroids. During the acute phase of the client's illness, what is most important for the nurse to do? 1. Monitor intake and output and daily weight. 2. Allow for frequent, uninterrupted rest periods. 3. Institute seizure precautions. 4. Protect client from injury that may cause bleeding.

1. Correct: Look at the clues in the stem. Proteinuria and hematuria. When you see proteinuria what do you need to worry about? The kidneys! Protein is a great big molecule. The only way for protein to be seen in the urine is if there are holes in the glomerulus. So the kidneys are being damaged. Thus, the nurse knows that the biggest problem to "worry" about here is renal failure. The best methods for monitoring fluid status and renal status for a client are to monitor I and O and daily weights. (Also, remember that one weight doesn't mean anything. The hematuria indicates that there has already been glomerular damage). 2. Incorrect: Systemic lupus erythematosus (SLE) is an autoimmune disease. In this disease, the body's immune system mistakenly attacks healthy tissue. It can affect the skin, joints, kidneys, brain, and other organs. Fatigue is a major symptom so allowing for frequent, uninterrupted rest periods is important for this client but monitoring for renal failure is more acute. 3. Incorrect: Seizures are a potential problem with SLE, but the ACTUAL problem depicted in the stem of the question, renal failure, takes priority. Look for the option that relates to the renal system. 4. Incorrect: Hemolytic problems can occur with SLE, but this is not the ACTUAL problem depicted in the stem of the question. The stem is indicating a renal problem, so look for a renal answer.

A client arrives at the Emergency Department after receiving 3rd degree burns to the upper chest, neck, and face area. What would be the priority nursing intervention? 1. Prepare for endotracheal intubation. 2. Monitor hourly urinary output. 3. Treatment of the open burn wounds. 4. Assessment and management of pain.

1. Correct: The evaluation and maintenance of the airway should always be the first priority with upper body burns. Edema may develop due to the increased capillary permeability that occurs with burn injuries. Since the burn is so close to the airway, the priority would be rapid assessment and management of the airway. The healthcare provider may need to perform tracheal intubation prophylactically to prevent airway occlusion in this client. If the airway becomes too edematous before intubation can be performed, the client will need an emergency tracheostomy. 2. Incorrect: Monitoring urinary output in burn clients is important due to the possibility of FVD occurring secondary to fluid loss from the burn area and 3rd spacing of fluids. This could result in shock, and decreased renal perfusion. Although this is a very important intervention, it is not priority over the airway. 3. Incorrect: Burn injuries can be disturbing to look at due to the extensive skin injury, but the priority of care would not be the treatment of the wound. The priority must be the airway. 4. Incorrect: Clients with burns may experience severe pain, and will very likely be requesting pain relief. However, the priority of this client must be focused on the airway, not pain. Always consider what can kill the client or cause the most harm.

Which task would be appropriate for the nurse to assign to an LPN/VN? 1. Changing a colostomy bag. 2. Hanging a new bag of total parenteral nutrition (TPN). 3. Teaching insulin self administration to a diabetic client. 4. Administering IV pain medication to a two day post op client.

1. Correct: The only procedure listed that is within the LPN/LVN's practice range is changing the colostomy bag. This is a task that can be delegated to the LPN/LVN. 2. Incorrect: Hanging a new bag of TPN is parenteral therapy requiring a central line. This is outside the scope of practice for the LPN/LVN. Therefore, the RN must perform this task and cannot delegate this to the LPN/LVN. 3. Incorrect: Teaching is outside the scope of practice for the LPN/LVN. Teaching can be reinforced by the LPN/LVN, but they cannot perform the initial teaching. Teaching insulin self administration cannot be delegated to the LPN. 4. Incorrect: The administration of parenteral pain medications is not in the scope of practice for the LPN/LVN. This should not be delegated to the LPN/LVN.

A farm worker comes into the clinic reporting headache, dizziness, and muscle twitching after working in the fields. What condition does the nurse suspect? 1. Pesticide exposure 2. Heat stroke 3. Anthrax poisoning 4. Gastroenteritis

1. Correct: These are symptoms of pesticide exposure when combined with the details given of coming from the fields. Death can result from severe acute pesticide poisoning. 2. Incorrect: The data provided does not lead the nurse to suspect heat stroke. The stem does not tell the temperature the farmer is working in. Heat stroke signs and symptoms include increased sweating, tachypnea and temperature greater than 105.8°F (41.0°C). 3. Incorrect: The data provided does not lead the nurse to suspect anthrax poisoning. The worker has been outside in a field. This is not a risk factor for anthrax exposure. Inhalation anthrax develops when you breathe in anthrax spores. It's the most deadly way to contract the disease, and even with treatment it is often fatal. Initial signs and symptoms of inhalation anthrax include: Flu-like symptoms, such as sore throat, mild fever, fatigue and muscle aches, which may last a few hours or days. Mild chest discomfort, Shortness of breath, Nausea, Coughing up blood, Painful swallowing. 4. Incorrect: The data provided does not lead the nurse to suspect gastroenteritis. These signs and symptoms do not go with gastroenteritis. Gastroenteritis signs and symptoms include diarrhea, nausea, vomiting, fever and abdominal cramping.

A client admitted with somnolence has a history of chronic bronchitis and heart failure. Vital signs on admit are T 101.8ºF (38.8ºC), HR 106, R 26/shallow, BP 90/58. ABGs are pH 7.2, PCO2 75, HCO3 26. The nurse determines that this client has which acid/base imbalance? 1. Respiratory acidosis 2. Respiratory alkalosis 3. Metabolic acidosis 4. Metabolic alkalosis

1. Correct: This client has a respiratory problem. Respiratory failure, COPD, and muscular weakness can lead to respiratory acidosis. Signs & symptoms: hypoventilation, sensorium changes, somnolence, semicomatose to comatose state. pH < 7.35, pCO2 > 45, HCO3 normal. 2. Incorrect: This is not alkalosis since the pH is 7.2 showing acidosis. 3. Incorrect: Not a metabolic related acid/base imbalance, because the HCO​2 is 26 and within the normal range. 4. Incorrect: Not a metabolic related acid/base imbalance, because the HCO​2​ is 26 and within the normal range.

The nurse is removing the client's peripheral IV line prior to discharge. The nurse completes the appropriate steps in what order? Stabilize cannula with one hand. Apply gauze and tape tightly. Wash hands and apply gloves. Loosen tape and tegaderm cover. Clamp IV line closed securely. Drag and Drop the items from one box to the other.

1. Wash hands and apply gloves. 2. Clamp IV line closed securely. 3. Stabilize cannula with one hand. 4. Loosen tape and tegaderm cover. 5. Apply gauze and tape lightly. When preparing to remove a peripheral IV line, the nurse begins by washing hands and applying non-sterile gloves. Next, the clamp is closed on the IV line or saline lock extension to prevent fluid or blood from leaking during process. Third, the nurse needs to stabilize the cannula with one hand to prevent trauma at the insertion site. Fourth, carefully begin to loosen all the tape on the site. The bottom dressing or tegaderm, is loosened last. Lastly, the nurse will place large, folded gauze square over the insertion site and gently pull the cannula out of the skin, while placing pressure on that gauze. After holding the gauze in place for a few moments, and checking for excess bleeding, the nurse will tightly tape that gauze square in place, providing pressure over the site. The client should be instructed to keep that dressing in place for at least one hour.

The community health nurse is planning to teach nutritional education to a group of adults attending a health fair. What tips about health eating should the nurse include? Select all that apply 1. Pay attention to fullness cues during meals. 2. Make one fourth of the plate fruits and vegetables. 3. Drink sweet tea rather than soft drinks with meals. 4. Eat foods low in dietary fiber. 5. Consume less than 30% of calories from saturated fatty acids. 6. Use a smaller plate for meals.

1., & 6. Correct: Pay attention to hunger and fullness cues before, during, and after meals. Use them to recognize when to eat and when you have had enough. Portion out foods before eating. A smaller plate will make the amount of food look larger. 2. Incorrect: Make half the plate fruits and vegetables. 3. Incorrect: Cut calories by drinking water or unsweetened beverages rather than drinks with sugar, such as soft drinks and sweet tea. 4. Incorrect: Diets should be high in fiber coming from fruits, vegetables, and whole grains. 5. Incorrect: Individuals should consume less than 10% of calories from saturated fatty acids (approximately 20 grams of saturated fat per day in a 2000 calorie diet).

A charge nurse is observing a new nurse for proper use of standard precautions for infection control. Which actions indicate that standard precautions are being followed? Select all that apply 1. Wearing clean gloves to convert an IV to a saline loc 2. Donning sterile gloves for a cesarean dressing change 3. Wearing a N95 respirator while caring for a child who has respiratory syncytial virus (RSV) 4. Putting on a gown to take care of a client who has toxoplasmosis 5. Performing hand hygiene after removing gloves

1., 2., & 5. Correct: Clean gloves are needed when there is a chance of coming into contact with blood or body fluids, which is likely to happen when converting an IV to a saline loc. Dressing changes in the hospital are a sterile procedure and require the use of sterile gloves. Hand hygiene should be performed before and after contact with a client immediately after touching blood, body fluid, non-intact skin, mucous membranes, or contaminated items. It should also be performed after removing gloves, before eating, and after using the restroom. 3. Incorrect:The N95 respirator is used with airborne precautions and RSV requires droplet precautions. 4. Incorrect: Toxoplasmosis is transmitted through the feces of infected cats or through ingestion of raw or rare meats. A gown is not required in the care of this client because transmission occurs through ingestion of the parasites.

A client presents in the emergency department with acute onset of fever, headache, stiff neck, nausea/vomiting, and mental status changes. What interventions should the nurse initiate? Select all that apply 1. Elevate HOB 30 degrees 2. Pad side rails 3. Provide sponge bath if temperature greater than 101°F (38.3°C) 4. Initiate airborne isolation precautions 5. Darken room

1., 2., 3. & 5. Correct: An acute onset of fever, headache, stiff neck, n/v, and mental status changes are consistent with bacterial meningitis. Elevate the head of the bed to promote comfort and decrease intracranial pressure. The client is at an increased risk for seizures, and the nurse should implement seizure precautions which include padding the side rails. A sponge bath is an independent nursing intervention appropriate for a fever greater than 101°F (38.3°C). Darkening the room is also a comfort measure as this client will have photophobia. 4. Incorrect: Droplet precautions should be initiated for the first 24 hours of antimicrobial therapy.

What actions should a nurse take to provide continuity of care when discharging a client diagnosed with hemiparesis to a long-term care facility for rehabilitation? Select all that apply 1. Document current functional status assessment 2. Notify the primary healthcare provider of transfer completion. 3. Transfer essential medical record to the receiving facility. 4. Phone report to the receiving nurse. 5. Send a day's worth of medications with the client to the receiving facility.

1., 2., 3., & 4. Correct: Documentation of the client's baseline functional status is important for the receiving facility to work with in further goal setting. It is the primary healthcare provider's role to receive acceptance for transferring a client to another facility. A copy of select parts of the medical recording, according to facility policy, is another form of communication that will support continuity of care. It is the nurse's responsibility to communicate the client's condition and care plan to the receiving facility nurse in order to support continuity of care. 5. Incorrect: Medications are not transferred with the client to a new facility. A list of current medications is sent to the facility.

The charge nurse tells a nurse that multiple sick calls from the upcoming shift has occurred. The charge nurse asks the nurse who works in a state where mandatory overtime is legal to work an additional 8 hours of mandatory overtime. The nurse has just completed a 12 hour shift. What options would be appropriate for the nurse to take? Select all that apply 1. Assess personal level of fatigue prior to making a decision regarding accepting or refusing assignment. 2. Suggest splitting the shift with another nurse. 3. Accept assignment, documenting personal concerns regarding work conditions. 4. Refuse the overtime assignment, being prepared for disciplinary action. 5. Simply accept the assignment since overtime is mandatory.

1., 2., 3., & 4. Correct: The nurse's level of fatigue must be considered especially under conditions of mandatory overtime. Splitting the overtime shift is an acceptable option that the nurse could suggest in order to solve the staffing problem and decrease the amount of time the nurse will be working. The nurse can accept the assignment, documenting your personal concerns regarding working conditions in which management decides the legitimacy of employee's personal concerns. This documentation should go to your manager. Refuse the assignment, being prepared for disciplinary action. If your reasons for refusal were client safety, nurse safety, or an imperative personal commitment, document this carefully including the process you used to inform the facility (nurse manager) of your concerns. Keep a personal copy of this documentation, provide a copy to the immediate supervisor, and send a copy to the Local Unit Officer. 5. Incorrect. The nurse can also accept the assignment; however, that nurse should document professional concern for client safety and the process you used to inform the facility (manager) of your concerns. Remember if you work overtime, fatigue is not a viable reason for a error and will not stand up in court.

A client is scheduled to have a Cardiac Positron Emission Tomography (PET). What pre-procedure information should the nurse provide to the client? Select all that apply 1. Avoid caffeinated food and drinks for 24 hours prior to test. 2. Do not eat for 4 to 6 hours before the test. 3. Do not wear jewelry. 4. Take calcium channel blocker prescription the day of the test. 5. Wear comfortable, loose-fitting clothing.

1., 2., 3., & 5 Correct: The client should avoid products containing caffeine for 24 hours prior to the stress test. Caffeine increases the heart rate and can affect the results of the test. Don't eat or drink anything except water for 4 hours before the test. The fullness from a meal makes it difficult to perform the stress test. Any jewelry or metal will show up on the scan and cause a false result. The client should dress in loose, comfortable clothing the day of the test because the stress test consists of intense exercise. 4. Incorrect: Clients are asked to hold beta-blockers, calcium channel blockers, and nitroglycerin medicines prior to a stress test. These medications either increase or slow down the heart rate, which can affect the test.

The nurse has been teaching the client about warfarin for prevention of pulmonary emboli. Which comments by the client indicate understanding of the medication? Select all that apply 1. "I must get my blood levels checked regularly." 2. "I shouldn't change my diet to include a lot of foods containing vitamin K without supervision." 3. "I should eat lots of foods containing vitamin K." 4. "I should report this medication to any primary healthcare provider that I see." 5. "I should not change the dosage without talking with my primary healthcare provider."

1., 2., 4. & 5. Correct: The client should comply with regular follow up visits for checks of INR level. INR is the international normalization ratio and is used for clients taking anticoagulants (blood thinning medications). The client should eat a normal healthy diet, but should not increase foods containing high amounts of vitamin K. The client should report using warfarin to any primary healthcare provider, as treatment may be changed due to this medication. The client should not manipulate the dosage unless instructed by the primary healthcare provider. An identification card or bracelet may also be recommended in case of emergencies. Clients should inform dentists and other healthcare providers especially before a medical procedure. The anticoagulant effect must be closely monitored. 3. Incorrect: Vitamin K reverses the anticoagulant effects of warfarin, so instruct the client to avoid foods high in vitamin K (examples are green leafy vegetables, brussels sprouts, prunes, cucumbers and cabbage).

A client reports crushing chest pain 3 hours prior to arrival in the emergency department. Initial assessment by the nurse reveals a BP of 90/50, a weak, thready pulse at 108/min, cool, clammy skin, and confusion. Which interventions should the nurse perform? Select all that apply 1. Initiate cardiac monitoring. 2. Monitor intake and output hourly. 3. Position client in recumbent position. 4. Limit physical activity. 5. Administer dopamine at 5 micrograms/kg/min.

1., 2., 4. & 5. Correct: This client exhibits signs of cardiogenic shock, a complication of myocardial infarction. Hypotension accompanied by clinical signs of increased peripheral resistance (weak, thready pulse and cool, clammy skin) and inadequate organ perfusion (altered mental status and decreased urinary output) are found in this client. Initiate cardiac monitoring, watching for dysrhythmias, monitor I&O hourly to make sure kidneys are perfused. Limit activity to decrease oxygen demand. Dopamine is administered to increase BP and cardiac output. 3. Incorrect: Position upright to promote optimal ventilation by reducing venous return and lessen pulmonary edema.

A client who is at high risk for developing a stroke has been advised to follow a Mediterranean type diet by the primary healthcare provider. Which food choices, if selected by the client, would indicate to the nurse that the client understands this diet. Select all that apply 1. Grilled eggplant 2. Purple grape juice 3. Bacon 4. Cashews 5. Skim milk 6. Salmon

1., 2., 4., 5., & 6. Correct: It is reasonable to counsel clients to follow a Mediterranean-type diet over a low-fat diet. The Mediterranean type diet emphasizes vegetables, fruits, and whole grains and includes low fat dairy products, poultry, fish, legumes, and nuts. It limits intake of sweets and red meats. 3. Incorrect: Substitute fish and poultry for red meat. When eaten, make sure it's lean and keep portions small (about the size of a deck of cards). Also avoid sausage, bacon and other high-fat meats.

An intubated client admitted to the intensive care unit appears anxious and fearful of the equipment in the room. The nurse observes this and takes the time to explain each piece of equipment and its role in providing care to the client. How does this action demonstrate client advocacy? Select all that apply 1. Providing information to the client. 2. Promoting client compliance. 3. Providing emotional support. 4. Ensuring the client's wishes for treatment are followed. 5. Fostering a sense of security.

1., 3. & 5. Correct: Client advocacy has been described in many different ways and involves many things such as assistance in gaining needed healthcare, assuring quality of care, protection of client's rights, and simply serving as a mediator between the client and the healthcare system as a whole. Client advocacy involves regular communication in which the nurse explains what is being done or likely to happen, reasons for tests or procedures, and simplifying medical terminology into words that can be easily understood. Emotional support is also an aspect of client advocacy that the nurse should employ. The nurse acts as a client advocate by providing information to the client to alleviate fear of the unfamiliar equipment and by fostering a sense of security. 2. Incorrect: This question addressing client advocacy is not related to client compliance. Client compliance may improve if the nurse served as an appropriate client advocate. However, promotion of compliance is not a basic part of advocacy. 4. Incorrect: This question addressing client advocacy is not related to client's healthcare treatment wishes. This would be related to the client's advance directive.

What should the nurse check when assessing a client's balance? Select all that apply 1. Walking on tiptoes 2. Babinski reflex 3. Romberg test 4. Muscle strength of legs 5. Dorsalis pedis pulses

1., 3., & 4. Correct: Asking the client to walk on the tips of the toes assesses foot strength and balance. Muscle strength is needed to maintain balance and a Romberg's test asks the client to stand erect with arms at their side and feet together. The nurse notes any sway or unsteadiness. Then the client does the same thing with their eyes closed for 20 seconds again noting imbalance and sway. A positive Romberg is seen with swaying and moving feet apart to prevent a fall. It indicates a problem with balance. 2. Incorrect: Babinski sign is an important neurologic examination based upon what the big toe does when the sole of the foot is stimulated. If the big toe goes up, that may mean trouble with the central nervous system. This is not part of assessment for balance. 5. Incorrect: Assessing the dorsalis pedis pulse is done as part of a circulatory check not while assessing balance.

An occupational health nurse is planning to teach a group of manufacturing workers how to prevent back injuries. What teaching points should the nurse plan to include? Select all that apply 1. Wear comfortable, low-heeled shoes. 2. When sitting, keep knees slightly lower than the hips. 3. Avoid movements that require spinal flexion with straight legs. 4. Squarely face the direction of anticipated movement. 5. Pivot to turn while holding an object.

1., 3., 4., & 5. Correct: Comfortable, low heeled shoes provide good foot support and reduce the risk of slipping, stumbling, or turning your ankle. Flexion of the spine with the legs straight (toe-touches, sit-ups) will injure the back. Avoid twisting of the back by squarely facing the direction of movement. Move toward or away from your center of gravity. Pivoting is a technique in which the body is turned in a way that avoids twisting of the spine. 2. Incorrect: When sitting, keep knees slightly higher than the hips.

Which client would be appropriate for the RN to assign to the LPN? Select all that apply 1. Client requiring enemas and antibiotics. 2. Newly admitted client with diagnosis of diabetic ketoacidosis (DKA). 3. Client returning from surgery post right upper lobectomy. 4. Client with frequent reports of nausea and vomiting following chemotherapy. 5. Client requiring frequent sterile dressing changes.

1., 4., & 5. Correct: Administering enemas and antibiotics to a client is within the scope of practice of the LPN. Nausea and vomiting are common side effects after a client receives chemotherapy. The LPN can administer antiemetics and monitor fluid status. It is within the scope of practice for the LPN to perform sterile dressing changes. 2. Incorrect: This client is a new admit who is in DKA and would be unstable. 3. Incorrect: This client will require frequent assessments and monitoring for postop complications.

Which tasks should the charge nurse complete at the end of the shift before leaving for the day? Select all that apply 1. Talk to each nurse about concerns related to assigned clients. 2. Call the family of a client suffering from dementia to discuss long term care placement. 3. Briefly assess every client. 4. Complete a client assignment sheet for the oncoming staff. 5. Receive report from the emergency department (ED) on a new client.

1., 4., & 5. Correct: Talking to the nurses about client concerns and completing the client assignment sheet for oncoming staff will provide for a thorough shift change report. It is crucial that the oncoming staff have an opportunity to voice any concerns regarding assignments and clarify any information provided.This proper exchange of information and concerns helps to ensure the safety of clients, provides continuity of care, and possibly prevents problems that might arise if these concerns had not been addressed. Taking the report from the ED could be delayed but is a courtesy to the ED and will provide information about the client that will be useful in making assignments for the next shift. 2. Incorrect: This will take some time and would be best accomplished by sitting with the family to discuss options. Doing this at the end of the shift could prevent completion of the client assignment sheet for the next shift and possibly create unnecessary overtime for the charge nurse. 3. Incorrect: The charge nurse does not have to assess every client. This will take a lot of time, and the charge nurse can get the information needed from the nurses caring for the clients in order to make appropriate client assignments for the next shift.

The primary healthcare provider prescribes an intravenous infusion of D5 W at 125 mL per hour. The tubing has a drop factor of 20 gtt/mL. How many drops per minute should the nurse administer? Round answer to the nearest whole number.

125 x 20 / 60 = 41.666 = 42 Since partial drops cannot be counted, always round to the nearest whole number which, is 42.

A client has developed preeclampsia at 30 weeks' gestation. The nurse is instructing the client on an appropriate diet for preeclampsia. The nurse knows the teaching was successful when the client selects what menu? Select all that apply 1. Caesar salad with feta cheese 2. Grilled cheese with tomatoes 3. Chipped ham on a croissant roll 4. Hot dog with a glass of soda pop 5. Chicken sandwich on wheat toast

2 and 5. CORRECT: A high protein, calcium rich diet is most important for the preeclampsia client who is losing protein in urine. Grilled cheese is an excellent selection for lunch, especially since it contains tomato slices, which adds another level of nourishment and vitamins. Additionally, a chicken sandwich, particularly on whole wheat toast, is very appropriate for this preeclampsia client. 1. INCORRECT: Caesar dressing is made with raw eggs, exposing the client to the potential for salmonella. Pregnant woman should not eat raw foods, including eggs, fish, or meat. Additionally, feta cheese is a 'soft cheese', exposing the client to another bacterium known as listeria. Although a salad could be a good choice, this particular salad is not healthy. 3. INCORRECT: The need to restrict salt is not a priority for preeclampsia clients, but chipped ham is a processed meat containing less protein than other meats and increasing the risk for contracting listeria. The croissant roll is made of refined white flour and sugar. The client would benefit more from whole grain products. 4. INCORRECT: Processed meats, such as hot dogs, are not the healthiest choice for the client, as they increase the risk for listeria. Also, a client with preeclampsia should avoid alcohol, caffeine, and refined sugar to help control the blood pressure. The glass of soda pop is not a healthy selection.

In which situations should the nurse notify the primary healthcare provider of a medication incident? Select all that apply 1. Every occurrence. 2. Client is harmed or dies. 3. Medication incident is a near miss. 4. Nurse administers an incorrect dosage. 5. Client questions the medication color.

2. & 4. Correct: The primary healthcare provider should be notified if harm is brought to the client or death occurs as a result of the medication incident. The primary healthcare provider should be notified if the nurse administers an incorrect dosage to the client, and an incident report needs to be completed in this situation. 1. Incorrect: The primary healthcare provider should be notified if harm is brought to the client but not for all events with medications. An incident report should be completed so the hospital can track incident patterns for quality improvement. 3. Incorrect: Near misses do not need to be reported to the primary healthcare provider. Following the rights of medication administration every time ensures medication error prevention. 5. Incorrect: The nurse should answer questions regarding medication color. Depending on the manufacturer, the shape and color of the medication can vary.

A new nurse enters the linen room for supplies and finds a pile of sheets on fire. What type of fire extinguisher is most appropriate for the nurse to use in this situation? 1. Foam type 2. Water only 3. Dry powder 4. Carbon dioxide

2. CORRECT. A "water only" fire extinguisher is used for Class A fires, which includes solid combustibles such as wood, paper and textiles. As long as no electric equipment is plugged into a socket in the room, the water only extinguisher is most appropriate. 1. INCORRECT. Although foam extinguishers can be utilized for both Class A and Class B fires, it is not the most appropriate extinguisher for a linen room since the nurse is unaware of electric equipment that may be charging in that room. Foam extinguishers are more appropriate for flammable liquids. 3. INCORRECT. Dry chemical, or powder, extinguishers are good for mixed material fires and electrical fires such as those that may occur in an office. However, these are not recommended for small, enclosed spaces such as a linen room because of the danger of inhaling the dry chemical. 4. INCORRECT. Carbon dioxide extinguishers are best for electrical fires or flammable liquids like paints because CO2 prevents conduction. This Class B type of extinguisher is appropriate for garage, car and truck fires or even tanker fires, but not Class A blazes.

A charge nurse is teaching a new nurse on the labor and delivery floor the proper positioning of a client following an epidural. The charge nurse knows the teaching was successful when the new nurse places the client in which position? 1. Lithotomy 2. Left-lateral 3. Semi-Fowler's 4. Right-lateral

2. CORRECT: The left-lateral position is most appropriate following epidural anesthesia. In this position, the placenta is well perfused and the client is less likely to experience side effects from anesthesia, such as hypotension. 1. INCORRECT: The lithotomy position is supine with legs separated, knees flexed and elevated with feet supported in stirrups. Such a position is appropriate for gynecologic exams, but would place too much pressure on the vena cava at this time. 3. INCORRECT: In this position, the client is supine with the head of the bed elevated between 30 and 90 degrees. This is a good position for those with breathing difficulties; however, following an epidural, elevating the head may drop the blood pressure, while leaving the client supine and putting pressure on the vena cava. 4. INCORRECT: The right-lateral position is on the right side, with left leg flexed toward the head, and is useful to avoid hypotension. But this is not the best position following an epidural for improving uteroplacental perfusion.

A client is prescribed phenobarbital to control seizures. Which medication prescribed for the client would the nurse recognize interacts with phenobarbital? 1. Lovastatin 2. Loratadine 3. Lansoprazole 4. Lactulose

2. Correct: Both of these drugs can cause CNS depression. There is a drug to drug interaction between antiseizure medications and antihistamines. Loratadine is the only medication in the answer options that can cause CNS depression. 1. Incorrect: Lovastatin is indicated for the treatment of increased cholesterol and triglyceride levels. There is no drug to drug interaction that exists between phenobarbital and lovastatin. 3. Incorrect: Lansoprazole is a proton-pump inhibitor indicated for the treatment of stomach ulcers and GI complaints. There is no drug to drug interaction that exists between phenobarbital and lansoprazole. 4. Incorrect: Lactulose is an ammonia reducer and laxative. It is indicated for the treatment of constipation and to decrease the ammonia level in the treatment of client's with liver disease. There is no drug to drug interaction that exists between phenobarbital and lactulose.

A client scheduled for electroshock therapy becomes anxious prior to the initial treatment and refuses the procedure. What is the nurse's priority at this time? 1. Administer pre-op sedation to help the client relax. 2. Notify the primary healthcare provider of the client's refusal. 3. Remind the client that the consent is already signed. 4. Ask the family to help convince the client to reconsider.

2. Correct. The client has withdrawn consent for the procedure; therefore, the primary healthcare provider should be informed immediately to cancel the treatment. The primary healthcare provider may wish to speak with the client, but the client can legally refuse any procedure at any time. 1. Incorrect. Pre-op sedation is considered part of the procedure for which the client has withdrawn consent. Giving this medication would violate the client's right to refuse treatment and could be considered assault by the nurse. 3. Incorrect. Signing a consent form indicates that the primary healthcare provider has informed the client of all potential risks of the procedure. The client's signature represents an acknowledgement and understanding of that explanation. It is not an iron-clad contract agreeing to have the procedure. 4. Incorrect. Involving family members to try to convince the client to have this procedure would be unethical and could be considered coercion. Depression does not make the client incompetent to make decisions about healthcare options.

Following a motor vehicle accident, a client is brought to the emergency room with shallow, labored respirations. The client is intubated and placed on a ventilator. What is the nurse's priority action immediately after the intubation? 1. Suction to clear all secretions 2. Listen for bilateral breath sounds 3. Secure the endotracheal tube 4. Obtain x-ray to verify tube placement

2. Correct: All actions are important but assessment is the nurse's immediate priority. Clear and equal bilateral breath sounds along with equal chest wall movement would confirm that the endotracheal tube has been correctly inserted into trachea. 1. Incorrect: Although suctioning after intubation is an appropriate action, the nursing process requires assessment first. Clearing secretions will be more effective if the endotracheal tube is actually in the correct location in the lungs. 3. Incorrect: Prior to securing the endo tube to the client's mouth/face, it is more important to verify that tube is in the correct location within the lungs. If the tube is not placed properly, the client will not be adequately ventilator, and the tape would have to be removed for reinsertion. 4. Incorrect: Follow up chest x-rays are needed to verify that the endotracheal tube has been properly placed in the lungs. However, this action is not an immediate priority for the nurse. Assessment is the nursing priority.

The nurse notices that the primary healthcare provider, who has been looking at a client's morning laboratory results, walked away from the computer work station without logging out of the system, leaving the page of client medical information visible on the computer screen. What is the most appropriate action by the nurse? 1. Log the primary healthcare provider off the facility's health information system. 2. Minimize the screen so that the client information is no longer visible, and then ask the primary healthcare provider if the computer can be logged out. 3. Do not interfere since the primary healthcare provider is responsible for this information. 4. Read the health information that the primary healthcare provider left visible on the computer screen to see if the document was completed.

2. Correct: It is appropriate to minimize or hide the screen so that the information is no longer visible, then inquire whether the user will be returning to the computer work station. The nurse is held responsible to ensure that client information on a computer screen remains confidential. Legislation legally protects a client's right to privacy and confidentiality of personal health information. 1. Incorrect: Simply logging the other person off the computer system could be a correct option if that person cannot be found, but it is professional to ask if they will be returning and safeguard client's personal health records. 3. Incorrect: The nurse needs to take action to protect client's confidential health information. Professionally remind the primary healthcare provider that they did not log out and left client information visible to unauthorized persons. 4. Incorrect: It is better to ask primary healthcare provider. The nurse can not accurately determine if the primary healthcare provider is finished.

A client is admitted to the LDR from the emergency department at 34 weeks gestation with profuse, painless, bright red vaginal bleeding. The priority action by the nurse is to prepare for which procedure? 1. Sterile vaginal exam 2. Ultrasound exam 3. Amniocentesis 4. Contraction stress test

2. Correct: Painless, bright red vaginal bleeding is a sign of a placenta previa. Ultrasound can confirm this diagnosis with minimal risk to the mother and her fetus. This is the safest action for this client and best for fixing the problem. 1. Incorrect: If the placenta is over the cervix, a finger can go right through the placenta and cause hemorrhage and fetal death so vaginal exams would be absolutely contraindicated. 3. Incorrect: Amniocentesis is done for genetic analysis or to determine fetal lung maturity when delivery is likely. It is preferable to delay delivery until the fetus is term. It would not be safe to puncture the abdomen of a client that is already hemorrhaging. 4. Incorrect: Contractions can cause further detachment of the placenta from the cervix, which would also cause hemorrhage.

A nurse notes redness, warmth, and pain at a client's intravenous (IV) insertion site. What does the nurse suspect? 1. Colonization 2. Phlebitis 3. Infectious disease 4. Bacteremia

2. Correct: Phlebitis refers to inflammation of a vein and it can be caused by any insult to the blood vessel wall, impaired venous flow, or coagulation abnormality. Clinical evidence includes redness, heat and pain. These signs and symptoms show that the client is experiencing a localized inflammation such as phlebitis. 1. Incorrect: Colonization is used to describe microorganisms present without host interference or interaction. There is an absence of tissue invasion or damage. 3. Incorrect: Infectious disease is the state in which the infected host displays a decline in wellness due to the infection. Clinical signs and symptoms may or may not be present. 4. Incorrect: Bacteremia is determined by presence of bacteria in the bloodstream. Bacteremia can lead to sepsis and signs and symptoms such as fever, hypothermia, tachycardia, tachypnea and inadequate blood flow to internal organs.

The nurse is teaching a client about the use of a cane. Which is the correct cane technique? 1. Place the cane on weaker side of the body to support the weaker leg. Using the cane for support, the client should step forward with strong leg, and then move the weaker leg and cane forward to the strong leg. 2. Place the cane on the stronger side of the body. The cane is placed forward 6 to 10 inches while the client advances the weak leg at the same time. 3. Place cane on weaker side of body. The cane is placed forward 6 to 10 inches while the client advances weaker leg to the cane. 4. Place cane on stronger side of body to help support weaker leg. Using cane for support, step forward with the strong leg and then move the weaker leg and the cane forward to the strong leg.

2. Correct: Place the cane on the stronger side of the body. The cane is placed forward 6 to 10 inches while the client advances the weak leg at the same time. The body weight is divided between the strong leg and the cane. 1. Incorrect: The cane should be on the stronger side of the body to create a wider base for balance as the client advances the strong leg and must use the weaker leg for support with the cane. If the cane is placed on the weaker side of the body, this would create a narrower base for support and balance and increase the risk of falling. 3. Incorrect: The cane should be on the stronger side of the body to create a wider base for balance as the client advances the strong leg and must use the weaker leg for support with the cane. If the cane is placed on the weaker side of the body, this would create a narrower base for support and balance and increase the risk of falling. 4. Incorrect: The cane should be on the stronger side of the body to create a wider base for balance as the client advances the strong leg and must use the weaker leg for support with the cane. If the cane is placed on the weaker side of the body, this would create a narrower base for support and balance and increase the risk of falling.

A client with a deep partial-thickness burn to the right forearm has returned from surgery with a skin graft to the burned area. Which graft site intervention would the nurse implement within the first 24 hours? 1. Monitor temperature every 12 hours. 2. Position arm to prevent pressure to the graft site. 3. Prepare to change the 1st dressing within 24 hours. 4. Perform passive range of motion exercises to the right arm.

2. Correct: The arm should be situated so there is no compression on the graft site. Applying pressure to the graft may cause the graft to move which may result in damage to the graft site. 1. Incorrect: The temperature should be monitored at least every 8 hours or less. An elevated temperature may indicate an infection under the graft or at another area. If the temperature is elevated, the source of the infection should be addressed. 3. Incorrect: The first dressing change is planned 2 to 5 days after surgery. The dressing is not changed in the first 24 hours to prevent any disturbance to the skin graft. 4. Incorrect: The burn is located on the forearm. Passive range of motion exercises are implemented to improve joint function. The client is not having difficulties with the wrist, elbow, or shoulder joints. The movement to the arm should be done cautiously to prevent any damage to the graft.

A nurse wants to find out a better way to perform oral care on unresponsive clients. What is the best first action for the nurse to take in order to achieve this goal? 1. Try different methods of oral care on unresponsive clients to see what works best. 2. Discuss the issue with the leader of the "best practices" committee. 3. Read all the current literature related to oral care on unresponsive clients. 4. Ask the primary healthcare provider to suggest the best oral care procedure.

2. Correct: The best first action for the nurse is to identify a problem, and follow up with the appropriate person. An experienced person who can research "best practice" regarding the issue is needed. The best practice committee works to improve clinical practice based on current research. 1. Incorrect: This is doing research, which requires the research process be implemented, including appropriate approval. The best practice committee utilizes current research in their recommendations. 3. Incorrect: This will take a lot of time and is best initiated from the "best practice" committee. The nurse could definitely be part of the committee. But the evidence-based care leaders are trained to help nurses through the proper process of evidence based research. 4. Incorrect: This is a nursing responsibility and the best practice committee is the best place to begin. The primary healthcare provider may have suggestions but this is not the best first action.

The nurse is caring for a client taking enoxaparin. Which group of symptoms should be reported to the primary healthcare provider? 1. AST of 12 U/L and ALT 20 U/L 2. Hematocrit of 46% decreased to 35% and blood pressure decreases from 122/78 to 108/54 3. Ecchymosis around the abdominal subcutaneous injection site and platelet count of 200,000. 4. Hemoglobin of 14.5 g/dL (2.3 mmol/L) increased to 16 g/dL (2.5 mmol/L) and increased erythema of oral mucus membranes.

2. Correct: These values indicate a drop in hematocrit and drop in blood pressure. Both of these could represent bleeding. These would be important to report to the primary healthcare provider. 1. Incorrect: The nurse would need to watch and report any signs of liver complications due to the use of enoxaparin. The AST and ALT are two liver enzyme values that would increase with liver complications. These two values represent normal AST (8-40 U/L) and ALT (10-30 U/L) values. 3. Incorrect: Bruising (ecchymosis) at the injection site is a frequent occurrence with administration of enoxaparin. This platelet count is within the normal range. 4. Incorrect: The Hgb and color of oral mucous membranes indicate an increase in Hgb. This would not indicate bleeding.

A RN is observing an unlicensed nursing personnel (UAP) feed a client who is on aspiration precautions. Which action by the UAP would require the nurse to intervene? 1. Elevating the head of the bed to a 90 degree angle 2. Instructing the client to lean the head back slightly when swallowing. 3. Adding a thickening agent to liquids. 4. Feeding the client small amounts of food per bite.

2. Correct: This is an incorrect action, and needs intervention by the nurse. The chin should be flexed to prevent the risk of aspiration. A chin down or chin tuck maneuver is widely used in dysphagia treatment to prevent aspiration. 1. Incorrect: This is a correct action. The head of the bed should be elevated which aids in esophageal peristalsis and swallowing is aided by gravity. 3. Incorrect: This is a correct action. Thickened liquids are easier to swallow without aspirating. Drinking liquids thickened will help to prevent choking and stops fluid from entering the lungs. 4. Incorrect: This is a correct action by the UAP, so the nurse does not need to intervene. Smaller amounts of food can be chewed more thoroughly and swallowed with less risk for aspiration.

A client is admitted with a hip fracture after falling. Based on these lab values, what is the nurse's priority nursing intervention? Exhibit Lab Values: Na+ 147 mEq/L (147 mmol/L) Specific Gravity 1.030 Hct 55% 1. Provide foods high in iron 2. Increase fluid intake 3. Obtain a urine for culture 4. Measure intake and output

2. Correct: We already know that the question is about what life threatening complication? A pulmonary embolism. And these lab values say that the client is what? Dehydrated! So the only thing that is going to fix that is....... Increasing fluids. 1. Incorrect: This will not prevent pulmonary embolism. The problem is dehydration. Do something to fix the problem. Foods high in iron will not fix the problem. 3. Incorrect: This will not prevent pulmonary embolism. How will obtaining a urine sample for culture fix dehydration? It won't. This client needs to increase fluid intake. 4. Incorrect: We do want to monitor intake and output to see how the client is doing, however, this will not fix the problem. Hydrating the client will help the problem.

An elderly client diagnosed with terminal cancer is the sole caregiver to a developmentally delayed adult child. The client is worried that the child, with a developmental age of seven years old, will need permanent placement in a long term care facility. What statement by the nurse is most accurate? 1. "Your child will need to be under constant supervision." 2. "A supervised group home would be an ideal setting." 3. "Maybe we could find someone to take in your child." 4. "We should start getting the child used to living alone."

2. Correct: With a developmental age of seven years old, group home supervision would be ideal. The adult child can complete most activities of daily living and will only need minimal assistance with such tasks as cooking, laundry or shopping. 1. Incorrect: This comment is inaccurate, based on Erikson's stages. Even a seven year old can manage most ADLs without assistance, such as bathing, dressing, and grooming. Constant supervision would not be necessary. 3. Incorrect: Take in implies the child would need a private individual to provide care round the clock in a home, which is not necessary for this individual's developmental age. There are many activities the adult child can complete without supervision, so private home placement is not needed. 4. Incorrect: An adult with a mental age of seven is not capable of living completely alone. While able to complete ADLs and many small tasks, this individual would not be able to be live independently.

The nurse's assessment of a client post-op abdominoplasty reveals tachycardia, restlessness and shallow slow breaths. The client was medicated with morphine 2 mg IVP one hour ago. The primary healthcare provider prescribes arterial blood gases (ABG). Which ABG report is consistent with this clinical picture? 1. pH 7.30, PaCO2 40, HCO3 29 2. pH 7.33, PaCO2 48, HCO3 25 3. pH 7.47, PaCO2 35, HCO3 29 4. pH 7.50, PaCO2 33, HCO3 22

2. Correct: You may only know abdomino-abdomen. That's ok. Plasty; well some kind of plastic surgery on the abdomen. Focus on what you do know, look at the scenario. See the hints, abdominal surgery, restless, shallow breaths, requested pain med.... We already know that we SHOULD be watching for respiratory acidosis so what lab values are consistent with this imbalance. Low pH, high CO2. 1. Incorrect: The pH is acidosis, but it is not respiratory acidosis the pC02 is normal. 3. Incorrect: This client is in metabolic alkalosis. 4. Incorrect: This client is in respiratory alkalosis.

The nurse is caring for a client with multiple episodes of diarrhea and suspected Clostridium Difficile (C. diff). Which interventions should be included in the plan of care? Select all that apply 1. Institute contact precautions only after confirmation of stool culture. 2. Instituting contact precautions for all who enter the client's room 3. Using alcohol based foam for hand hygiene. 4. Dedicating equipment for use only in the client's room. 5. Requesting antidiarrheal medication for the client.

2., & 4. Correct: Contact isolation will be needed to prevent the spread of infection. Also the electronic equipment for vital signs must not be used in the room. The client will need a disposable stethoscope, BP cuff and thermometer dedicated for use in that patient room. 1. Incorrect: Precautions should be instituted and a stool sample sent for any client with persistent diarrhea. Isolation should be in place with suspected c. diff. 3. Incorrect: Soap and water must be used to clean the hands. Alcohol based foams do not have enough alcohol in them to destroy the c diff spores. 5. Incorrect: Medications to stop diarrhea will not be prescribed with c. diff. because they cause even further irritation.

A client has been admitted with multiple severe allergies, including food and medications. The nurse knows what actions are most important to protect the client? Select all that apply 1. Assign client to a private, sterile room. 2. Place allergy alert bracelet on client. 3. Have client wear mask when in hallway. 4. Attach sign listing allergies above the bed. 5. Send list of allergies to dietary department.

2., & 5. CORRECT: It is crucial to place the facility's allergy alert bracelet on the client upon admission, generally on the same wrist as the facility ID bracelet. Each time the client's ID is verified, staff will also see the allergy bracelet. Dietary department must also be alerted of all allergies in writing as should the pharmacy. In most facilities allergy alert stickers are attached on the outside of the chart, on the medication sheet, and facility-specific areas to remind the staff caring for the client. 1. INCORRECT: A client with allergies does not have to be placed into a private room, even if the allergies are environmental. There is no such thing as a "sterile" client room, which implies a sealed location with reverse airflow system. Although operating rooms try to maintain a sterile atmosphere, the entire room itself is not sterile. 3. INCORRECT: Allergies would not require the client to wear a mask in the hallway, or when transported to another department. Even if a client allergy is airborne, such as dust, a mask does not provide significant protection. 4. INCORRECT: A sign listing exact client allergies would violate HIPAA regulations and privacy policies. In severe cases, a facility may choose to place a plain allergy sticker in the client's room, but cannot place any specific, identifying client information in public view.

The nurse is teaching a group of teenagers about decreasing the risk of developing skin cancer. What information should the nurse include? Select all that apply 1. Use sunscreen with a sun protection factor (SPF) of at least 30. 2. A self-tanning product containing dihydroxyacetone (DHA) is safe to use. 3. Put on sunscreen every day, even on days when it is cloudy. 4. Stay in the shade between 9 AM and 4 PM. 5. Tanning beds are safer than outdoor tanning.

2., 3., & 4. Correct: People of all ages should avoid tanning. If you like the look of tanned skin, choose a self-tanning product that contains the active ingredient DHA. This active ingredient has been approved by the U.S. Food and Drug Administration and is safe to use. Most are fast-acting and will give skin a darker appearance in a matter of a few hours. This "tan" will last about a week. Put on sunscreen every day, even on days when it is cloudy. UV rays from the sun can reach you on cloudy and hazy days, as well as bright and sunny days. UV rays also reflect off surfaces like water, cement, sand, and snow. The hours between 10 a.m. and 4 p.m. Daylight Saving Time (9 a.m. to 3 p.m. standard time) are the most hazardous for UV exposure outdoors in the continental United States. UV rays from sunlight are the greatest during the late spring and early summer in North America. 1. Incorrect: Use sunscreen with a SPF of 15 or higher, and both UVA and UVB (broad spectrum) protection. 5. Incorrect: Tanning beds are dangerous. Avoiding the sun but replacing it with a tanning bed does not reduce the risks that are associated with UV damage to the skin.

A nurse is planning to teach a group who works at a local mall about proper use of automated external defibrillators (AED). Which points should the nurse emphasize? Select all that apply 1. The standard AED can be used on children over the age of 5. 2. All users of the AED must be trained in its operation. 3. CPR should be taught to users. 4. Primary healthcare provider oversight is needed to ensure proper maintenance. 5. The local EMS should be notified of the type and location of AEDs.

2., 3., 4. & 5. Correct: These are appropriate actions. 1. Incorrect: Standard AEDs can be used on children over the age of 8. For children ages 1-8, the AHA recommends the pediatric attenuated pads that are purchased separately.

What electrolyte imbalance should the nurse monitor for when caring for a client diagnosed with chronic alcoholism? Select all that apply 1. Hypochloremia 2. Hypokalemia 3. Hypophosphatemia 4. Hypomagnesemia 5. Hypocalcemia

2., 3., 4., & 5. Correct: The number one way of getting rid of potassium is through the kidneys. What does alcohol make you do? Diuresis. Acute hypophosphatemia is seen in up to 50% of patients over the first 2-3 days after they are hospitalized for alcohol overuse. Hypophosphatemia is manifested as rhabdomyolysis (muscle breakdown) and weakness of the skeletal muscles. Magnesium deficiency occurs due to that increase in diuresis as well. Hypomagnesemia is often accompanied by hypocalcemia, or lowered calcium levels, which may be aggravated by a deficiency of vitamin D. 1. Incorrect: Hypochloremia is usually caused by excess use of loop diuretics, nasogastric suction, vomiting or diarrhea due to small bowel abnormalities, and loss of fluids through the skin occurring because of trauma such as burns.

A nurse is participating in a cancer risk screening program. Which signs/symptoms would indicate to the nurse that a client needs further investigation? Select all that apply 1. Unexplained weight gain of 10 pounds 2. Leukoplakia 3. Prolonged hoarseness 4. Hematuria 5. Persistent abdominal bloating

2., 3., 4., & 5. Correct: White patches inside the mouth or white spots on the tongue may be leukoplakia, which is a precancerous area that is caused by frequent irritation. It is often caused by smoking or other tobacco use. People who smoke pipes or use oral or spit tobacco are at high risk for leukoplakia. If untreated, it can become mouth cancer. A cough that does not go away and prolonged hoarseness may be a sign of cancer. Hematuria may be a sign of bladder or kidney cancer and needs further investigation. Although women may experience bloating with changes in the menstrual cycle, constant bloating should be investigated to rule out ovarian cancer. 1. Incorrect: Unexplained loss of weight or loss of appetite may indicate some types of cancer. Weight gain is not typically associated with cancer.

The nurse is preparing to administer a dose of potassium iodide 300 mg by mouth to a client diagnosed with hyperthyroidism. The nurse has not administered this medication before and is using a drug reference to review information about the medication. Which client and drug reference information supports the nurse's decision to hold the potassium iodide dose and notify the primary healthcare provider? Exhibit: Client Information: Medical diagnosis: Hyperthyroidism Current vital signs: BP 142/88, HR 102, R 20 Medical history: Hypertension Physical examination: Alert/oriented. PERRLA. Skin warm/dry. Lungs sounds clear bilaterally. Normal S1/S2 without murmurs, clicks, rubs. Lab test results: Glucose- 98 mg/dl (5.4 mmol/L), Sodium- 139 mEq/L (139 mmol/L), Potassium- 5.5 mEq/L (5.5 mmol/L), Creatinine - 0.9 mg/dL (79.5 µmol/L), Creatinine Clearance 110 mL/min Current medications: Losartan 50 mg one by mouth daily, Methimazole 10 mg by mouth daily Drug Reference: Medication: Potassium iodide Classification: Antithyroid agent Indications: Adjunct with other antithyroid drugs in preparation for thyroidectomy. Treatment in thyrotoxic crisis. Radiation protectant following radiation emergencies or administration of radioactive iodine. Contraindications/Precautions: Hypersensitivity; hyperkalemia; pulmonary edema; impaired renal function. Use cautiously in tuberculosis; bronchitis; cardiovascular disease. Adverse reactions/Side effects: Confusion, weakness, GI BLEEDING, diarrhea, nausea, vomiting, hyperkalemia, tingling, joint pain. Interactions: Use with lithium may cause increased hypothyroidism. Increases the antithyroid effects of methimazole and propylthiouracil. Increased hyperkalemia may result from combined use with potassium-sparing diuretics, Ace inhibitors, angiotensin II receptor antagonists or potassium supplements. Route/Dose: 300-500 mg three times a day by mouth Select all that apply 1. Creatinine - 0.9 mg/dL (79.5 µmol/L) 2. Potassium- 5.5 mEq/L (5.5 mmol/L) 3. Glucose- 98 mg/dl (5.4 mmol/L) 4. Taking losartan 50 mg one by mouth daily. 5. Currently taking methimazole 10 mg by mouth daily. 6. Creatinine Clearance 110 mL/min

2., 4., & 5. Correct: The medication is potassium iodide, which can lead to hyperkalemia when administered, so it is contraindicated if the client already has hyperkalemia. This client's potassium level is 5.5 mEq/L (5.5 mmol/L), which would support the nurse holding the medication and contacting the primary healthcare provider. Additionally, the drug guides states that potassium iodide increases the antithyroid effect of methimazole and propylthiouracil. Increased hyperkalemia may result from combined use with potassium-sparing diuretics, Ace inhibitors, angiotensin II receptor antagonists or potassium supplements. This client is currently on both losartan, an ARB, and methimazole. 1. Incorrect: This is a normal creatinine level. Normal range is 0.8 - 1.4 mg/dL (70-124 µmol/L) in males and 0.56-1.0 mg/dL (50-88 µmol/L) in females. 3. Incorrect: Potassium iodide does not affect glucose and this is a normal glucose level. 6. Incorrect: The normal creatinine clearance is 75-125 mL/min. Therefore, 110 mL/min is within normal limits and would not require withholding the potassium iodide.

A client reports dizziness and weakness while walking down the hall. The nurse notes the client's cardiac rhythm displayed on the telemetry monitor. What actions should the nurse take? Exhibit: Select all that apply 1. Have client ambulate back to bed. 2. Initiate 100% oxygen per non rebreather mask. 3. Obtain client's blood pressure. 4. Prepare for cardioversion. 5. Auscultate lung sounds. 6. Administer nitroglycerin 1 tab SL.

3. & 5. Correct: The client is dizzy and weak. This client is at risk for falling, so think safety and get the client back in bed. Use a wheelchair to accomplish this. Then obtain the client's BP. It may be low indicating poor tissue perfusion to the vital organs. One cause of premature ventricular contractions (PVCs) includes heart failure, so assess the lungs for adventitious sounds. 1. Incorrect: This client is dizzy and weak. Having the client ambulate back to the bed is a safety risk. The client could fall. 2. Incorrect: Oxygen may abate the PVCs; however, it should be initiated at 2 liters/NC rather than at 100%. Start with the least amount of oxygen that could relieve symptoms. 4. Incorrect: Cardioversion is not indicated with an underlying rhythm that is normal (NSR) with PVCs. Oxygen may decrease the PVCs. If not, medication can be administered to decrease the rate of the PVCs. 6. Incorrect: Nitroglycerin would be given if the client is experiencing chest pain or is suspected of having an MI. Get the client back in bed and provide the client with oxygen at 2 L/NC first.

A pregnant client's initial blood work shows a negative rubella titer. The nurse is aware this result indicates what important course of action? 1. Client needs to be isolated until delivery. 2. Client is immune to rubella currently. 3. Client should be given rubella vaccine after delivery. 4. Client has never been exposed to rubella.

3. CORRECT: A negative titer indicates the client has no rubella antibodies present currently. But because the rubella vaccine contains a live virus, the client cannot be safely vaccinated until after delivery. 1. INCORRECT: Although the client may be cautioned about being around groups of children until after delivery, there is no need for total isolation for the duration of the pregnancy. 2. INCORRECT: If the client were immune to rubella, the titer would have been positive, indicating the presence of rubella antibodies. This client is not immune currently. 4. INCORRECT: Whether the client has ever been exposed to rubella cannot be determined from the information presented in this question.

The nurse in the pediatric intensive care unit (PICU) is caring for a preschool child three days after open heart surgery. What assessment finding should the nurse report immediately to the primary healthcare provider? 1. Increased episodes of fussy crying. 2. A hacking, non-productive cough. 3. Oral temperature of 100.9°F (38.3°C). 4. Chest tube draining 30 mL per shift.

3. CORRECT: An oral temperature of 100.9°F (38.3°C) is considered too elevated for 3 days post-op. An oral body temperature greater than 100.5°F (38.1°C) indicates the potential for infection. Although no other vital signs are given in the scenario, a temperature this elevated would need to be reported immediately by the nurse to the primary healthcare provider. 1. INCORRECT: Increasing episodes of crying could indicate many things in a preschool child, including pain, fear, loneliness, or even elevated body temperature. While this change in the client's status will need to be investigated further, the nurse would not need to report this behavior at this time. 2. INCORRECT: A hacking, non-productive cough, even several days after open heart surgery, could be attributed to the effects of intubation, anesthesia, or even certain cardiac medications. Clients are always encouraged to cough and deep breathe in order to prevent pulmonary complications. If the cough becomes productive or breathing becomes labored, the nurse would need to report this to the primary healthcare provider. This is not an urgent concern for the nurse. 4. INCORRECT: Chest tube drainage is common following open-heart surgery, even three days later. It is impossible to evaluate whether 30 mL in one shift is a change since there are no parameters to compare the previous shift's output. The nurse would not need to report this drainage at this time.

A client being discharged home following hip surgery is prescribed to use a walker. While observing the client walk across the room, the nurse is most concerned when the client does what? 1. Applies shoes securely before ambulating with walker. 2. Checks walker to be certain the legs are securely locked. 3. Slides walker slowly forward when walking across the room. 4. Places walker to right of the chair after sitting down in chair.

3. CORRECT: The nurse is observing the client ambulate with a walker prior to discharge, to determine whether the client is using the assistive device safely. The nurse becomes concerned upon noting the client sliding the walker during ambulating. The correct use of a walker involves the client lifting and placing the walker approximately one-foot length ahead, then stepping into the non-moving walker. It is important for the walker to remain stationary when the client takes a step forward. 1. INCORRECT: This action by the client is appropriate. Proper, gripping footwear should be worn by the client at all times when ambulating. This prevents the possibility of slipping and falling. There is no cause for concern with this action. 2. INCORRECT: Another smart move is to verify the cross bars are securely locked before ambulating. When a walker is folded for storage, the locks are unlatched. When the walker is open, the locks must click into place to verify the device is safe for ambulating. No concerns here. 4. INCORRECT: When a client sits down, the walker can be placed to either side of the chair. The most important factor is for the client to use the walker to safely maneuver into the chair rather than placing the walker aside before sitting down. Placing the walker next to the chair after being seated is appropriate.

Which task would be appropriate for the Labor, Delivery, Recovery, Postpartum (LDRP) charge nurse to assign to an LPN/LVN? 1. Administering IV pain medication to a client three days postoperative cesarean section. 2. Drawing a trough vancomycin level on a client 3 days postpartum with bilateral mastitis. 3. Reinforce how to perform perineal care to a primipara who is four hours postpartum. 4. Drawing routine admission labs on a client admitted in final stages of labor.

3. Correct: Client teaching may be reinforced by an LPN/LVN on a stable client. 1. Incorrect: Administering IV pain medications is out of the scope of practice of LPN/LVN. 2. Incorrect: Drawing lab work on a client with severe infection and only 3 days postpartum is an unstable client and needs care from the RN. 4. Incorrect: Drawing routine admission labs on a client in final stages of labor would be inappropriate because the client is potentially unstable and needs experienced LDRP nursing care.

A 70 year old client was admitted to the unit during the night shift with chronic hypertension. At 0830, the unlicensed nursing assistant (UAP) reports that the client's blood pressure is 198/94 mm Hg. What would be the best action for the charge nurse to delegate at this time? 1. Ask the UAP to put the client back in bed immediately. 2. Tell the UAP to take the BP in the opposite arm in 15 minutes. 3. Have the LPN/LVN administer the 0900 furosemide and enalapril now. 4. Ask the LPN/LVN to assess the client for pain.

3. Correct. Administering the client's blood pressure medicines are aimed at correcting the hypertension. The therapeutic action of furosemide is diuresis which will lower the blood pressure. Enalapril is an angiotensin converting enzyme (ACE) that treats hypertension. These medications can be administered within 30 minutes of 0900. 1. Incorrect. Assisting the client back to bed is appropriate, but does not address the problem of lowering the client's BP. Administration of furosemide and enalapril will benefit the client with hypertension. 2. Incorrect. Retaking the BP in the opposite arm is within the scope of practice of an UAP, but does not address the problem of lowering the client's blood pressure. Additionally this should be completed prior to 15 minutes time. The priority is to get the BP down by giving the prescribed medications for hypertension. 4. Incorrect. The LPN can ask the client if they have chest pain. The client does have a BP of 198/94 which could lead to chest pain. The priority is to get the BP down to decrease the risk of complications associated with hypertension, such as MI, and stroke.

The nurse on a large surgical unit needs to evaluate several clients returning from procedures. Which client should the nurse assess first? 1. Lumbar puncture reporting a headache. 2. Cystogram reporting burning on urination. 3. Thoracentesis reporting shortness of breath. 4. Cardiac catheterization with a decreased pedal pulse below insertion site.

3. Correct: A thoracentesis is performed to remove fluid from the pleural cavity and improve the client's respiratory status. This client should report an improved respiratory, not shortness of breath. The worst complication following a thoracentesis is a possible pneumothorax; therefore, the nurse should assess this client first. 1. Incorrect: A lumbar puncture involves removing cerebrospinal fluid from the subarachnoid space to diagnose specific diseases or the presence of bacteria. Headache following this procedure is a potential side effect and would not be the priority concern for the nurse. 2. Incorrect: The purpose of a cystogram is to examine the inside of the bladder to confirm the presence or absence of abnormalities, or even obtain a biopsy. Because a scope is inserted through the urethra for this procedure, the client may experience burning or frequency immediately following this test. Although this will require assessment, this client is not the priority at this time. 4. Incorrect: A slightly decreased pedal pulse to the affected extremity is not unusual following cardiac catheterization. This invasive procedure results in some edema to the vessel used for the procedure but assessing only one pedal pulse does not provide sufficient data to verify a complication.

An LPN/VN has been floated to the emergency room following a chemical plant explosion. What task would be best to assign to the LPN/LVN? 1. Identify and assess each incoming client. 2. Triage and assign color-coded tags to each client. 3. Gather and apply dressings to open wounds. 4. Initiate oxygen and IV lines as needed.

3. Correct: An LPN/LVN's scope of practice includes tasks such as wound care. Covering open wounds will help to decrease bacterial exposure until the registered nurse or primary healthcare provider can assess and treat each wound. If the LPN notes any serious bleeding situations, it would need reported immediately to the RN. 1. Incorrect: Although it will be crucial to identify each incoming client, the LPN/LVN's scope of practice does not include assessment. That task would require an RN or primary healthcare provider. 2. Incorrect: In a mass casualty situation, triage allows the nurse or primary healthcare provider to quickly determine which clients are critical versus those stable enough to wait. Because this involves assessment, an LPN/LVN would not be assigned this task. 4. Incorrect: Initiating intravenous lines is not within the scope of the LPN/LVN. Additionally, the decision to apply oxygen involves assessment of the respiratory system, which also is not within the LPN/LVN's scope of practice.

A concerned mother is asking the nurse about activities that would be best for her child who has been diagnosed with asthma. In order to minimize the risk of exercise induced asthma, which activity would be best for the nurse to suggest? 1. Track 2. Basketball 3. Baseball 4. Soccer

3. Correct: Baseball is an activity that is considered "asthma friendly". It requires short, intermittent periods of exertion and is therefore tolerated better by children with asthma. 1. Incorrect: This activity requires extended periods of exertion and is not considered "asthma friendly" and often not tolerated by clients with asthma. 2. Incorrect: This activity requires extended periods of exertion and is not considered "asthma friendly" and often not tolerated by clients with asthma. 4. Incorrect: This activity requires extended periods of exertion and is not considered "asthma friendly" and often not tolerated by clients with asthma.

A client is sedated. His wife asks the nurse about her husband's test results. The client does not have a healthcare proxy or durable power of attorney executed at this time. How should the nurse respond in compliance with HIPAA (Health Insurance Portability and Accountability Act) regulations regarding the confidentiality of the sedated client's health information? 1. I can't give you those results. You should ask his primary healthcare provider the next time that he comes in to examine your husband. 2. Those test results are confidential, but since you are his wife I can give them to you. Let me look them up in the computer system. 3. The health information of all clients is confidential and is protected by law. Those test results cannot be released without the consent of the client in order to protect the client's right to choose who receives health information. 4. Your husband is only lightly sedated. I can wake him up and ask him if it is all right to release these test results to you.

3. Correct: Each client's health information is confidential and protected by law. The nurse should inform the client's wife of this fact, and explain the rationale for health information confidentiality. Family members are often offended or angry upon learning that health information cannot be released to them without the client's consent , but healthcare employees are bound by law to confidentiality. 1. Incorrect: The wife is not automatically able to receive personal health information about her husband.The husband has to list the wife as a person who can receive personal health information. The Healthcare employees, including primary healthcare providers, are bound by law to keep health information confidential without the client's consent. 2. Incorrect: Healthcare employees, including primary healthcare providers, are bound by law to keep health information confidential without the client's consent. 4. Incorrect: A client who has received sedative medications cannot give legal consent, as these medications alter a client's level of consciousness and impair the ability to make informed decisions.

A client is brought to the emergency room following a serious motor vehicle accident. Standing orders include initiating an IV line and inserting a foley catheter. What action should the nurse take first? 1. Obtain all supplies for the procedures. 2. Explain the procedure to the client. 3. Check the client's identification band. 4. Verify client has signed consent forms.

3. Correct: Even in an emergency, the nurse follows the nursing process by initially gathering data, including identifying the client before beginning any ordered interventions. The client's identity must always be verified before any procedure or treatment. 1. Incorrect: It is important to make sure all necessary supplies are present before beginning an intervention. Stopping in the middle of a procedure to get supplies could expose the client to infection or other complications. However, gathering supplies is an action, which is not the first step when providing care to any client. 2. Incorrect: While it is important to explain any procedure to the client, the scenario does not indicate if this client is even conscious. The nurse has another important priority. 4. Incorrect: When stabilizing an injured client, consent is implied for life-saving procedures such as initiating an IV or applying oxygen. Additionally, obtaining or verifying consent is not a nursing responsibility.

The charge nurse on the postpartum unit is making assignments. Report from the night shift nurse for one client included the recent development of the following findings: BP 150/100, proteinuria, severe headache, blurred vision, and abdominal pain. Which nurse should be assigned to care for this client? 1. The RN with 8 years' experience in the Intensive Care Unit. 2. The RN with 10 years' experience pulled from the ER. 3. The RN with 5 years' experience in the Labor and Delivery unit. 4. The RN with 2 weeks' experience on the postpartum unit.

3. Correct: First, you must recognize that this client has the signs and symptoms of postpartum preeclampsia. The RN who has worked in Labor and Delivery would have knowledge and experience caring for clients with preeclampsia. This client needs careful monitoring and specialized care. Therefore, the nurse with Labor and Delivery experience would be the most appropriate one to assign to care for this client who has postpartum preeclampsia. 1. Incorrect: Although this nurse may be accustomed to caring for clients in acute situations requiring a higher level of care, this nurse is not familiar with caring for clients with preeclampsia. Therefore, the nurse with the labor and delivery experience would be more appropriate to assign to this client. 2. Incorrect: Emergency room nurses deal with life threatening emergencies but are not specialized in the care of clients with preeclampsia. Therefore, this would not be the most appropriate nurse to assign to this client. 4. Incorrect: Although this nurse is working on the postpartum unit, did you recognize the length of experience? This nurse does not have much experience on this unit and may not have cared for a client with postpartum preeclampsia before. This situation needs advanced monitoring and care, so this nurse with very little postpartum experience would not be the most appropriate to assign to this client.

A nurse is caring for client with a left above the knee amputation 48 hours postop. The client is experiencing left lower leg pain on a scale of 6 out of 10. Which pain relief intervention would the nurse implement? 1. Position the client in a supine position. 2. Rewrap the ace bandage on the stump. 3. Instruct the client in guided imagery techniques. 4. Initiate range of motion exercises to the knee.

3. Correct: Phantom limb pain (PLP) may be experienced in the amputated part after surgery. The client may describe the PLP as crushing, cramping, and burning. Complementary therapy is a non-pharmacological comfort measure that can be utilized to reduce the client's PLP. Instructing the client to implement guided imagery techniques will assist the client in reducing PLP. 1. Incorrect: Placing the client in various positions in bed by the nurse will not reduce the client's PLP. The client's PLP can be addressed with complementary therapy and medications such as calcitonin, beta-blockers, antiepileptics, antispasmodics or antidepressant medications. 2 Incorrect: Rewrapping the ace bandage on the stump by the nurse will not reduce the phantom limb pain. Wrapping the stump will decrease edema, secure the dressing, and assist in shrinking the limb. 4. Incorrect: Range of motion exercises will decrease the possibility of flexion contractures of the hip and knee. The improved flexion of the hip and knee with range of motion will not decrease PLP.

Which activity by the unlicensed assistive personnel (UAP) assisting a client with Parkinson's disease would require intervention by the nurse? 1. Assisting the client with ambulating to the bathroom and back to bed 2. Reminding the client not to look down while walking 3. Performing bathing and oral care for the client 4. Encouraging the client to feed self

3. Correct: The UAP should encourage the client to be as independent as possible. The nurse should intervene and teach UAP about client performing as much care as possible to encourage independence. 1. Incorrect: The UAP should assist the client when ambulating. This would not require intervention. 2. Incorrect: The UAP should remind the client to watch the horizon and not look down. This would not require intervention. 4. Incorrect: The UAP is encouraging independence. This is appropriate intervention and would not require intervention.

The nurse manager is making rounds in a long-term care facility and discovers an unfamiliar client standing in the hallway in a puddle of liquid. What is the nurse manager's priority action? 1. Ask client to state name and room number. 2. Find dry clothes and clean client completely. 3. Wipe up puddle of liquid and call housekeeping. 4. Contact unit charge nurse to identify the client.

3. Correct: The issue in this scenario is safety. Whether the nurse manager knows this client is not important at the moment. Liquid on the floor poses a safety hazard for clients, visitors, and staff. The priority action is to remove this risk immediately before an injury occurs. 1. Incorrect: Even though the nurse does not recognize this particular client, the priority concern is not focused on identifying this individual. 2. Incorrect: While the client will certainly need clean, dry clothes, the most immediate concern is a safety issue. The nurse can summon a UAP to help the client after addressing the safety issue. 4. Incorrect: The identity of the client is not the initial priority at this time. Consider the whole picture when thinking about safety.

What action should the nurse take when testing a client's near vision? 1. Have client read a Snellen chart from 20 feet away. 2. Have client read Ishihara plates at 30 inches (75 cm). 3. Have client read a newspaper at 14 inches (36 cm). 4. Have client alternate gaze from a near object to a distant object.

3. Correct: The nurse can get a general idea of near visual acuity by asking the client to read from a newspaper. The newspaper should be held 14 inches from the eyes. This exam can also be done with the Jaeger chart containing a few short lines or paragraphs of printed text. The size of the print gradually gets smaller. The client is asked to hold the card about 14 in. (36 cm) from the face and read aloud the paragraph containing the smallest print he/she can comfortably read. Both eyes are tested together, with and without corrective lenses. This test is routinely done after age 40, because near vision tends to decline as one ages (presbyopia). 1. Incorrect: The Snellen chart is used to test distant vision. To test distance vision, individuals stand 20 feet from the Snellen eye chart, cover one eye, read aloud the smallest line they can clearly see, and then repeat this process with the other eye. After performing an eye test, a person's visual acuity is written as a fraction. Normal vision is defined as 20/20 visual acuity, which means at 20 feet away from the eye chart, the person is able to read the line that most human beings with normal vision can read at 20 feet away.​ 2. Incorrect: Ishihara plates are used to assess color blindness, and are not used to test near vision. 4. Incorrect: Having the client alternate their gaze from a near object to a distant object tests for accommodation. It does not test for near vision.

A nurse working on the pediatric oncology unit is beginning the shift and has received report which included some new laboratory data for the clients. Based on the information provided in report, which client condition should be the nurse's priority? 1. Potassium level of 3.4 mEq/L (3.4 mmol/L) in a child with vomiting and diarrhea. 2. Platelet count of 95,000/mm3 in a child with a nose bleed. 3. Absolute neutrophil count of 400/mm3 in a child with fever. 4. Hemoglobin level of 9 g/dL (90 g/L) in a child with reports of fatigue.

3. Correct: The nurse should recognize that this child has a very low absolute neutrophil count (ANC), which is referred to a neutropenia. This client is at a high risk of infection. We see that the temperature is already elevated, which makes us worry that infection is present. Therefore, measures should be instituted to reduce the risk of the development of an overwhelming infection and sepsis. This client would be the priority based on the need for prompt recognition and treatment of the neutropenia and signs of infection present. 1. Incorrect: Although the potassium level of 3.4 mEq/L (3.4 mmol/L) is slightly decreased, this level can be corrected and should improve when the vomiting and diarrhea subside. The nurse should continue to monitor the potassium level, but it does not take priority over the extremely low ANC in the child with fever. 2. Incorrect: This platelet level of 95,000/mm3 is below the normal range of 150,000/mm3 to 400,000/mm3. When the level gets below 100,000/mm3, the clients should be monitored for bleeding such as a nose bleed, which this client has. However, nose bleeds are not that uncommon and can often be controlled by applying pressure to the nares for 5 to 10 minutes. We would not expect to see severe hemorrhage until the levels are much lower, so this client would not be a priority over the client with the low ANC with fever. 4. Incorrect: This hemoglobin level of 9 g/dL (90 g/L) in a child who has reported fatigue is below the normal of 11-15 g/dL (110-150 g/L). However, the fatigue can be managed by regulating the activity to conserve oxygen expenditure and prevent fatigue. The child with the low absolute neutrophil count with signs of an advancing infection would take priority over this child with a slightly low hemoglobin.

Which nursing intervention will be most helpful to a middle-aged client experiencing insomnia? 1. Instruct the client to initiate an exercise routine during the day. 2. Educate the client on ways to adjust the sleep environment. 3. Instruct the client on progressive relaxation techniques to be used just before bedtime. 4. Instruct the client to decrease caffeine intake.

3. Correct: The use of progressive relaxation techniques would help the client relax before bedtime and promote sleep. 1. Incorrect: Exercise might be helpful, but not most helpful. 2. Incorrect: This intervention is not specific. 4. Incorrect: Instructing the client to decrease caffeine intake may help the client but not specific enough.

A newly hired unlicensed assistive personnel (UAP) at a long-term care facility is being instructed on the proper method of feeding a stroke client with dysphagia. The nurse knows teaching was successful when the UAP makes what statement? 1. "Feeding the client in semi-fowler's position is easier." 2. "I should not allow the client to do any self-feeding." 3. "Thickened liquids are safer for the client to swallow." 4. "I am offering the client a drink after each bite to help digestion."

3. Correct: Thickened liquids allow for easier swallowing and less choking, thus decreasing the chance of aspiration. The client should be sitting upright and fed small amounts of food slowly, allowing time for chewing and swallowing. This statement indicates the UAP understands proper feeding protocols. 1. Incorrect: Semi-fowler's is a "semi-reclining" position, which would greatly increase the risk of aspiration during meals. This comment indicates the UAP would need further instruction. 2. Incorrect: It is crucial to encourage a stroke client to participate as much as possible in self-care, including feeding and bathing. If this client is capable of using utensils, such as modified silverware, it is important to allow as much participation in activities of daily living (ADL) as possible. If the UAP made this comment, further teaching is indicated. 4. Incorrect: Liquids after every bite would quickly fill up the client, decreasing the amount of food intake. Feeding slowly and allowing the client time to swallow after each bite is sufficient for digestion. Such a statement from the UAP means further instruction is needed.

The nurse is preparing to educate a client about human papillomavirus (HPV). What information should the nurse include? 1. There is no vaccine to prevent HPV. 2. HPV is the cause of most ovarian cancers. 3. The only way to prevent HPV is refraining from any genital contact with another. 4. HPV is cured by removal of genital warts.

3. Correct: This is a true statement. Latex condoms have been associated with lower risk, however, there is still a risk of coming into contact with the virus even when a condom is used correctly. 1. Incorrect: There is a vaccine against the human papillomavirus. 2. Incorrect: HPV is the cause of most cervical cancers. 4. Incorrect: Even after genital warts are removed, HPV remains, and viral shedding will continue.

The nurse is evaluating care provided by an unlicensed assistive personnel (UAP). Which action should the nurse interrupt the UAP from performing? 1. Draining the colostomy bag on a client with diarrhea. 2. Performing passive range of motion (ROM) on the client with right sided paralysis. 3. Placing the traction weights on the bed to transfer the client to x-ray. 4. Discarding the first urine voided by the client starting a 24 hour urine test.

3. Correct: Traction should never be relieved without a primary healthcare provider's prescription. It can result in muscle spasm and tissue damage. This client could be transferred with traction still maintained. 1. Incorrect: A colostomy client with diarrhea will have a lot of drainage requiring frequent emptying of the colostomy bag. Draining of the bag is a routine toileting procedure for the colostomy client and.is within the scope of practice for the UAP. 2. Incorrect: Passive ROM is performed with paralysis and can be delegated to the UAP. Each ROM movement should be repeated 5 times during the session. 4. Incorrect: The first void of a 24 hour urine is discarded and can be delegated to the UAP. The nurse would then start the 24 hour urine once the 1st void has been discarded. The nurse also needs to be aware of the color and amount of urine voided.

The nurse is performing sterile wound care for partial thickness burns on a client's lower right leg. Prior to initiating this procedure, what action should the nurse complete first? 1. Position client upright with right leg elevated. 2. Obtain wound culture before cleaning wound. 3. Assess current pain level and medicate. 4. Encourage client to verbalize concerns.

3. Correct: Wound care on burns is a painful process, particularly with partial thickness burns (formerly referred to as second degree) because nerve endings are intact and exposed. Pre-medicating is a priority action, since pain medication can take up to 30 minutes to activate within the body. Clients are more cooperative and heal faster when pain is well controlled. 1. Incorrect: Proper visualization during wound care is vital, as is client comfort during the procedure. However, completion of this process does not require the client to be in an upright position. In fact, that may be counter productive at this time. Additionally, whether the right leg needs elevated depends on the size or location of the burn on the right leg, and that information has not been provided in the question. 2. Incorrect: While it is true that any wound culture must be obtained prior to cleaning the affected area, this action is not presently the nurse's first priority. Consider the nursing process and choose another option. 4. Incorrect: Therapeutic communication is an ongoing process during any client interaction, particularly when the nurse needs to explain an upcoming procedure. Allowing the client to express fears, verbalize concerns or ask questions enhances cooperation. Although this exchange of information is occurring throughout this period of time, the nurse has another priority action that should be completed first.

The nurse is reviewing the plan of care for a client during the first day post-craniotomy. Which actions can the nurse delegate to an experienced LPN/LVN working in the ICU? Select all that apply 1. Determine Glasgow Coma Score. 2. Check endotracheal tube (ET) cuff pressure every shift. 3. Reposition client from side to side every 2 hours. 4. Administer acetaminophen via nasogastric tube for temperature greater than 101ºF (38.3ºC). 5. Monitor intake and output every hour.

4., & 5. Correct: Both of these actions are within the scope of practice for the LPN/LVN. 1. Incorrect: Assessing the Glasgow Coma Score should be done by the RN. 2. Incorrect: ET tube cuff assessment is accomplished by an experienced RN. 3. Incorrect: Usually, repositioning a client would be within the scope of practice for the LPN/LVN; however, this client is at risk for increased ICP during position changes. The RN must monitor.

The nurse is discharging a client post right radial percutaneous transluminal coronary angioplasty (PTCA) with stent insertion. Which instructions should the nurse give the client to reduce the risk of complications? Select all that apply 1. Do not use the wrist to lift more than 5 pounds (2.27 kg) for 24 hours. 2. Stop taking aspirin in one week. 3. Drink at least 8 glasses of water a day. 4. Wear loose fitting sleeves. 5. Do not shower or soak in a tub for one week. 6. Take short walks around your house.

3., 4., & 6. Correct: Drink eight to ten glasses of water to flush the contrast material from the client's system. The client should wear loose sleeves. We do not want any constriction to the surgical site. In general, the client will need to take it easy for the first two days after getting home. The client can expect to feel tired and weak the day after the procedure, but it is important to take walks around the house. This will help prevent blood clots. 1. Incorrect: The client should limit the use of the wrist. It is important to allow the artery to heal. So, no straining of the wrist. Do not use the wrist used in the procedure to lift more than 2 pounds (0.9 kg) for 24 hours. 2. Incorrect: Clients are maintained on aspirin indefinitely after percutaneous coronary intervention to prevent future thrombotic events. 5. Incorrect: The client can shower after the pressure dressing is removed (usually the day after surgery). The client should keep the area clean and dry when not showering.

What information should the nurse include when providing community teaching on burn prevention strategies? Select all that apply 1. Have chimney professionally inspected every 5 years. 2. Microwave a baby bottle rather than heating on the stove. 3. Clean the lint trap on the clothes dryer after each use. 4. Keep anything that can burn at least 3 feet (0.91 meters) away from space heaters. 5. Hold a hot beverage or hold a child, not both at the same time. 6. Home hot water heater should be set at a maximum of 120°F (48.8°C).

3., 4., 5., & 6. Correct: Lint that accumulates in the lint trap of a dryer can cause a fire, so the lint trap should be cleaned after each use. Space heaters need space at least three feet (0.91 meters) away from anything that can burn. A hot beverage can easily spill on a child by accident when trying to handle both the beverage and child at the same time. Home hot water heater should be set at a maximum of 120°F (48.8°C), especially when small children, the elderly, or diabetics are in the home. 1. Incorrect: A chimney should be professionally inspected every year prior to use. It should also be cleaned if necessary. 2. Incorrect. Never microwave a baby bottle. The formula inside the bottle can become scalding hot in the center.

A client returns from post anesthesia care unit (PACU) following a mastectomy with a Jackson-Pratt drain in place. What action by the nurse is important? 1. Empty drain every eight hours. 2. Irrigate drain with NS every shift. 3. Drape tubing above breast incision. 4. Empty and compress bulb when 2/3 full.

4. CORRECT: A Jackson-Pratt drain is not connected to wall suction, but instead uses gravity and compression to create suction. For maximum efficiency, the bulb must be emptied at only 2/3 capacity. If the bulb becomes filled, suction fails, and fluid will build up in the tissues, possibly leading to wound dehiscence. 1. INCORRECT: The purpose of a surgical drain is to prevent the buildup of fluid under the incision, thus decreasing the chance of dehiscence. Emptying the drain is based on the amount of fluid in the bulb, not timed by shift. 2. INCORRECT: A drainage device connected to the body is meant to pull liquid from inside the body, and therefore is never irrigated. Doing so would increase the chance of infection, even if using sterile normal saline. Pushing fluid into a drain could damage the incision, or even lead to dehiscence. 3. INCORRECT: Any drain or tubing inserted into the body should always be draped below that area or system of the body to improve gravity drainage. The tubing of the Jackson-Pratt drain must hang below the level of the incision.

A client with a history of angina has returned to the unit following a cardiac catheterization. What nursing action has the highest priority? 1. Obtain vital signs every thirty minutes. 2. Assess pedal pulses every ten minutes. 3. Place the call bell within client's reach. 4. Keep affected extremity immobilized for 6 hours.

4. CORRECT: The greatest risk following a cardiac catherization is the potential for hemorrhage, most often from the insertion site. Therefore, the affected extremity must remain straight and immobilized for 4-6 hours after the procedure. 1. INCORRECT: The frequency of vital signs is determined by facility protocol, but generally vital signs are obtained every ten minutes for the first half hour, then every fifteen minutes for another half hour. While vital signs provide valuable information to compare to baseline, another action is more important. 2. INCORRECT: It is vital to assess pedal pulses in order to verify circulation following a catherization. The frequency is based on facility protocol. However, this action is not the highest priority. 3. INCORRECT: Because the client is on bed rest, it is crucial for the client to be able to summon staff when needed. Despite the importance of this action, there is an even more important action.

A tour bus is involved in an accident, sending several clients to the emergency room (ER) for treatment. An unconscious client with multiple internal injuries requires immediate surgery. When itemizing the client's belongings, the nurse finds a wallet containing four thousand dollars. What is the appropriate method for the nurse to secure the money? 1. Place wallet inside client's pants and then in belongings bag. 2. Secure the money in an envelope in the ER narcotics drawer. 3. Sign money over to the hospital CEO until client is discharged. 4. Tally cash with 2nd nurse, document and lock in hospital safe.

4. Correct: All personal valuables in the possession of an unconscious client, including money or jewelry, must be tallied in the presence of two nurses and then documented in the client's main chart. Valuable items such as watches, rings or necklaces must also be secured until a family member is contacted, or the client is able to designate disposition of same. With large amounts of cash, a passport or other such important items, it is vital to account for and secure those items until returned to client or family. When dealing with money, two nurses must count the cash and document the total on the client's chart. The funds are then locked in the main hospital safe until the client is discharged or delegates a family member to retrieve same. 1. Incorrect: Even though the client's belongings bag is personal property, it is not a secure location. The bag is usually kept in the client's room or closet which does not provide security for a large amount of money. 2. Incorrect: While locking the cash into the ER narcotics drawer may be a temporary solution during care of the client, this is not an adequate long-term solution. The client will be sent to the operating room, and then admitted to a room. The money is personal property which should remain with the client in a secured manner. 3. Incorrect: Entrusting the funds to a single individual, even the facility CEO, is not the appropriate method of securing valuables.

A child who fractured the ulna and radius following a fall is experiencing itching under the cast. What would be an appropriate nursing intervention to help alleviate the itching? 1. Apply a small amount of hydrocortisone cream with a cotton tip applicator. 2. Use a soft, sterile, cotton tip applicator to gently rub area under the cast. 3. Apply warm, dry heat to the outside of the cast with a lightweight heating pad. 4. Circulate air under the cast utilizing a blow dryer on the cool setting.

4. Correct: An acceptable, safe way to try to alleviate itching is to use a blow dryer on the cool setting to circulate air under the cast. This is the only safe option provided. 1. Incorrect: Although you may be thinking that hydrocortisone cream is an antipruritic, keep in mind the safety aspects of cast care. Nothing should ever be placed underneath the cast. Because of the risk of skin breakdown, the parents and child should be cautioned to not attempt to place any medication or object under the cast. 2. Incorrect: Remember safety! Skin breakdown could occur, even when using a "soft" object. Never place anything under the cast to try to reduce itching. 3. Incorrect: Applying heat to the cast would most likely increase the itching and could create moisture under the cast. The use of heat on the cast should be avoided.

Which client should the nurse recognize as being at greatest risk for the development of cancer? 1. Smoker for 30 plus years 2. Bodybuilder taking steroids and using tanning salons 3. Newborn with multiple birth defects 4. Older individual with acquired immunodeficiency syndrome

4. Correct: Cancer has a high incidence in the immune deficiency client and in the older adult with both of these risk factors together, this one is the highest risk for cancer. 1. Incorrect: Although smoking is a known environmental carcinogen, this one risk factor alone is not the highest risk. 2. Incorrect: These are known environmental carcinogens, but do not rank as highly as aging and immune deficiency. 3. Incorrect: Birth defects are not a risk factor for cancer.

A client with cervical cancer received an internal cervical radiation implant. What should be the initial nursing action if the radiation implant becomes dislodged and is found lying in the bed? 1. Call the client's primary healthcare provider. 2. Pick up the implant immediately with gloved hands and place it in double biohazard bags. 3. Notify the radiology department. 4. Utilize long-handled forceps to pick up the implant and dispose of it in a lead container.

4. Correct: If a client is receiving a radiation implant, a lead container and long-handled forceps should be placed in the client's room and kept for the duration of the therapy. If the implant becomes dislodged, the nurse should pick up the implant with long-handled forceps and place it in the lead container. 1. Incorrect: The placement of the implant into the lead container should be done initially. The primary healthcare provider may be notified but this is not the initial nursing action needed. 2. The implant should be picked up with long forceps for distance and reduction of contact. In addition, a biohazard bag is not sufficient for proper disposal of the radiation implant. 3. The initial action is to use long-handled forceps and dispose of the implant in a lead container. Calling the radiology department is delaying care and exposing individuals to the implant.

The nurse is repositioning a client who is in the supine position to the right lateral position. Which nursing intervention would be implemented to position the client in the right lateral position? 1. The right leg is positioned on a pillow in front of the left leg. 2. Both knees are kept in the extension position. 3. Both feet are placed in the inversion position. 4. The left shoulder should be positioned forward.

4. Correct: The left shoulder should be adducted. The position of adducting the shoulder forward promotes improved chest expansion and decreases strain on the shoulder. 1. Incorrect: The right leg is positioned forward in the left lateral position. For the right lateral position, the left leg is positioned on a pillow in front of the right leg. 2. Incorrect: Both legs should not be extended for the right lateral position. The left leg should be positioned forward with the knee flexed to decrease the internal rotation of the femur. 3. Incorrect: Inversion of the feet is described as positioning the ankles toward the midline of the body. The feet should be positioned in the neutral position to maintain proper ankle alignment.

The nurse notices the primary healthcare provider removes gloves after performing an invasive procedure on a client. The healthcare provider then enters another client's room without washing hands. What is the initial action by the nurse? 1. Ignore it since the primary healthcare provider knows best. 2. Contact the nursing supervisor. 3. Notify the chief of medical staff. 4. Remind the primary healthcare provider of the importance of standard precautions.

4. Correct: The nurse is the client's advocate and can remind the primary healthcare provider of the importance of washing hands before entering a client's room. Hand washing should be performed when going from one room to another. 1. Incorrect: Nurses are to be client advocates and resolve a problem that they see. The primary healthcare provider should wash their hands prior to entering another client's room. 2. Incorrect: The nursing supervisor is not the first step, the nurse is. This incident may be reported to the charge nurse at a later time but the client's safety is priority. 3. Incorrect: This is not the first step. The nurse should address the problem when it is witnessed. The nurse should follow the chain of command when reporting a problem but speaking to the chief of medical staff is not the best action at this time.

An injured client brought to the emergency room by ambulance insists on leaving before being seen by the primary healthcare provider. What is the nurse's priority action? 1. Explain potential risks of leaving without proper care. 2. Insist the client sign "Against Medical Advice" form. 3. Calmly convince client to wait for needed treatment. 4. Notify primary healthcare provider immediately.

4. Correct: The nurse must notify the primary healthcare provider immediately about the client's desire to leave without care. The client cannot be physically prevented from leaving, or threatened with possible dire consequences by the nurse. The primary healthcare provider can explain potential risks of non-treatment and obtain a signature on the AMA form. 1. Incorrect: The client must be informed about the potential risks of leaving without medical treatment and that information is best explained by either the emergency room healthcare provider or primary healthcare provider based on knowledge of the client's potential injuries. 2. Incorrect: An "Against Medical Advice" (AMA) form is designed to protect staff and facility from potential litigation filed by clients leaving without treatment. However, a client cannot be forced to sign the form and this is not the nurse's priority action. 3. Incorrect: The nurse can use therapeutic techniques to discuss the situation and try to discover why the client wants to leave. However, there is another priority more important for the nurse.

A 68-year-old client with a history of angina presents to the emergency department (ED) reporting flu like symptoms progressively worsening over the past 24 hours.What action is most important for the nurse to initiate? Exhibit: Client's Chief Complaint: "I have the flu. I have been vomiting every couple of hours, running a fever and my chest hurts." Vitals Signs: Pulse-132 beats/minute Respirations-26 breaths/minute Blood pressure-94/60 mmHg Temperature-101.3° F (38.5°C) orally Capillary refill - 4 seconds Primary Healthcare Provider Prescription: Rapid Influenza Diagnostic Test Normal Saline 1 liter at 250 mL/hour, then Normal Saline at 100 mL/hour. Chest X-ray Acetaminophen 500 mg po now. 1. Administer acetaminophen. 2. Initiate IV of Normal Saline at 250 mL/hour. 3. Notify radiology and lab of diagnostic test prescriptions. 4. Discuss IV prescription with primary healthcare provider.

4. Correct: This client needs fluid because of dehydration, but did you notice that this client is elderly and has a history of cardiac problems? I hope so, because giving this client NS rapidly could throw our heart client into pulmonary edema, which would be a bad thing! Talk to the primary healthcare provider. 1. Incorrect: Acetaminophen needs to be administered but it is not the most important thing for the nurse to do. Clarification regarding the IV fluid prescription is necessary here to prevent a possible complication. 2. Incorrect: If this client receives an isotonic IV solution at this rapid rate, the client will be at increased risk of developing FVE and pulmonary edema. 3. Incorrect: Again, the radiology and lab departments can be notified of the test prescriptions to be completed. However, the nurse can assign this task to the unit secretary.

The nurse is planning care for a client admitted with Alzheimer's Disease. What interventions can the nurse delegate to the LPN/VN? Select all that apply 1. Teach caregivers memory enhancement aids. 2. Evaluate client's safety risk factors. 3. Make referrals to community services. 4. Determine caregiver's stress level and coping strategies. 5. Monitor for behavioral changes. 6. Check environment for potential safety hazards.

5. & 6. Correct: The LPN/LVN can monitor for behavioral changes and can look for potential safety hazards. 1. Incorrect: The RN is responsible for teaching. This task cannot be delegated to the LPN/LVN. The LPN/LVN can reinforce teaching. 2. Incorrect: The RN is responsible for assessment and evaluation of clients. The LPN/LVN can gather data, but the RN is responsible for validating and interpreting that data to assess and evaluate. 3. Incorrect: The RN is responsible for developing the plan of care which would include necessary referrals. 4. Incorrect: This again is assessment which is the role of the RN only.

The client with a new diagnosis of hypertension has been instructed to maintain a low sodium diet. Which foods does the nurse plan to teach the client to include on a low sodium diet? Select all that apply 1. Lemonade 2. Broccoli 3. Apple 4. Smoked sausage 5. Boiled shrimp 6. Tomato soup

1, 2, & 3. Correct: Lemonade has about 5 mg of sodium. Broccoli and apples have 0 mg of sodium per serving. 4. Incorrect: Sausage is made from ground meat such as pork, beef, or veal with salt and other spices added. A serving of sausage can have 644 mg of sodium. 5. Incorrect: Shellfish or shrimp are high in sodium. A serving of boiled shrimp can have 111 mg of sodium. Also, the seasoning for the shrimp has sodium added. 6. Incorrect: Processed foods are high in sodium unless the food label states "low sodium". Even though the food may read "low sodium", the client should read the food label to evaluate the sodium content.

The nurse is planning care for four clients with different medical issues. With which diagnosis would a client benefit most from an integrative medicine healthcare strategy? 1. Chronic fatigue syndrome who has had no relief of fatigue. 2. Diabetes whose blood sugars are out of control and refuses to take the prescribed oral and injection medications. 3. Cholecystitis who wants surgery to treat the symptoms definitively. 4. Productive cough with green sputum, fever of 104.2 degrees Fahrenheit (40.1 degrees C), and chest pain.

1. Correct: Chronic fatigue syndrome is a chronic health problem that is difficult to treat using only traditional medicine and responds well to the use of an integrative medicine healthcare strategy by using a combination of traditional and holistic therapies. Integrative medicine is an approach to care that puts the patient at the center and addresses the full range of physical, emotional, mental, social, spiritual and environmental influences that affect a person's health. 2. Incorrect: Clients with acute illness symptoms are more appropriately treated with traditional medicine strategies. 3. Incorrect: Clients with acute illness symptoms are more appropriately treated with traditional medicine strategies. 4. Incorrect: Clients with acute illness symptoms are more appropriately treated with traditional medicine strategies.

Following a thyroidectomy, a client reports shortness of breath and neck pressure. Which nursing action is the best response? 1. Remove the dressing and elevate the head of bed. 2. Call a code, open the trach set, and position the client supine. 3. Obtain vital signs. 4. Immediately go to the nurse's station and call the primary healthcare provider.

1. Correct: The nurse should identify that the client is in respiratory distress. So get the dressing off the neck, elevate the HOB and see if they can breathe any better. Stay with the client. 2. Incorrect: Calling a code and opening a trach set is premature. What is likely the problem? Swelling around the airway. Do something that will decrease swelling. Placing the client flat will make the swelling and breathing worse. 3. Incorrect: Don't just look and check. The nurse must do something. This is delaying treatment. Checking the vital signs will not correct the problem. 4. Incorrect: Never leave an unstable client. If the client is having trouble breathing, then that client is unstable. The nurse can call the primary healthcare provider from the room.

A nurse is monitoring a newly hired unlicensed assistive personnel (UAP) perform a bed bath on a client needing total care. Which action by the UAP would require further teaching? Select all that apply 1. Lowers side rails on both sides of bed. 2. Washes eyes with mild soap and water from the inner to outer canthus. 3. Makes certain bath water temperature is between 110-115°F (43-46°C). 4. Uses long, firm strokes to wash from wrist to shoulder of each arm. 5. Performs a back massage after completing the bath.

1., & 2. Correct: The nurse needs to intervene in these situations. Both side rails should not be lowered because the client could fall out of the bed. The UAP should lower the side rail closest to themselves and keep the opposite rail up. Wash eyes with water only since soap is very irritating to the eyes. 3. Incorrect: This would be a correct action by the UAP. The nurse does not need to intervene. Temperatures less than 110°F (43°C) can chill the client, and a temperature greater than 115°F (46°C) may be too hot and burn the client. 4. Incorrect: This is a correct action and does not require intervention by the nurse. Firm strokes from distal to proximal areas promote circulation by increasing venous blood return. 5. Incorrect: A back massage is appropriate after a bath and does not require nursing intervention. A back massage is a way of providing relaxation for the client.

A nurse is caring for a client diagnosed with pneumonia. What nursing interventions should the nurse implement for the client's night sweats and fever. Select all that apply 1. Keep water by the bedside 2. Place a plastic cover over the pillow 3. Administer an antipyretic every 4 hours 4. Keep a change of linen in the room 5. Position the client in a semi-fowlers position

1., 2., & 4. Correct: The nurse should encourage the client to consume liquids to replace insensible water loss and sweating. The plastic cover will protect the pillow from contact with perspiration. A time management technique for caring for a client with frequent fever episodes is to keep a change of linen in the room. 3. Incorrect: Antipyretics should be administered to reduce a client's fever. Antipyretics are usually prescribed as needed, not every 4 hours. 5. Incorrect: Placing a client in different positions will not affect the client's fever. Positioning the client in a semi-fowler's position may be more appropriate for the client, but will not affect the client's temperature.

What actions should the nurse take when administering fentanyl? Select all that apply 1. Remove old fentanyl patch prior to applying new patch. 2. Cleanse area of old fentanyl patch. 3. Shave hair where fentanyl patch will be applied. 4. Place fentanyl patch over dry skin. 5. Apply adhesive dressing over the fentanyl patch. 6. Dispose of fentanyl patch in trash.

1., 2., & 4. Correct: These are correct actions. Apply patch to dry, hairless area of subcutaneous tissue, preferably the chest, abdomen, or upper back. The old patch should be removed prior to applying a new patch so that too much medication is not given. This is also why the old site should be cleaned. The patch should be placed on dry skin. Do not place over emaciated skin, irritated or broken skin, or edematous skin. 3. Incorrect: Do not shave area where patch will be applied and do not apply over dense hair areas. If there is hair on the skin, clip the hair as close to the skin as possible, but do not shave. 5. Incorrect: Do not apply adhesive dressing over patch. It can interfere with absorption. If the patch comes loose, you may tape the edges and remove and apply a new patch. 6. Incorrect: Dispose of fentanyl patch in sharps container. Fentanyl patches that have been worn 3 days still contain enough medication to cause serious harm to adults and children.

Prior to shift report, the charge nurse is making assignments for the nurses on the shift. Which client can be assigned to the LPN? Select all that apply 1. Client with arthralgia who is receiving regularly scheduled pain medications and has warm compresses prescribed. 2. Client who is a diabetic experiencing diabetic neuropathy. 3. Client who requires teaching about the use of a patient-controlled analgesia (PCA) pump. 4. Client who received blunt abdominal trauma in a motor vehicle accident who is reporting a worsening of the abdominal pain. 5. Client with ureterolithiasis who requires frequent PRN pain medication.

1., 2., & 5. Correct: A LPN should be able to care for a client with arthralgia who requires pain medication on a regular schedule and is receiving warm compresses. The client is apparently stable and does not require any advanced assessment skills or specialized care. Did the words diabetic neuropathy make you think that a LPN should not be assigned to this diabetic? Well, many diabetics experience diabetic neuropathy and it is not a situation that makes this client unstable or critical. LPNs can provide the client with needed analgesics or may simply guide the client with diversional activities for managing this type pain. Did you recognize ureterolithiasis as "kidney stones"? Yes! So, this client who is receiving PRN pain medication is certainly someone that the LPN could be assigned to. 3. Incorrect: Here, you have a client who needs teaching about intravenous pain management using a patient-controlled analgesia (PCA) pump. Teaching is not in the role of the LPN and therefore, this client would need to be assigned to the RN, not the LPN, for the teaching needs of the client. 4. Incorrect: What seems to be going on with this client? The abdominal pain is worsening. This could indicate a worsening of this client's condition. Therefore, this client needs the advanced assessment skills of the RN and should not be assigned to the LPN.

The nurse is observing a new nurse inserting a nasogastric (NG) tube. Which action by the student nurse needs to be corrected by the nurse? Select all that apply 1. Measures from the tip of the nose to the xiphoid process of the client. 2. Lubricates the NG tube with petroleum gel. 3. Aspirates the NG tube to test gastric contents with a pH stip. 4. Marks the tubing at measurement mark with tape and secures to nose. 5. Places tube end into a glass of water to assess for bubbling.

1., 2., & 5. Correct: These actions by the new nurse are not done properly. The measurement for tube placement should be nose to ear and then xiphoid process. Lubricate the tube with a water solution, not a petroleum gel. Never place the tube in water because if the tube is in the trachea, the client can aspirate the water into the lungs. 3. Incorrect: This is the proper technique for checking placement of the NG tube. The pH should be less than 5 if in the stomach. 4. Incorrect: Yes, the tubing should be marked with a piece of tape and secured to the nose with tape or a commercial device if available.

The nurse is educating a group of sexually active teenagers about Chlamydia. What should the nurse teach these clients to prevent them from acquiring or transmitting this disease ? Select all that apply 1. Use a latex condom when having sex to protect against Chlamydia. 2. Seek the advice of a primary healthcare provider if there is vaginal discharge or burning on urination. 3. Suggest that the teens be screened for Chlamydia. 4. Reassure the teens that if they have no symptoms, they have no disease. 5. Take prescribed medication if diagnosed with Chlamydia, and repeat screening in three months.

1., 2., 3. & 5. Correct: Consistent use of latex condoms protects against STIs. Although chlamydia may have no symptoms, burning and discharge should be reported for further evaluation. It is recommended that all sexually active young women less than 25 years of age be screened for chlamydia on an annual basis. Medication should be taken as prescribed, and rescreening should occur in 3 months to make sure that there is no more disease present. 4. Incorrect: Chlamydia does not always produce visible symptoms, and, if left untreated, can lead to pelvic inflammatory disease (PID). False security may lead to unsafe sex practices.

The nurse is reviewing some clients' prescriptions. Which prescription should the nurse question and have corrected? Select all that apply 1. Furosemide 40 mg PO q.d. 2. Lisinopril 20.0 mg PO daily 3. Start MgSO4 at 3g/hr IV 4. Risperidone .5 mg PO daily 5. Dexlansoprazole 30 mg PO daily

1., 2., 3., & 4. Correct: Nurses must use and recognize appropriate terminology and abbreviations to avoid potential client harm. There are potential problems in Options #1, 2, 3, and 4 and should be questioned and corrected. So what is wrong with option #1? Well, do you see the q.d.? This is on the "Do Not Use" list of abbreviations because the period after the "Q" can be mistaken for "I", which would be interpreted as qid (four times a day) instead of the intended once daily dosage. Now, in Option #2, we see a dangerous prescription. There is a trailing zero after the prescribed dose. This could be devastating to the client if the decimal point is missed and the client receives 200 mg instead of the intended 20 mg of lisinopril. For Option #3, you may have recognized MgSO4 as being magnesium sulfate. However, it is on the "Do Not Use" list of abbreviations because it can be confused with morphine sulfate (MSO4). Administering 3 g/hr IV of morphine would be extremely dangerous. In option #4, we see that the leading zero is missing from the prescription. If the decimal point is missed in this situation, the client could receive 5 mg instead of the intended dose of 0.5 mg of risperidone. 5. Incorrect: This prescription is written correctly.

The nurse is preparing to educate a client diagnosed with essential hypertension on how to decrease the risk of developing complications. What topics should the nurse include? Select all that apply 1. Following the DASH dietary plan. 2. Use of blood pressure monitoring device. 3. Diaphragmatic breathing exercises. 4. Brisk walking for 30 minutes 3-4 times/week. 5. Reduce sodium intake to less than 2700 mg/day.

1., 2., 3., & 4. Correct: The DASH Eating Plan is recommended for clients who have hypertension. It is high in vegetables, fruits, low-fat dairy products, whole grains, poultry, fish, beans, and nuts and is low in sweets, sugar-sweetened beverages, and red meats. Home blood pressure monitoring can help the client keep closer tabs on their blood pressure, show if medication is working, and even alert the client and primary healthcare provider to potential complications. However, home blood pressure monitoring isn't a substitute for follow-up visits. Reducing stress as much as possible helps to decrease blood pressure. Healthy coping techniques, such as muscle relaxation, deep breathing or meditation are good options. Getting regular physical activity and plenty of sleep can help, too. Regular physical activity can lower blood pressure, manage stress, reduce the risk of several health problems and keep weight under control. 5. Incorrect: A limit of 1500 mg of sodium per day is preferred on the low sodium DASH diet. On the standard DASH diet 2,300 mg of sodium are allowed each day.

The client diagnosed with active tuberculosis has been prescribed isoniazid 300 mg by mouth every day. What should the nurse teach this client? Select all that apply 1. "Notify your healthcare provider if your urine turns dark." 2. "Your healthcare provider has prescribed B6 along with the isoniazid to prevent neuritis." 3. "You should avoid eating aged cheeses and smoked fish." 4. "Eat foods such as tuna twice a week." 5. "Rise slowly from lying to sitting, or sitting to standing."

1., 2., 3., & 5 Correct: Signs of hepatotoxicity from this medication include dark urine, jaundice, and clay-colored stool. Isoniazid- induced pyridoxine (Vitamin B6) depletion causes neurotoxic effects. Vitamin B6 supplementation of 10-50 mg usually accompanies isoniazid use. Aged cheeses and smoked fish are high in tyramine which may cause palpitations, flushing, and blood pressure elevation while taking isoniazid. Avoid these foods during treatment. Isoniazid should be taken on an empty stomach, one hour before or two hours after food. Some clients experience orthostatic hypotension while taking isoniazid, so caution against rapid positional changes. 4. Incorrect: Histamine containing foods such as tuna and yeast extracts may cause exaggerated drug response (H/A, hypotension, palpitations sweating, itching, flushing, diarrhea).

The nurse educator has provided education to newly hired emergency department nurses regarding mandatory reporting laws. Which suspected instances provided by the new nurses indicates to the nurse educator that education was effective? Select all that apply 1. Financial abuse of an elder 2. Negligence of a colleague 3. Spousal abuse denied by the victim 4. Gunshot victim 5. Client diagnosed with Gonorrhea 6. Client diagnosed with West Nile virus

1., 4., 5., & 6. Correct: Federal and state laws require that certain individuals, particularly those who work in health care with the elderly, with children, and other vulnerable populations, have an affirmative duty to report to a specified state agency when violence occurs against those populations. This includes physical, mental, and financial abuse. Gunshots and knife injuries are reportable to law enforcement. Certain communicable diseases such as gonorrhea and West Nile virus are reportable to the CDC. 2. Incorrect: Suspected negligence of a colleague is not in the realm of mandatory reporting to authorities, but the nurse should discuss with the supervisor. 3. Incorrect: A spouse is not considered a vulnerable person so it is not required by law to report. You should encourage the spouse to report the abuse but you, as the nurse, are not bound by law to do so.

A client diagnosed with a brain injury continues to attempt to get out of the bed without assistance. Which nursing interventions would the nurse implement? Select all that apply 1. Ask a familiar person to stay with the client. 2. Apply position change sensor to the bed. 3. Move client closer to the nursing station. 4. Reinstruct the client to not get out of the bed. 5. Provide positive and negative reinforcement.

1.,2. & 3: Correct: Having a person directly monitor the client will decrease the possibility of the client getting out of the bed. In addition, a familiar person in the room can have a calming effect on the client. Bed alerts will notify the healthcare team that the client is moving in the bed. This will result in a quicker response time to evaluate, if the client is trying to get out of bed. The intervention of moving the client closer to the nursing station will increase the observation of the client. This increased visualization can allow the healthcare team to intervene if the client tries to get out of the bed. 4. Incorrect: Due to the brain injury, the client's ability to process information, including instructions is limited. The client may become agitated and exhibit restless behaviors. Reinstructing the client will not be effective if the client is having difficulty processing the initial instructions. 5. Incorrect: Due to the brain injury, cognitive deficits occur resulting in the decreased ability for the client to interpret information. The client will not have the ability to recognize positive reinforcement messages. The client should not be subjected to any negative reinforcement actions.

A hospitalized client reports needing scented candles to aid sleep. The nurse informs client lit candles are not permitted in the facility. What appropriate alternatives could the nurse suggest to the client to assist with the sleep process? Select all that apply 1. Use an electric potpourri burner. 2. Place dry potpourri in nightstand. 3. Bring in live flowers to keep in room. 4. Spray scented air freshener frequently. 5. Dab scented oil on corner of the sheets.

2 and 5. CORRECT: The nurse must provide the client with alternatives methods to aid sleep that do not present a safety hazard. Potpourri is fragrant dried flowers or plant stems which emit a smell based on the assortment. Sprinkling a small amount inside the nightstand drawer would allow the scent to gently permeate the area next to the client's bed without presenting a safety hazard and the aroma would be consistent over long periods of time. Also, a tiny drop of an essential oil dabbed on the corner of the pillow case or sheet would also provide the client with desired needed sleep enhancement without impacting health or safety issues. 1. INCORRECT: Hospitals have specific regulations about outside electronics, requiring most to be checked by maintenance staff prior to use in the facility. An electric potpourri burner melts scented wax in a small open ceramic dish, providing odor as it dissipates. An open container of hot wax plus the need to keep the burner plugged in all night are extreme safety hazards to client and staff. 3. INCORRECT: This inefficient and costly method to aid client sleep is impractical on several levels. The fragrance of flowers rapidly fades, based on ambient temperature and room size. Additionally, the inconsistency of smell would not provide the same restful level of sleep during the night. 4. INCORRECT: Spraying non-hospital approved air freshener could prove problematic for other clients, since that odor would not be confined to just the client's room. Secondly, the quick dissipation of the spray would not provide an entire night of restful sleep.

A client is to be discharged following a left modified-radical mastectomy. When reviewing ADL's to be completed at home, the nurse anticipates the client will experience the most difficulty doing what tasks? Select all that apply 1. Cooking a meal. 2. Shampooing hair. 3. Doing the laundry. 4. Vacuuming carpets. 5. Changing bed linens.

2, 3, & 5. CORRECT: The modified-radical mastectomy is a surgical approach to cancer in which the breast tissue, nipple, and axillary lymph nodes are removed but the chest muscles remain intact. Following surgery, individuals usually experience pain and stiffness when resuming normal daily activities, particularly tasks which require stretching the arm above the head. Shampooing or drying hair would be challenging, as would moving loads of heavy laundry between washer and dryer. Also difficult is changing bed sheets because it involves lifting and stretching across the bed. 1. INCORRECT: The process of cooking food can be modified in such a way the client would not need to extend the surgical arm above the head or in a painful position. 4. INCORRECT: Vacuuming carpet does not require lifting or reaching if the client uses an upright sweeper. This task should not present difficult challenges and can be completed with the non-surgical arm.

A hospitalized client using a K-pad on an injured muscle reports the pad is not warming up. What should be the nurse's initial action? 1. Unplug unit and plug into another wall outlet. 2. Check temperature setting on the heating unit. 3. Call maintenance to repair unit immediately. 4. Increase temperature on unit till pad heats up.

2. CORRECT. The nurse is utilizing the nursing process by first collecting data pertinent to the situation. The actual problem could be related to the temperature dial on the unit, or even a malfunction in the pad itself. However, the nurse must assess the situation by checking the basics, such as whether the equipment is even turned on. 1. INCORRECT. While it is possible the outlet itself may be defective, this is not likely in a large facility. Additionally, an electric appliance should never be re-connected to an outlet while still in contact with the client. 3. INCORRECT. It is unlikely maintenance would be available to examine the device immediately and most repairs should not be attempted in the client's room because of safety considerations. 4. INCORRECT. The exact problem with the heating unit has not yet been established. Simply turning up the temperature setting is not safe since the pad may quickly get hotter, injuring the client.

The nurse is reviewing discharge instructions with the spouse of client following a laminectomy. When the nurse explains the need to log roll the client, the spouse expresses doubt about the ability to do so independently. What statement by the nurse is appropriate? 1. "Many spouses have been able to learn this procedure." 2. "Which part of this procedure has you most concerned?" 3. "Don't you have any family to help you with this procedure?" 4. "Are you worried about caring for your spouse?"

2. CORRECT. The nurse's question is open-ended since it allows the spouse to elaborate on any specific areas of concern or doubt. This approach encourages the spouse to express feelings with any care after discharge, and not just the log rolling technique. 1. INCORRECT. This statement by the nurse directs attention away from the spouse's expressed concerns, ignoring feelings stated by the spouse. It implies anyone could perform the needed log rolling and is dismissive of the spouse. 3. INCORRECT. Although the nurse may have meant to suggest others could help the spouse, the phrasing of the question insinuates the spouse should seek others to help, whereas the nurse should focus on educating and encouraging the spouse to perform the task independently. 4. INCORRECT. While the spouse's verbalized concerns may be subconsciously connected to overall care of a post-surgical client, the nurse's comment is an assumption and is confrontational.

When the surgical transport team arrives to take a client to the operating room, the client is sitting in a chair in the room. What is the best way for the nurse to get the client onto the transport litter? 1. Using a footstool, assist client to step up and crawl onto litter. 2. Have client return to bed and utilize slide board to transfer to litter. 3. With feet placed apart, grasp client around waist and lift onto litter. 4. Put Hoyer pad under client, using Lift to move client from chair to litter.

2. CORRECT. The safest, most efficient manner by which to place the client on the litter properly is to have client first return to bed. The bed can then be raised to the height of the litter, allowing staff to utilize a slide board to easily position the client onto the litter. This method decreases safety risks for both staff and client. 1. INCORRECT. Even with assistance, stepping up onto a stool greatly reduces the client's stability to that small surface area. Crawling on the litter would make it difficult for the client to get properly positioned. The stool may be too short for staff to even assist. 3. INCORRECT. It is never safe for a nurse to lift a client by grasping around the waist, no matter how feet are positioned. This method could potentially injure both the nurse and client while increasing the risk of a fall. 4. INCORRECT. Positioning a Hoyer lift pad under a sitting client is impractical and nearly impossible. Proper placement can best be accomplished with the client supine in bed. This individual is obviously mobile, negating the need for any lifting device. This is not the best action.

A home health nurse is educating a female client about home care considerations for intermittent catheterization. Which statement by the client would let the nurse know that the client understands what has been taught? 1. "After insertion, I will tape the tubing to my lower abdomen." 2. "I will wash the rubber catheter thoroughly with soap and water after use." 3. "It is important that I keep the drainage bag below the level of my bladder." 4. "Catheterization should be done hourly."

2. CORRECT: There are many foods and drinks that might cause discomfort for the client, particularly in the early stages of treatment. Although specific foods can vary among individuals, usually spicy foods, caffeine, and even alcohol can contribute to the burning sensation reported by clients with GERD. This statement by the client is accurate. 1. INCORRECT: Lying down after a large meal often contributes to reflux because the pressure of food permits stomach contents and acid to flow back up the esophagus, leading to heartburn and possibly regurgitation. Clients should remain upright for a period of time after eating, which allows gravity to keep acid below the level of the esophagus. 3. INCORRECT: Omeprazole is a proton-pump inhibitor which decreases stomach acid and works to heal existing ulcers. This medication is taken once daily at the same time, and should never be doubled unless ordered to do so by the primary healthcare provider. Any increase in discomfort while taking this medication should be immediately reported. 4. INCORRECT: A proton-pump inhibitor is taken once daily, usually in the morning prior to breakfast. This medication is not administered only in the presence of pain. Taking this medication consistently over time will decrease stomach acid and help heal any damaged stomach tissue.

The nurse is instructing a client newly diagnosed with gastroesophageal reflux disease (GERD) who has been prescribed omeprazole. What comment by the client indicates to the nurse that the teaching was successful? 1. "I should lay down after eating a big meal." 2. "Spicy food and caffeine might cause me pain." 3. "If the pain gets worse, I should take two pills." 4. "I will take the omeprazole whenever I have pain."

2. CORRECT: There are many foods and drinks that might cause discomfort for the client, particularly in the early stages of treatment. Although specific foods can vary among individuals, usually spicy foods, caffeine, and even alcohol can contribute to the burning sensation reported by clients with GERD. This statement by the client is accurate. 1. INCORRECT: Lying down after a large meal often contributes to reflux because the pressure of food permits stomach contents and acid to flow back up the esophagus, leading to heartburn and possibly regurgitation. Clients should remain upright for a period of time after eating, which allows gravity to keep acid below the level of the esophagus. 3. INCORRECT: Omeprazole is a proton-pump inhibitor which decreases stomach acid and works to heal existing ulcers. This medication is taken once daily at the same time, and should never be doubled unless ordered to do so by the primary healthcare provider. Any increase in discomfort while taking this medication should be immediately reported. 4. INCORRECT: A proton-pump inhibitor is taken once daily, usually in the morning prior to breakfast. This medication is not administered only in the presence of pain. Taking this medication consistently over time will decrease stomach acid and help heal any damaged stomach tissue.

The nurse is discussing foot care with a client who was recently diagnosed with diabetes. Which statement by the client indicates an understanding of foot care? 1. "I will soak my feet for 30 minutes a day." 2. "I will avoid using a heating pad on my feet." 3. "I can use scissors to remove the corns on my toes." 4. "I enjoy walking without my shoes around the house."

2. Correct: One of the long-term complications of diabetes is peripheral neuropathy. As the neuropathy progresses the feet have reduced sensation and may eventually become numb. The client should avoid using heating pads and hot water bottles. Due to the decrease sensation of the feet, the client is in danger of blistering and burning the feet. 1. Incorrect: A complication of diabetes is an increased risk of foot infections. The client is immunocompromised which impairs the leukocytes that destroy bacteria. The client should not allow moisture to accumulate between the toes. 3. Incorrect: Due to the possibility of the client experiencing peripheral neuropathy, the client should not remove any corns from their toes. If a cut occurs while removing the corn, the client is a risk for an ulcer developing. A primary healthcare provider should prescribe the appropriate treatment for corns. 4. Incorrect: Walking without appropriate shoes is dangerous for the client diagnosed with diabetic peripheral neuropathy. After stepping on an object, the client cannot feel the damage to the skin which could result in a scratch or cut.

After a thoracotomy, which interventions will the nurse initiate to reduce the risk of acute respiratory distress? Select all that apply 1. Allow 4 hours of rest between deep breathing and coughing exercises. 2. Splint the incision during deep breathing and coughing exercises. 3. Have the client drink a glass of water before coughing. 4. Perform percussion and vibration every 2 hours. 5. Promote incentive spirometer use several times per hour while awake.

2., & 5. Correct: Splinting helps with the ability to control pain and produce an effective cough. Incentive spirometry encourages deep inspiratory efforts, which are more effective in re-expanding alveoli than forceful expiratory efforts. 1. Incorrect: They need to cough more often than every 4 hours. It is the best when this is done every 2 hours. 3. Incorrect: It takes longer than a few minutes to liquefy secretions and, if the stomach is full, vomiting may occur which would put the client at risk for aspiration. 4. Incorrect: After the surgery, we do not want to percuss and vibrate the incision. Besides being extremely painful, this could potentially disrupt the suture line.

A client is being scheduled for a cat scan (CT) of the abdomen with contrast. When considering client safety, what should be the priority action for the nurse to implement? 1. Verify that informed consent has been provided. 2. Confirm with client the accuracy of allergies listed. 3. Force fluids following procedure. 4. Monitor output following procedure.

2. Correct: When considering client safety, the nurse should confirm allergies with the client. Clients should be asked about allergies to iodine or shellfish. The radiocontrast agents in the dye contain iodine and have resulted in severe reactions and even death in a few cases. If the client is allergic to iodine, the healthcare provider should be notified before the CT is performed. The use of contrast dye for the procedure will typically be omitted to avoid the risk of a severe reaction. 1. Incorrect: It is very important that a client receive information regarding risks and benefits of a procedure before providing consent (informed consent), but assuring that the consent was provided is not the priority for client safety over the risk of a severe reaction to the dye. 3. Incorrect: Again, it is very important to implement increased fluid intake following procedures, such as this CT of the abdomen, to help flush the dye through the kidneys. However, the safety priority remains the potential for a severe reaction that should be avoided by asking about allergies to iodine. 4. Incorrect: Monitoring urine output is an important nursing action following the CT because sometimes dye can lead to kidney problems or can increase problems in clients with existing renal disease. But this is not a priority over assessing for allergies that could lead to severe reactions.

Following a passenger train derailment, local hospitals are notified to activate disaster protocols on all floors. Which actions should be instituted by each unit's charge nurse? Select all that apply 1. Turn on local news for up-to-date information on the train derailment. 2. Prepare a list of clients who could quickly be discharged or transferred. 3. Determine which personnel could be sent to the command center. 4. Notify clients that the disaster plan has been put into effect. 5. Alert all off-duty personnel to stand by in case of call- in.

2., 3. & 5. Correct: All facilities are required to develop a disaster plan, per JCAHO (Joint Commission on Accreditation of Healthcare Organizations) regulations, though the plans vary. However, there are some basic points which are standard among all facilities. This situation is considered an external disaster which means the hospital will be expecting multiple victims. The charge nurse on each unit needs to prepare a list of possible discharges or transfers to be given to the appropriate primary healthcare providers for further action. When handling any disaster, a facility must have a "command center" that is operated by outside personnel such as a Fire chief, Police, Swat or other outside emergency persons. This center functions as 'information central', where all plans or activities are coordinated and determined by those personnel. However, each unit must have one designated representative to send to the command center, when requested, to receive and then relay, pertinent information back to the unit. These individuals are selected by the charge nurse, and do not have to be nurses. Additionally, off-duty personnel may be needed and should be alerted to stand by; however, the command center alone makes the determination whether extra personnel should be called in, or if it would put more individuals in jeopardy. 1. Incorrect: First, the local news does not necessarily have the most accurate information on the disaster. Secondly, staff will be far too busy to watch television or listen to the radio with all the activity occurring hospital wide. The command center is the only reliable source of information and will make any decisions needed by hospital personnel. 4. Incorrect: This would unnecessarily alarm the clients. Most likely, the clients will be aware of the disaster already, and further information could be confusing or frightening. Those clients who may be discharged or transferred will be informed, but it is not appropriate to alert every client.

A client who has Parkinson's disease has a new prescription for benztropine. What should the nurse include when teaching the client and spouse about this medication? Select all that apply 1. This medication blocks dopamine in the brain to decrease tremors and muscle stiffness. 2. Notify your primary healthcare provider if you develop urinary retention. 3. Benztropine can reduce the ability to sweat, so do not become overheated. 4. No lab tests are needed while taking this medication. 5. Sit up or stand up slowly to prevent lightheadedness.

2., 3., & 5. Correct: Urinary retention is a side effect of benztropine. Benztropine can reduce the ability to sweat and cause the body to overheat. Do not become overheated in hot weather or while you are being active because heat stroke may occur. Benztropine may cause dizziness, lightheadedness, or fainting. Alcohol, hot weather, exercise, or fever may increase these effects. To prevent these negative effects, sit up or stand slowly, especially in the morning. Sit or lie down at the first sign of any of these effects. 1. Incorrect: Benztropine is an anticholinergic. It works by decreasing the effects of acetylcholine, a chemical in the brain. This results in decreased tremors or muscle stiffness, and helps improve walking ability for clients with Parkinson's disease. 4. Incorrect: Lab tests, including liver function, kidney function, lung function, blood pressure, fasting blood glucose, and blood cholesterol, may be performed while using benztropine. These tests may be used to monitor the client's condition or check for side effects.

The nurse is planning care for the prevention of skin breakdown in a client diagnosed with a stroke. What intervention is important for the nurse to include? Select all that apply 1. Massage reddened skin areas located over bony prominences. 2. Place pillows under lower extremities to raise heels off the bed. 3. Position client on paralyzed side for one hour. 4. Apply emollients to dry skin. 5. Place a gel seat cushion on the wheelchair seat. 6. Shift client weight every two hours while sitting in a wheelchair.

2., 4., & 5. Correct: These interventions will decrease the risk of skin breakdown by eliminating sustained pressure to areas at greatest risk of breakdown. 1. Incorrect: Do not massage the damaged area because this may cause additional damage. 3. Incorrect: This is way too long. The client should only be on their paralyzed side for 30 minutes. 6. Incorrect: This is way too long. Skin breakdown can result within this period of time. The client's weight should be shifted within the wheelchair every 15-20 minutes.

The nurse is assisting a client with right-sided weakness to transfer from the hospital bed to a wheelchair. The client has an IV attached to an IV pole on the right side of the bed. How should the nurse complete this transfer? Select all that apply 1. Place the wheelchair on the left side of the bed. 2. Place the wheelchair on the right side of the bed. 3. Face the wheelchair toward the foot of the bed. 4. Face the wheelchair toward the head of the bed. 5. Have client grab the wheelchair with the right arm. 6. Have client grab the wheelchair with the left arm.

2., 4., & 6. Correct: The wheelchair should be placed on the right side of the bed where the equipment is located. It needs to face the head of the bed so the client can reach the chair with the strong left arm to help with the transfer. The client should grab the wheelchair arm with the strong left arm. 1. Incorrect: Since the IV and IV pole are on the right side of the bed, the wheelchair should be placed on the right side rather than the left side of the bed. There would not be enough slack in the IV tubing to get out on the left side. 3. Incorrect: If the wheelchair faces the foot of the bed, then the client would not be able to reach with the wheelchair arm with the strong left arm. The client needs to be able to use the left arm for stability. 5. Incorrect: The client should grab the wheelchair arm with the strong left arm. The right side is weak and grabbing with this side puts the client at an increased risk for falls and injury.

A hospitalized client diagnosed with rheumatoid arthritis is receiving IV methylprednisolone every six hours. What is the best method for the nurse to provide client safety? 1. Place "fall precautions" sign above client's bed. 2. Change the intravenous site for steroids daily. 3. Restrict any visitors with visible illnesses. 4. Put client on full contact precautions.

3. CORRECT: Rheumatoid arthritis is an autoimmune disease that affects not only body joints but also organs of the body. Receiving methylprednisolone as treatment further suppresses the immune system, making the client even more at risk of infection. Restricting visitors with colds, respiratory problems and other infectious processes is the best method to protect the client. 1. INCORRECT: The question states the diagnosis is rheumatoid arthritis, but there is no indication the client is unsteady or needs to be on "Fall Precautions". Although the client is fatigued and has brittle bones, there is no evidence the client needs assistance ambulating. A sign is not necessary. 2. INCORRECT: Most facilities have policies to change an IV site at specific intervals, usually every three days. Changing the site daily exposes the client to an increased chance of infection from the invasive procedure. Steroids do not irritate veins and do not require frequent site changes. 4. INCORRECT: There is no rationale for contact precautions since the client's disease process is not contagious. The main concern is to protect the client from other individuals.

The nursing supervisor is reviewing several instances in which restraints have been used. The nurse is aware the only acceptable use of restraints is what? 1. An elderly male had a chest restraint applied after crawling over bed rails several times. 2. An Alzheimer client's room door is closed to prevent wandering during shift change. 3. A confused client with a closed head injury had hand mitts applied after pulling out IV 4. A dementia client with sundowners is placed in Geri-chair with lap belt at nurse's station.

3. CORRECT: Restraints are considered a last resort when caring for a client, whether soft cloth or chemical restraints. The most acceptable use is to prevent a client from harming self or others. In this instance, a confused client has previously pulled out a prescribed IV. Therefore, the use of hand mitts is the most appropriate, least-restrictive method to prevent the client from further self-harm. 1. INCORRECT: There are several problems here. The client had side rails up, which are considered a form of restraint and in many facilities are no longer permitted. By applying a chest restraint, the client has been restrained twice. Just because a client is elderly does not mean restraints are needed. This restraint is not acceptable. The nurse should provide regular toileting periods and determine why this client is climbing out of bed. 2. INCORRECT: Closing a client into a room is overly restrictive and unsafe. This Alzheimer's client needs to be observed and closing the room door prevents visual access. Additionally, closing the door may violate fire safety codes in certain facilities. At shift change, when staff is occupied with report, special arrangements should be made so that the client can be observed and not restrained. 4. INCORRECT: Depending on the facility, placing a client upright at night, using a Geri-chair and a lap belt is overly restrictive. A client with dementia is challenging, particularly in the presence of sundowner syndrome. However, keeping a client upright all night, belted into a chair for the purpose of observation, is neither safe nor healthy for the client.

Following surgery, a client has an indwelling urinary catheter attached to a collection bag. The nurse empties the collection bag at 0900. At the change of shift at 1500, the collection bag contains 100 mL of urine. The system has no obstructions to urinary flow. What would be the nurse's most appropriate initial response? 1. Elevate the head of the client's bed. 2. Start giving the client 8 ounces of oral fluid per hour. 3. Check circulation and take the vital signs of the client. 4. Continue monitoring, because this is an expected finding.

3. Correct: A urine output (U/O) of 100 mL over a 6 hour period is dangerously low. This client could be experiencing hypovolemic shock. In clients who are "shocky", the kidneys stop making urine to try to hold on to what little volume the body has left. The nurse is checking the vital signs for low BP and increased HR, indicators of hypovolemic shock. Also, when the urine output is this low, the client is at risk for renal failure. 1. Incorrect: Elevating the head of the client's bed is a good choice when the client is having difficulty breathing, but not here. Raising the HOB will cause the BP to drop lower. Clients in shock should be supine. 2. Incorrect: Normally, pushing fluids is a good choice if the urine output were low. 100 mL over six hours requires more aggressive treatment to combat shock. 4. Incorrect: This is not an expected finding. Urine output less than 240 mL in an eight hour time frame should alert the nurse to a serious problem such as shock.

A client with the diagnosis of mild anxiety asks the nurse why the primary healthcare provider switched medications from lorazepam to buspirone. What should the nurse tell the client? 1. "Lorazepam takes longer to start working than buspirone so the primary healthcare provider decided to switch medications." 2. "Buspirone can be stopped quickly if necessary." 3. "Buspirone does not depress the central nervous system like lorazepam does, so you should not have as much sedation." 4. "You need to ask your primary healthcare provider why the medication was changed from lorazepam to buspirone."

3. Correct: Buspirone does not depress the CNS system and is believed to produce the desired effects through interaction with serotonin, dopamine, and other neurotransmitter receptors. 1. Incorrect: Buspirone takes 1-2 weeks to take effect and can take up to 4-6 weeks to achieve full clinical benefits. Lorazepam is a benzodiazepine and begins to work within a few hours to 1-2 days. 2. Incorrect: The client should not stop taking any anti anxiety medications abruptly. Serious withdrawal symptoms can occur: depression, insomnia, anxiety, abdominal and muscle cramps, tremors, vomiting, sweating, convulsions, delirium. 4. Incorrect: The nurse should be able to discuss medication administration with the primary healthcare provider.

A float nurse arrives on the unit to assist in the care of clients for the shift. During report, the nurse notes that the float nurse appears disheveled, flushed, and is trembling slightly while drinking coffee. Based on this information,what should the nurse do? 1. Ask the float nurse, "Have you been drinking?" 2. Assist the float nurse with the clients case. 3. Notify the charge nurse of the observations. 4. Notify the board of nursing (BON) that the float nurse is an alcoholic.

3. Correct: If suspicious behavior occurs, it is important to keep careful, objective records. Confrontation should occur in the presence of a charge nurse or supervisor. This can prevent harm to client's. 1. Incorrect: If alcohol or drug dependency is suspected, confrontation will result in hostility and denial. The nurse should not lecture, scold or argue with the float nurse. 2. Incorrect: This response overlooks a potentially severe problem. Nurses dependent on drugs or alcohol can harm clients. The nurse should not be assigned to provide care if impairment is suspected. Patient safety must remain the priority. 4. Incorrect: If a report is made to the BON, it should be a factual documentation of specific events and actions, not a statement of impairment. The report should contain consequences. Each state BON differs in that also some have treatment programs they administer themselves.

A nurse is caring for a client on the second day after a thoracotomy. The client reports incisional pain. The nurse assesses the client and evaluates the vital signs. Based on the data documented in the chart, what action should the nurse take first? Exhibit Nursing Notes: Client reports incisional pain as 8/10. WOund is clean and dry, without redness, edema, or drainage. Shallow respirations noted at 24/min. Adventitious lung sounds noted in bilateral bases. Vital Signs: Oral temperature 100º F/37.8ºC Heart rate 92/min and regular BP 130/80 Respirations 24/min 1. Have client cough and deep breathe. 2. Administer acetaminophen for fever 3. Administer the prescribed analgesic 4. Assist the client to ambulate.

3. Correct: The client described in this question is post thoracotomy. With ANY post-op client, the number one concern, especially as a brand new nurse, is preventing pneumonia. A thoracotomy is very painful and the client is unlikely to breathe deep unless the pain is relieved. Temperature of 100º F/37.8ºC, HR 92, respirations 24, bilateral crackles (indicating atelectasis) all reveal this client is heading for pneumonia. 1. Incorrect: Coughing and deep breathing exercises are exactly what the client needs, but the client will not cough and deep breathe if it hurts. Give pain medication first. 2. Incorrect: Acetaminophen is not potent enough to relieve pain. The goal is to "fix the problem". The problem is that the client is not properly deep breathing due to pain. 4. Incorrect: Assisting the client to ambulate is a good idea, but the nurse has to fix the problem, and the problem is that the client is not deep breathing.

Which immediate action should a nurse take if a client's chest tube is accidentally disconnected from the disposable water-seal system? 1. Have client hold breath 2. Administer oxygen 3. Place the tubing coming from the client into sterile water 4. Raise the head of the bed

3. Correct: The nurse should immediately place the tubing coming from the client into at least 2 cm of sterile water. A disconnected chest tube can allow air to travel into the client's chest cavity and create a life-threatening tension pneumothorax. By putting the end of the disconnected tube into sterile water, a water seal is recreated that will prevent air from entering but will still allow air to escape. 1. Incorrect: For how long? No, a water seal needs to be created. 2. Incorrect: Oxygen is not always the first thing the nurse should do. Re-establish the water seal. 4. Incorrect: Elevating the HOB will not fix the problem. Re-establish the water seal.

A nurse is caring for a client admitted to the hospital for a total hip replacement. In preparing the post-operative plan of care for this client, the nurse recognizes which goal as the highest priority? 1. Prevent complications of shock. 2. Prevent dislocation of prosthesis. 3. Prevent respiratory complications. 4. Prevent skin breakdown.

3. Correct: The postoperative client with a total hip replacement is at risk for thromboembolism and fat emboli which can travel to the lungs and cause respiratory distress. Without proper turning, coughing, and deep breathing, pneumonia and atelectasis may occur. So preventing respiratory complications is high on the priority list. Remember the ABCs - airway, breathing, then circulation. Preventing respiratory complications is the highest priority because of the possibility of sudden death from the complications of deep vein thrombosis and pulmonary embolism. 1. Incorrect: This client is at risk for hemorrhage and/or hematoma formation related to surgical trauma to blood vessels (the hip is a very vascular area) and use of anticoagulants or antiplatelet agents before and after surgery. So the nurse will need to monitor for shock caused by loss of volume. The nurse should monitor drains, wound dressings, and intake and output. But remember, Airway and Breathing take priority. 2. Incorrect: Dislocation of the prosthesis is another complication to worry about. It will cause pain and possible deformity and is very important, but airway is the priority. Dislocation of the hip prosthesis is related to weakness of the hip muscles, improper positioning or movement of the operative extremity, and/or noncompliance with weight-bearing limitations. 4. Incorrect: The client is at risk for skin breakdown if not turned and repositioned properly or ambulated as soon as prescribed. However, Airway is still the priority for this client.

The nurse is caring for four clients. Which client should the nurse see first? 1. The client hospitalized with dehydration related to diarrhea. 2. The seizure client who is currently in the postictal phase. 3. The post-op client who received Morphine 4 mg IV 15 minutes ago. 4. The client who is due pre-op medication now.

3. Correct: This client is at risk for respiratory depression caused by morphine and should be assessed. Remember airway, breathing and circulation (ABCs). Decreased or suppressed respiration are priority. 1. Incorrect: Dehydration can produce postural hypotension, fever, confusion, agitation and if it develops quickly or is severe, coma and seizure may occur. Decreased respiratory rate would be priority. 2. Incorrect: Postictal is the phase after the seizure where they are drowsy, lethargic, and possibly asleep. Make sure the client is safe and in the recovery position. Client would need to be seen soon, but again, decreased respirations takes priority. 4. Incorrect: Decreased or suppressed respirations would be priority over the client needing pre-op medications.

A client with recurrent angina and hypertension has been started on new medications. When reviewing the admission forms, the nurse should immediately question which prescription? Exhibit Primary Healthcare Provider Prescription: Spironolactone 50 mg. PO once daily Metoprolol 25 mg. PO once daily Diltiazem 120 mg. PO once daily Potassium 10 meq PO once daily 2 gm. sodium diet 1. 2 gm sodium diet 2. Metoprolol 25 mg PO once daily 3. Potassium 10 meq PO once daily 4. Diltiazem 120 mg PO once daily

3. Correct: This client is being treated for recurrent angina with hypertension. The admission prescription includes spironolactone daily, which is a potassium-sparing diuretic; therefore, the client should NOT be taking a daily dose of potassium. 1. Incorrect: A 2 gram sodium diet is considered a low salt diet, which would be appropriate for a client with hypertension. Excessive dietary salt leads to water retention and increased blood pressure. This prescription is appropriate for the client and does not need to be questioned. 2. Incorrect: Metoprolol is a beta-blocker used to decrease preload, which will also decrease pulse and blood pressure. The dose is appropriate for this client and does not need to be questioned. 4. Incorrect: Diltiazem is a calcium channel blocker which vasodilates the arterial system and reduces recurrent angina by decreasing afterload. Additionally, calcium channel blockers help to decrease blood pressure. This medication and dose are appropriate for this client.

While preparing an IV in the med room, you observe a new nurse drawing up a dose of insulin in a tuberculin syringe. What is your priority action? 1. Report the incident immediately to the charge nurse. 2. Tell new nurse you will prepare and give the insulin dose. 3. Discuss procedure to prepare insulin with the new nurse. 4. Draw up insulin but let new nurse administer the injection.

3. Correct: This situation is dangerous since the new nurse obviously does not know the proper process or even the correct syringe with which to draw up insulin. This lack of knowledge could lead to serious or even fatal consequences for a client. It is vital the new nurse be properly instructed and then supervised on how to prepare insulin. Additional measures to ensure client safety can then be pursued. 1. Incorrect: Client safety is always priority, and therefore the new nurse must be stopped immediately before going any further with this procedure. Informing the charge nurse of the situation can wait until steps to ensure client safety are completed. 2. Incorrect: Completing the task personally may keep this client safe at the moment; however, it does not ensure the error will not be repeated with another client. 4. Incorrect: Completing part of insulin administration process for the new nurse does not guarantee the error will not be repeated. In fact, there is no way to be sure the new nurse understands the specifics of injections either. This individual needs to be instructed, supervised and evaluated on all parts of the insulin administration process, including injections. There may be more errors in the process that need addressed.

A nurse is feeding a client diagnosed with a stroke who is exhibiting dysphagia. Which action by the nurse would be appropriate? 1. Elevate the head of the bed to 15 degrees. 2. Request the client to not hold food in their mouth. 3. Monitor for frequent throat clearing after eating. 4. Orient the client to the location of food on their plate.

3. Correct: When helping to feed a client with dysphagia, the nurse should monitor for signs of aspiration such as frequent throat clearing during and after meals. The client is trying to move the bolus of food down esophagus. Aspiration is a condition where food, liquids or saliva moves into the lungs instead of the esophagus during eating. 1. Incorrect: The client should be sitting upright or a high-fowler's position. This position allows for more flexibility of neck movement to promote swallowing. The nurse can keep the client's neck in the neutral position or their chin lowered to their chest. 2. Incorrect: Why is the client holding food in their mouth? Are they disoriented or with a cognitive impairment? Asking a client who is disoriented or with a cognitive impairment to not hold food in their mouth is not an effective intervention. 4. Incorrect: Does the location of the food on the plate affect how a client swallows the food. The location of the food will not affect the possibility of the client aspirating during swallowing food.

Which task would be appropriate for the nurse to assign to an unlicensed assistive personnel (UAP)? Select all that apply 1. Clean client's halo fixation insertion sites with hydrogen peroxide. 2. Insert acetaminophen suppository in client's rectum. 3. Reapply pneumatic compression device to client's legs. 4. Check client's gag reflex prior to feeding. 5. Set up suction equipment in client room. 6. Reposition client every 2 hours.

3., 5., & 6. Correct: The UAP is trained on use of routine equipment such as pneumatic compression devices and can reapply the device to a client. Gathering needed equipment and supplies is within the scope of duties for the UAP. Repositioning a client every 2 hours is within the UAP's ability and can be assigned by the nurse. 1. Incorrect: The UAP can provide routine hygiene. The nurse would be responsible for wound care, including halo insertion pin site care. This requires skill beyond the UAP's knowledge. 2. Incorrect: The UAP cannot administer medications. 4. Incorrect: The UAP cannot assess or evaluate a client. The RN most do this part of the nursing process.

A client awaiting discharge for a broken left tibia is to be sent to physical therapy for crutches and crutch walking. The client reports having brought a pair of crutches borrowed from a family member. What is the most appropriate action for the nurse to take now? 1. Cancel physical therapy and allow client to leave. 2. Ask client to stand with crutches to check the size. 3. Tell client insurance will not permit use of old crutches. 4. Send client with crutches to physical therapy for evaluation

4. CORRECT. The physical therapy department is best qualified to assist a client in adjusting to the use of crutches prior to discharge. Because the client wants to use older crutches, it is even more important for a physical therapist to determine whether it is safe for the client to do so. Physical therapy can evaluate the condition of the old crutches, the client's ability to manage that equipment and to walk safely with those crutches. 1. INCORRECT. It is permissible for a client to use previously owned medical equipment. However, the stability of that equipment and the client's ability to use the equipment safely must be evaluated by physical therapy. Cancelling physical therapy would also violate the physician's orders and place the client at risk for injury upon discharge. 2. INCORRECT. While the nurse may be able to adjust the old crutches to the client's height, crutch safety and walking should be evaluated by physical therapy to be certain the previous equipment is appropriate. 3. INCORRECT. Insurance does not designate whether assistive medical devices can be reused by clients or if a new device must be purchased. It is cost effective to reuse durable medical equipment if it is appropriately suited to the client's current needs.

A terminal client begins reminiscing about the past, expressing grief and regret over life choices. What response by the nurse would best help the client cope at this time? 1. "You can't change the past so try not to dwell on it." 2. "Would you like me to call a priest for you to talk with?" 3. "You still have time to make amends if you want." 4. "I can sit here with you while you continue to talk."

4. CORRECT: Anytime a client expresses the desire to talk, the nurse should respond with an open-ended response, encouraging the client to continue to verbalize in a non-judgmental environment. More importantly, the nurse should remain with the client, even if there is no talking, to provide visual comfort. 1. INCORRECT: This non-therapeutic response denies the client's right to review past events or express feelings, which is a normal reaction at end of life. The nurse's closed response does not provide the client with the opportunity to verbalize. 2. INCORRECT: The nurse is ignoring the client's need to talk and is transferring care away to another individual, even if that individual is a clergyman. This is an incorrect action. 3. INCORRECT: While the client may regret some life choices, there is no mention of the desire or need to correct the past. The nurse is making an assumption.

Prior to removal of cataracts, the client is to receive eye drops in both eyes. The nurse knows what action takes priority? 1. Remove any exudate around eyes with warm water. 2. Instill exact number of drops into lower conjunctival sac. 3. Instruct client to look upward when drops are instilled. 4. Avoid dropping the medication directly on the cornea.

4. CORRECT: The most important safety consideration when instilling eye drops is to avoid dropping the medication directly onto the cornea. The extreme sensitivity of the cornea before, and after, eye surgery could cause serious eye problems if meds were dropped onto the cornea. 1. INCORRECT: It is important to clean away any exudate prior to instilling eye drops to maintain aseptic technique and decrease chance of infection. Though this is an important action, there is another task which takes priority. 2. INCORRECT: Instilling the exact number of drops is appropriate when implementing written prescriptions from the primary healthcare provider. This is an important nursing action but not the priority. 3. INCORRECT: Instructing the client to look upward helps prevent drops from running out of the eye but there is another issue more important.

An elderly client is to be ambulated for the first time following a hip replacement. The client refuses to get out of bed, indicating an extreme fear of falling. What statement by the nurse is most therapeutic? 1. "Don't be afraid because I will not let you fall." 2. "Your doctor says you must walk twice today." 3. "I'll get another nurse to help so you won't fall." 4. "What worries you most about getting out of bed?"

4. CORRECT: The nurse needs to focus on the client's psychological as well as physical needs. An open-ended question or statement encourages the client to elaborate and share concerns that the nurse needs to address. It would be inappropriate to force the client to participate in an activity that causes extreme fear and distress. 1. INCORRECT: The nurse is dismissing the client's right to experience a specific emotion, rather than actively seeking the reason behind those feelings. The nurse is not utilizing appropriate communication techniques. 2. INCORRECT: This tactless response focuses on the orders provided by the primary healthcare provider, rather than the client's expressed concerns. Such a comment by the nurse is non-therapeutic because it ignores the client's psychological needs. 3. INCORRECT: Although the nurse offers a solution to the client, there is no chance for the client to verbalize feelings and concerns. It is more important to present the client with the therapeutic opportunity to discuss fears.

A child with acute lymphocytic leukemia (ALL) is receiving chemotherapy through a single lumen Groshong catheter. During the infusion, the child reports nausea and has vomited. The primary healthcare provider has prescribed ondansetron IV. What action should the nurse take? 1. Ask primary healthcare provider for an oral antiemetic. 2. Give ondansetron IVPB with the chemotherapy. 3. Wait until chemotherapy is complete to infuse ondansetron. 4. Stop chemotherapy temporarily and flush line to give ondansetron.

4. Correct: A Groshong catheter is implanted when other venous access sites are no longer useable. The child has begun to react to the chemotherapy and needs medicated now. Because this implanted device has only one lumen, the nurse must stop the chemotherapy infusion temporarily, flush the port, administer the ondansetron, flush again and restart the chemotherapy infusion. 1. Incorrect: Because this client is vomiting, changing the medication to the oral route would not be effective. The medication takes longer to work if given orally, which means the client may vomit again before the medication activates, losing part of the dose. 2. Incorrect: Chemotherapy infusions should not be mixed with other categories of drugs, such as an antiemetic, because of the possibility of drug interactions. Certain chemical mixtures could also cause precipitates to form in the tubing, which is dangerous to the child. 3. Incorrect: The child is experiencing nausea and vomiting at this time. Waiting to give the antiemetic until after the chemotherapy is completed causes the child to suffer needlessly. The nurse should take action immediately to alleviate symptoms.

The nurse is talking to the parent of a 3 year old child who was constipated 1 week earlier. The child is on a regular diet. What statement by the parent indicates to the nurse that the prescribed treatment for constipation has been effective? 1. "My child drinks 1000 mL of fluids daily." 2. "My child is eating more fruit every day." 3. "I administered the prescribed oil-retention enema 6 days ago to my child." 4. "My child has had a soft, formed, brown stool every day for 6 days without straining."

4. Correct: A client is assessed as constipated when they have hard stools, difficulty passing the stool, and incomplete passage of stool. The desired outcome of constipation therapy is for the child to have soft, formed, bowel movements. The mother states that the child has had soft, formed, brown stools every day for the last 6 days. 1. Incorrect: Can you evaluate the effectiveness of the treatment plan by the parent describing how much fluid the child consumes in 24 hours? No. This is an action based on the need to increase the fluid intake to help decrease constipation. This is not an evaluation of the prescribed treatment for constipation. 2. Incorrect: This statement is a description of the classification of food that the child is eating. The effectiveness of the prescribed plan for the constipation is not addressed. 3. Incorrect: The mother is describing the completion of a prescribed treatment. The results of the enema are not listed, and it does not evaluate the prescribed treatment for constipation.

How would a case manager best describe a clinical pathway to nursing students? 1. A decision-making flowchart that uses the if/then method to address client responses to treatment. 2. A set of practice guidelines developed by a professional medical organization such as the American College of Surgeons. 3. A standardized set of preprinted primary healthcare provider prescriptions for client care, which expedite the prescription process and can be customized to individual clients. 4. A set of client care guidelines based on a specific client diagnosis, which provides an overview of the multidisciplinary plan of care.

4. Correct: A clinical pathway is a set of multi-disciplinary client care guidelines for a specific diagnosis or condition. It can be used to guide the plan of care and to identify deviations from the plan of care. These clinical pathways reduce the degree of variation in clinical practice, improves outcomes, and promote organized and effective client care based on evidence based practice. Clinical pathways are different from algorithms, practice guidelines, and protocols because they incorporate a multidisciplinary team approach and focus on coordination and quality of care. 1. Incorrect: A decision-making flowchart that uses the if/then method is the definition of an algorithm. The algorithm direction changes based on the information gained at each level of the algorithm, so decisions for actions will be different. 2. Incorrect: A set of practice guidelines developed by professional medical organizations is the definition of a practice guideline. These guideline assist in decision making about appropriate healthcare for specific clinical situations but are not fixed protocols that are designed to be followed in an exact manner. They are recommendations for consideration. The practice guidelines are specific to practice areas rather than having a multidisciplinary approach. 3. Incorrect: A standardized set of preprinted primary healthcare provider prescriptions. These preprinted prescriptions are available for immediate access and use with clients, include commonly prescribed interventions, and reduces oversight of interventions by having a standardized format. Other advantages have also been identified for the use of preprinted prescriptions.

A client is admitted with irritable bowel syndrome (IBS) and shingles. The nurse is discussing the client assignments with the charge nurse. Which staff member should not be assigned to this client? 1. The nurse with a history of roseola. 2. The unlicensed assistive personnel (UAP) with no history of roseola. 3. The UAP with a history of chickenpox. 4. The LPN/LVN with no history of chickenpox.

4. Correct: A nurse who has not had chickenpox could contract it and should not be assigned a client with shingles. Those who have not developed antibodies to the varicella zoster virus are susceptible to chickenpox. Chicken pox and shingles are both from the varicella virus. 1. Incorrect: Roseola is a rose colored rash and would not have any effect on the assignment. It is a generally mild infection that usually affects children by age 2, and rarely adults. It is caused by 2 strains of herpes virus, rather than the varicella virus. 2. Incorrect: Roseola is a rose colored rash and would not have any effect on the assignment. It is a generally mild infection that usually affects children by age 2, and occasionally adults. It is caused by 2 strains of herpes virus, rather than the varicella virus. There is no relationship between roseola and shingles. 3. Incorrect: Shingles is caused by a reactivation of the varicella-zoster virus (which causes chicken pox). Those who have not developed antibodies to the varicella-zoster virus are susceptible to chickenpox. Therefore, the UAP who had chicken pox could be assigned this client.

A client is awake in the recovery room following a cardiac catheterization performed through the left radial artery. During the assessment, the nurse notes severe swelling of the left upper arm with a diminished left radial pulse, indicating an internal arterial hemorrhage. The cardiologist states the client will require immediate surgery to repair the leaking artery. The nurse understands what fact about the current consent form? 1. Can be assumed since it's an emergent situation. 2. Should be signed by client who is currently awake. 3. Is not needed since client consented to catheterization. 4. Must be approved by family or a spouse.

4. Correct: An additional procedure requires a new consent form which describes specifically what the cardiologist plans to do. Even though the client is awake, residual sedation from the catheterization makes it necessary for a family member or spouse to sign the consent form. 1. Incorrect: Emergent situations are those in which the client's life or limb is threatened. That type of consent is called "implied" consent; however, despite the seriousness of the situation, implied consent is not valid in this case. 2. Incorrect: Though awake following the catheterization, the client is considered impaired because of the sedation used during the catheterization. Even if the client understands what is occurring, a signature by the client is not considered legal at this time. 3. Incorrect: Once the surgery and potential risks are explained to the client, a consent form is completed specifically describing the procedure to be performed by the cardiologist. That form does not cover any additional procedures, even if directly connected to the original surgery.

The parents of a child hospitalized with cystic fibrosis have been given discharge instructions. The nurse knows that teaching has been successful when the parents make what statement? 1. "Our child will need to have a gluten free diet." 2. "The enzymes should be given at bedtime daily." 3. "Salt needs to be decreased in our child's diet." 4. "We need to prepare high calorie, high fat meals."

4. Correct: Cystic fibrosis is an inherited disorder in which abnormally viscous secretions affect the respiratory and digestive systems. Because the client is unable to absorb nutrients, several dietary adaptations are crucial, including frequent small meals along with digestive enzymes to help the client process food. The meals should be high calorie, high fat with increased amounts of sodium to help stabilize fluids. 1. Incorrect: A gluten free diet is not associated with cystic fibrosis. This special diet is generally required for clients with Celiac disease and certain food allergies, although clients with either of these diseases will need the addition of fat soluble vitamins A, D, E and K. This statement by the parents indicates the need for further teaching. 2. Incorrect: Pancreatic digestive enzymes, such as Creon or Pancreaze, must be given with every meal or snack in order to help the digestive system absorb nutrients properly. Because clients with cystic fibrosis need frequent small meals throughout the day, digestive enzymes must also be provided throughout the day with any food. 3. Incorrect: Clients with cystic fibrosis lose abnormally large amounts of salt in sweat, and the glands are unable to reabsorb needed sodium into the body system. Rapid dehydration is common due to decreased sodium levels, which are exacerbated during exercise or hot weather. These clients are encouraged to increase salt intake.

The primary healthcare provider has prescribed ampicillin and ciprofloxacin piggyback in the same hour, every 6 hours. How will the nurse administer these medications? 1. Administer one of the medications every 4 hours and the other every 6 hours. 2. Administer the medications by combining them into 150 mL of normal saline (NS). 3. Administer the medications at the same time by connecting the secondary tubing to two separate ports on the primary tubing. 4. Administer the medications separately, flushing with normal saline (NS) between medications.

4. Correct: Even though two IV piggyback medications have been ordered at the same time, they can both be infused separately on time. It just takes planning. The nurse must follow the medication rights (right client, right medication, right route, right dose, right time). The antibiotics need to be administered one at a time and normal saline is used to flush the remaining medication of the first antibiotic before the second is administered. 1. Incorrect: The primary healthcare provider will prescribe the dosing schedule. Its beyond the scope of practice for the nurse to independently the dosing schedule. 2. Incorrect: The properties of each antibiotic are different. The two different antibiotics cannot be mixed together. 3. Incorrect: Administering the antibiotic into different parts of the IV tubing is the same as mixing the IVs together. Only one antibiotic should be administered at a time.

A client has been prescribed a decongestant. The nurse identifies that the client has a diagnosis of glaucoma. Which nursing intervention would the nurse implement after identifying the client's diagnosis of glaucoma? 1. Administer the decongestant. 2. Reassess the client in 4 hours. 3. Identify when the client was diagnosed with glaucoma. 4. Notify the primary healthcare provider regarding the glaucoma diagnosis.

4. Correct: The primary healthcare provider should be notified of the client's diagnosis of glaucoma. Glaucoma is the result of elevated eye pressure due to a buildup of aqueous humor that flows throughout the inside of your eye. Decongestants can cause the pupil to dilate. This response can result in an acute glaucoma attack in a client diagnosed with narrow-angle glaucoma or angle-closure glaucoma. 1. Incorrect: The primary healthcare provider should be notified of the client's diagnosis of glaucoma and the prescription for a decongestant. The medication should not be administered until the primary healthcare provider is consulted. 2. Incorrect: Continual assessment of the client is recommended, but the nurse should not delay consulting with the primary healthcare provider. The primary healthcare provider should be notified of the client's diagnosis of glaucoma and the prescription for a decongestant. 3. Incorrect: The nurse needs to determine whether to administer the decongestant due to the client's diagnosis of glaucoma. The nurse does not need to identify when the client was diagnosed with glaucoma.

Which action by an unlicensed nursing assistant would require the nurse to intervene? 1. Collecting I & O totals for unit clients at the end of shift. 2. Elevating the head of the bed 30°- 40° for the client post thoracotomy 3. Ambulating a client who is 2 days post vaginal hysterectomy 4. Turning off continuous tube feeding to reposition a client, then turning the feeding back on

4. Correct: The unlicensed nursing assistant should not turn tube feedings off or on. The nurse should do this when repositioning is needed. Prior to turning feeding back on, tube placement needs to be verified. 1. Incorrect: Obtaining the urinary output of a client at the end of the shift is appropriate for the nursing assistant and should be documented and reported to the RN. 2. Incorrect: This is appropriate because this position will improve gas exchange and breathing for a client after thoracic surgery. 3. Incorrect: The hysterectomy client needs to be ambulated to avoid post op complications. This is an appropriate and safe action for the unlicensed nursing assistant to do.

In what order should the nurse assess assigned clients following shift report? Place in priority order. Client diagnosed with Crohn's disease who had three semi-formed stools over the past shift. Client diagnosed with hemorrhoids who had some spotting of bright red blood on toilet tissue with last bowel movement. Client diagnosed with gastroenteritis who reported 300 mL diarrhea stool x2 in the last hour. Elderly client admitted 30 minutes ago with reports of constipation for four days. Drag and Drop the items from one box to the other

All these clients have a GI problem. So, now you must decide which of these high priority clients should be seen in what order. The first client the nurse needs to assess is the elderly client admitted 30 minutes ago with reports of constipation for four days. This is an elderly client who is a new admit. The client reports constipation for 4 days which may be an indication of worse problems. The client is considered unstable until assessed by the nurse. The second client the nurse needs to see is the client diagnosed with gastroenteritis who had two 300 mL diarrhea stools in one hour. Did you think dehydration and fluid volume deficit? The third client that should be assessed by the nurse is the client diagnosed with hemorrhoids who had some spotting of bright red blood on toilet tissue with last bowel movement. This is normal for clients with hemorrhoids. But the client does need to be assessed prior to the client with Crohn's disease who is improving. The fourth client the nurse should assess is the client diagnosed with Crohn's disease who had three semi-formed stools over the past shift. Semi-formed stools are great news! The client is getting better. During exacerbation, the client will have many diarrhea stools.

A client was admitted with a diagnosis of Type II diabetes. The primary healthcare provider initiated the Insulin Sliding Scale Protocol for Type II Diabetic Clients. The prescription regimen was to begin at the high dose regimen with regular insulin AC & HS. How much insulin should the nurse administer at 2100 hours? Exhibit: Glucose flow sheet: Date/Time: 1/19 @ 0730, Glucose Level: 368 mg/dL, Insulin Dose Regimen: High dose regimen, Insulin Dose: 16 units regular insulin Date/Time: 1/19 @ 1130, Glucose Level: 256 mg/dL, Insulin Dose Regimen: High dose regimen, Insulin Dose: 12 units regular insulin Date/Time: 1/19 @ 1700, Glucose Level: 164 mg/dL, Insulin Dose Regimen: High dose regimen, Insulin Dose: 4 units regular insulin Date/Time: 1/19 @ 2100, Glucose Level: 248 mg/dL, Insulin Dose Regimen: ___________, Insulin Dose: ___________ Sliding Scale Insulin Protocol: Regimens: Low Dose Regimen: Suggested as starting for the thin and elderly. Medium Dose Regimen: Suggested as starting point for average weight. High Dose Regimen: Suggested as starting point for overweight clients. Very High Dose Regimen: Suggested as starting point for clients with infections or receiving steroids. Insulin Type: Regular Insulin Frequency of Monitoring: ACHS Protocol: 1. If Potassium is <3.5, call M.D. 2. Advance to the next higher dose regimen if glucose level is >250 two (2) times in 24 hours and all readings were > 100. 3. Decrease to the next lower dose regimen if glucose level is between 60-100 twice in 24 hours. Glucose Level (mg/dL): FSBS < 60, Low Dose Regimen: Hypoglycemia Protocol and Call MD, Medium Dose Regimen: Hypoglycemia Protocol and Call MD, High Dose Regimen: Hypoglycemia Protocol and Call MD, Very High Dose Regimen: Hypoglycemia Protocol and Call MD Glucose Level (mg/dL): 60-150, Low Dose Regimen: 0, Medium Dose Regimen: 0, High Dose Regimen: 0, Very High Dose Regimen: 0 Glucose Level (mg/dL): 150-200, Low Dose Regimen: 0, Medium Dose Regimen: 2, High Dose Regimen: 4, Very High Dose Regimen: 6 Glucose Level (mg/dL): 201-250, Low Dose Regimen: 3, Medium Dose Regimen: 6, High Dose Regimen: 8, Very High Dose Regimen: 10 Glucose Level (mg/dL): 251-300, Low Dose Regimen: 4, Medium Dose Regimen: 8, High Dose Regimen: 12, Very High Dose Regimen: 14 Glucose Level (mg/dL): 301-350, Low Dose Regimen: 6, Medium Dose Regimen: 10, High Dose Regimen: 14, Very High Dose Regimen: 18 Glucose Level (mg/dL): 351-400, Low Dose Regimen: 9, Medium Dose Regimen: 12, High Dose Regimen: 16, Very High Dose Regimen: 22 Glucose Level (mg/dL): > 400, Low Dose Regimen: 9 units and Call MD, Medium Dose Regimen: 12 units and Call MD, High Dose Regimen: 16 units and Call MD, Very High Dose Regimen: 22 units and Call MD

Answer: 10 Rationale: Prescription: The prescription regimen was to begin at the high dose regimen with regular insulin AC & HS using the Insulin Sliding Scale Protocol for Type II Diabetic Clients. The protocol states to advance to the next higher dose regimen if glucose level is greater than 250 two (2) times in 24 hours and all readings are greater than 100. All glucose readings were greater than 100, and the readings were greater than 250 three times. So, the client should move to the next highest dose regimen which indicates that 10 units of regular insulin should be given at 2100 hours for a glucose of 248.

Shortly after admitting a client to the unit, the nurse prepares to start an IV and hang 0.9% normal saline solution. At what rate should the nurse set the IV infusion pump? Answer using numbers only. Exhibit Primary Healthcare Provider Prescriptions: Physician Orders: 1. Admit to Cardiac Stepdown Unit with telemetry monitoring. 2. Oxygen at 2 L/min. 3. Monitor oxygen saturation. 4. Morphine IV 2 mg every 2 hours PRN shortness of breath. 5. Furosemide IV 40 mg every 6 hours. 6. Dobutamine 5 mcg/kg/min IV. 7. 0.9% Normal Saline Solution IV at 150 mL/hour. 8. Maintain bedrest except for ambulation to bathroom. 9. Elevate head of bed 30 degrees. 10. Monitor vital signs hourly.

Answer: 150 Rationale: Prescription: 0.9% normal saline solution IV at 150 mL/hour. Think, when an infusion pump is used, the flow rate is prescribed by the primary healthcare provider and programmed by the nurse by setting the device for milliliters per hour (mL/h). Rule - To regulate an IV by infusion pump, Total mL prescribed/Total hr prescribed = mL/hr If you forgot that, then work the formula for gtts/min - mL/hr x drop factor/time in minutes = gtts/min The drop factor for the infusion pump is 60. 150 mL x 60 / 60 minutes = 150 Therefore, the pump should be set at 150 mL per hour

The nurse is preparing to give a client's prescribed azithromycin dose. How many tablets will the nurse give to the client? Answer with numbers only. Exhibit Primary Healthcare Provider Prescription: Azithromycin 1 gram by mouth times one dose now Medication Cart Drug Availability: Zithromax (azithromycin) tablets 500 mg

Answer: 2 Rationale: Prescription: Azithromycin 1 gram by mouth times one dose now Available: Azithromycin 500 mg/tablet Step 1 is to convert grams to mg (1 gram = 1000 mg) Step 2: Think, 1 tablet is 500 mg and you need to give 1000 or twice the amount that is available. Step 3: D/H x Q = X 1000mg/500 mg x 1 tablet = 2/1 = 2 tablets

The nurse is preparing to give 250 mL D5W IV over 2.5 hours. How many gtts/min will the nurse need to set the IV rate at? Exhibit IV Tubing: 15 Drops/mL

Answer: 25 Rationale: Prescription: 250 mL D5W IV over 2.5 hours. Step 1: Determine what the drop factor is on the IV tubing. The one shown in the exhibit has a drop factor of 15 drops/mL. Step 2: Remember the formula for gtts/min - mL/hr x drop factor/time in minutes = gtts/min Step 3: 250 ml/2.5 hours x 15 gtts/1 mL x 1 hour/60 min = 250/2.5 x 15/1 x 1/60 = 3750/150 = 25 gtts/min

The primary healthcare provider has prescribed ear irrigation for a client with earwax accumulation. In what order would the nurse perform the procedure? Remove any debris in the outer canal. Squeeze syringe with moderate force. Aim syringe at back side of ear canal. Tilt client's head to the opposite side. Fill bulb syringe with lukewarm water. Pull ear pinna upward and backward. Drag and Drop the items from one box to the other

Ear wax is a substance produced by the body to clean and protect the ear canal. Usually, the ear is able to remove wax that has dried up and become useless. However, in certain circumstances, a nurse may receive a prescription to irrigate ears for clients when the wax has become impacted. After gathering the needed equipment and explaining the process to the client, the nurse should first fill the bulb syringe with warm water. Next, positioning the client is always the nurse's responsibility. For irrigating ears, the client's head should be tilted slightly toward to opposite side, so the affected ear is easily accessible. Third, using one hand, the nurse pulls the ear pinna gently upward and back to straighten the ear canal. Fourth, aim the bulb syringe toward the back and side of the ear canal. Next, the nurse squeezes the syringe with moderate force to flush out loose debris or dried wax. Sixth, any debris visible in the outer canal can be carefully removed with tweezers.

A mass casualty disaster has occurred and clients are being received at the emergency department. In what order should the nurse assess these clients? Sort from highest priority to lowest priority. Client with a 4 inch (10.16 cm) laceration to the lower leg with moderate bleeding. Client with blunt trauma to the spine that is unable to move extremities. Client with traumatic amputations with agonal respirations. Client with an open chest wound that is beginning to show signs of tracheal deviation. Drag and Drop the items from one box to the other

The client with an open chest wound should be seen first. This client is one whose life could potentially be spared if lifesaving measures are taken. This client may be developing a tension pneumothorax and may need an immediate needle decompression. The client would also need a dressing that is taped down on 3 sides applied over the open chest wound. The second client to be seen is the one with blunt trauma to the spine. Although this client needs emergency treatment as soon as possible due to having probable spinal injury with paralysis, this client's condition is not likely to deteriorate as fast as the client with the open chest wound who is developing a tension pneumothorax. The third client to be assess by the nurse should be the client with the laceration. Did you see laceration with bleeding and think that something would have to be done immediately? Well, there is only moderate bleeding, so although this client needs obvious treatment, this client can wait and would not be a priority over the clients with the open chest wound and blunt trauma to the spine. The last client to be assessed should be the client with agonal respirations. Although this client is still alive, during a mass casualty, the nurse would recognize that the client has agonal respirations and would not have a very good chance of survival with intervention. This client would not take priority over a critical client who has a better chance of survival.

A nurse is caring for a group of clients and is considering the risk of infection for each. Place the client conditions in rank order from the highest to least potential for infection. Indwelling foley catheter inserted the previous day Thermal burns covering 30% of body surface area (BSA) 2 days ago Total hip prosthetic device placement 3 days ago Laparoscopic exploration of right knee 2 days ago Drag and Drop the items from one box to the other

The client with the greatest risk of infection would be the client with thermal burns covering 30% of the BSA. Burns are considered contaminated wounds. Normally, skin provides a natural barrier against invasive microorganisms. However, with this major burn injury, the client is predisposed to infection as a result of the loss of skin integrity. Additional factors that will place this client at higher risk for infection include the development of eschar, which bacteria loves to live in, and the fact that thermal injuries alter the body's natural immunity. So, are the clients with the other conditions at risk for infection? Well, they could be, but the risk is not as great. Let's consider why the risk is less. The client with the total hip arthroplasty (replacement of the damaged hip with a prosthetic device implanted) would be the next highest in ranking for risk of infection. This client has a relatively large surgical incision and a prosthetic device that infection, when present, tends to migrate to the area. But, this type surgery is performed using sterile technique in sterile environments to minimize the risk of infection. In addition, any dressing changes should be performed using sterile technique. The next client at risk of infection would be the client with the laparoscopic exploration of the right knee. Again, there is surgical perforation of the skin. However, these are smaller puncture sites that are created under sterile conditions, and when cared for appropriately, do not carry a high risk for infection. Finally, the client who has the indwelling foley catheter is the least at risk for infection. The catheter is a portal of entry into the body, but if inserted using sterile technique and proper catheter care is provided, the risk of infection can be kept to a minimum. The longer the foley catheter remains in place, the risk of infection will increase.

A roommate overhears the primary healthcare provider discussing a client's laboratory results, including a positive HIV test. The roommate requests to be moved immediately to another room. In what priority order should the nurse complete these tasks? Encourage the client to verbalize feelings regarding situation. Educate roommate about transmission of HIV and AIDS. Contact social services to address client's future needs. Transfer roommate to another location as soon as available. Notify nurse manager regarding breach in confidentiality. Drag and Drop the items from one box to the other

The first action by the nurse is to address the roommate's concerns and fears about contracting HIV by presenting information regarding disease transmission. Next, since this situation represents a definite breach of confidentiality, it must then be reported to the nurse manager. Third, despite educating the roommate on modes of transmission, the nurse should attempt to honor the request to be moved to another room. Fourth, address the roommate's needs, by encouraging the client to express feelings about the diagnosis and current situation. Additionally, the client will have other needs related to the diagnosis which can best be handled by the social services department.

Four clients arrive for their appointment at a diabetic clinic. In what order should the nurse see the clients? Client eating a simple-carb snack due to weakness. Client scheduled for a dressing change to foot ulcer. Client to receive dietary education. Client reporting a headache and has a fruity breath. Drag and Drop the items from one box to the other

The first client needing the nurse's attention is the one reporting a headache and has a fruity odor to their breath. Remember, pick the killer answer first! This client is likely in metabolic acidosis due to diabetic ketoacidosis (DKA). What was the hint? Fruity breath. The second client that needs to be seen by the nurse is the client having weakness, a sign of hypoglycemia. This is a diabetic clinic. This client is eating a simple carb snack, but the nurse needs to check the client's blood glucose level to see if the snack has helped. The third client would be the one needing a dressing change. Nothing life threatening, but an assessment needs to be made regarding the ulcer. The last client would be the one needing dietary education. Nothing life threatening. This client can wait until the others are treated.

In what order should the emergency department triage nurse send these clients to a room for treatment? Place in priority order. Client who has multiple injuries from a motor vehicle accident. Elderly client who fell and fractured the left femoral neck. Client reporting epigastric pain and nausea after eating. Female client stating she has been raped. Drag and Drop the items from one box to the other

The first client who needs treatment is the one with multiple injuries from a motor vehicle accident. Injuries from a motor-vehicle accident can be life threatening. The client should be assessed first to rule out respiratory difficulty and hemorrhage. The second client that should be given a treatment room is the elderly client who fell and fractured the left femoral neck. Elderly clients have special fluid and electrolyte issues after a fall. The cause of the fall may be cardiac, but the question does not indicate this. The third client that should be sent back for treatment is the female client stating she has been raped. We do not know the extent of her injuries based on what the option tells us. There will likely be both physical and emotional injury that needs attention, which places this client third. The last client that should be sent back for care is the client experiencing epigastric pain and nausea after eating. This is likely cholelithiasis, which will need to be checked out. This is the most stable of the four clients which places this client last to be seen.


Set pelajaran terkait

romanticism and transcendentalism (11/6)

View Set

American History Since 1877 Exam 1 Review: Quizzes (ch. 17-20)

View Set

UWorld Safety and Infection Control Part 1

View Set

Finance 3715 Exam 1 Chapters 1-3

View Set

Part 3: Oral Surgery, Endodontics, Pediatrics, Orthodontics, Behavior Science, Public Health, OSHA & Infection Control

View Set

Brunner Nursing Concept - Perfusion

View Set